Naked Science Forum

Non Life Sciences => Physics, Astronomy & Cosmology => Topic started by: londounkm on 20/10/2010 10:52:06

Title: What is Time? If there was no light would Time cease to be?
Post by: londounkm on 20/10/2010 10:52:06
Please excuse me if I am asking a stupid question, but I am trying to understand exactly what time is. When I think about time I immediately link it to light. Astronomers often tell us that they are able to take pictures of the universe billions of years in the past and that this is due to the amount of time it takes for light to travel. As I understand it, if I am looking up into the night sky what I am seeing is actually the past, in the case of the sun light I am seeing approximately 8 minutes into the past where as looking at the stars I could be looking many millions of years into the past. This is where I start to get boxed in with my own limited knowledge and perhaps some of you more knowledgeable people could enlighten me. If there was no light would time cease to exist altogether? I appreciate anyone taking the time to help me try and understand this.

Graham
Title: What is Time? If there was no light would Time cease to be?
Post by: Joe L. Ogan on 20/10/2010 16:58:17
Time is in three parts:  Future-which never arrives and exists only in ones' imagination.  Present-which exists only for an infinitesimal split second and Past which goes on forever in ones' memory but never returns except as mistakes which we make and are inclined to repeat.  Thanks for comments.  Joe L. Ogan
Title: What is Time? If there was no light would Time cease to be?
Post by: Bill S on 20/10/2010 17:57:44
Quote from: londounkm
Please excuse me if I am asking a stupid question, but I am trying to understand exactly what time is.

Where understanding time is concerned, I think there is no such thing as a stupid question. St Augustine shared our problem and expressed it well: "What then is time? If no one asks me, I know; if I wish to explain it to someone who asks, I know not".


Title: What is Time? If there was no light would Time cease to be?
Post by: raghusesha on 20/10/2010 18:42:52
Here is what my limited knowledge says:
There was a similar question on sound in one of the naked scientist shows and the question was that if a tree falls in a forest and no one is around to hear the sound of tree fall, can we conclude that the tree did not make a sound while falling? Answer is obvious.
Light consists of particle traveling at a fixed speed (thanks to relativity theory).  If there is nothing happening on stars to produce light, the stars still exist in time and time does pass.
From my very little knowledge of relativity theory, speed of light is a upper bound for the velocity of any particle. Electromagnetic waves travel at this speed.  So, if none of these electromagnetic waves existed, will time exist?  My guess is yes, it will.  Only that a small set of people on a planet like ours may not be able to "see" the rest of the universe.
Title: What is Time? If there was no light would Time cease to be?
Post by: Soul Surfer on 20/10/2010 19:02:44
A difficult question because it depends a bit precisely what the questioner means.  If you just mean would time cease to exist if you could not see anything and could only be aware of things by touching them.  groping around an drecognising things by feel requires a sense of time so clearly time exists.

However most of the interactions of which we are aware of are mediated by electromagnetic radiation and light is one particular form of electromagnetic radiation and without this no electromagnetic interactions could happen.  This would still leave other sorts of interactions notably gravity by which changes in the environment could be sensed so on the whole Time appears to be a fundamental part of the sort of universe we can understand.
Title: What is Time? If there was no light would Time cease to be?
Post by: Murchie85 on 21/10/2010 01:58:48
Personally I think time is poorly understood simply because its a man made concept and not an actual physical phenomena that can  be captured in its entirety or leave a mark on the universe.

It is relative, it is changing for different observers, it is never agreed upon by all parties and it certainly doesn't have a quantity that can be defined with universal precision (atomic clocks using cesium atoms are not the last say) and may never be.

Time can be considered as a measurement of passing events it is limited only by the measurer and time its self leaves no trace in the physical world. Its not a piece of string in which people consider a time line that can be traced backwards or forwards like a video cassette.

Time is however important, it is connected with entropy in that we know we are going forward and not backwards in time. It helps us compare certain events with other events but in terms of the light, it does not require this to exist although light does require time to exist.

Adam
Title: What is Time? If there was no light would Time cease to be?
Post by: Geezer on 21/10/2010 05:32:59

not an actual physical phenomena that can  be captured in its entirety or leave a mark on the universe.


However, everything in the Universe is controlled by time.
Title: What is Time? If there was no light would Time cease to be?
Post by: Murchie85 on 21/10/2010 12:36:16
I disagree, time in my opinion is not a real entity, time can not control anything it does not "steer a series of events" it simply is a result we obtain from measuring the passing intervals of certain events for example.
Title: What is Time? If there was no light would Time cease to be?
Post by: Bill S on 21/10/2010 14:32:29
Quote from: Murchie85
it simply is a result we obtain from measuring the passing intervals of certain events

So far I agree in general with what you have said about time, but it is interesting to see how assumptions creep in.  E.g. the assumption that "intervals of certain events" are passing, rather than that the events are stationary, and we are doing the passing.

Nit picking? Perhaps; but whole books have been written about this.
Title: What is Time? If there was no light would Time cease to be?
Post by: londounkm on 21/10/2010 16:08:32
Firstly, thank you all very much for your thoughts and ideas it is much appreciated.

Quote
Time is in three parts:  Future-which never arrives and exists only in ones' imagination.  Present-which exists only for an infinitesimal split second and Past which goes on forever in ones' memory but never returns except as mistakes which we make and are inclined to repeat.  Thanks for comments.  Joe L. Ogan

What a lovely and concise description of time. I read in the paper this morning that the first observed galaxy to appear out of the big bangs white soup has been imaged by Hubble and confirmed by the VLT at appx 600 million years P.B.B(Post Big Bang).

Quote
Where understanding time is concerned, I think there is no such thing as a stupid question. St Augustine shared our problem and expressed it well: "What then is time? If no one asks me, I know; if I wish to explain it to someone who asks, I know not".

Haha brilliant, St Augustine has almost exactly described how I feel about time. Only when I really applied thought to time did I realise how little I understood it. It is certain to me that individualism and separation in society has all but destroyed the long conversations that used to take place on issues such as this. Websites like the Naked Science Forum are a blessing to those who wish to expand there understanding and knowledge of the things around them.

Quote
A difficult question because it depends a bit precisely what the questioner means.  If you just mean would time cease to exist if you could not see anything and could only be aware of things by touching them.  groping around an drecognising things by feel requires a sense of time so clearly time exists.

However most of the interactions of which we are aware of are mediated by electromagnetic radiation and light is one particular form of electromagnetic radiation and without this no electromagnetic interactions could happen.  This would still leave other sorts of interactions notably gravity by which changes in the environment could be sensed so on the whole Time appears to be a fundamental part of the sort of universe we can understand.

In all honestly I believed that light would exist even if light did not, because as you quite rightly pointed out if the light were "turned off" in the Universe we would still be able to grope around touching and feeling. In asking the question I guess I was trying to understand the nature of time. I understand that time is not a physical thing, such as a particle but rather a measurement applied to universe by an observer. When I read that an image was taken by Hubble appx 600 millions years after the big bang, what they really mean by that is that an image was taken over a distance of X billions of miles relative to our position in space and due to the time take for the light to reach us what are seeing is actually X number of years in the past. I also understand that whether and observer is observing or not time still exists as it is simply the measurement of before, now and next.

I think I am going to stop there for now and think some more on this topic. I really appreciate all your comments and suggestions and once again would like to say thank you very much.
Title: What is Time? If there was no light would Time cease to be?
Post by: Bill S on 22/10/2010 00:48:57
A few years ago, when I was trying to get my head round the idea of time travel, I found I was going round in circles.  My wife suggested I should write every thing down.  The following is a small extract, which might be relevant.

    One of the much discussed aspects of time is that of its apparent motion.  We talk of an arrow of time which we observe as progressing steadily from future to past, but does time actually progress in any direction.  We do not talk of an arrow of space.  We perceive space as static, and ourselves as moving through it.  Perhaps our perception of this would be different if our only experience was of being aboard a space craft, many light years away from our nearest celestial body; as we travelled steadily through empty space, for us, the only static object would be the space craft.  As long as our craft maintained a steady direction, we would, with time, observe the Universe drifting slowly past us.  Perhaps we would reason that space had a directionality of movement - an arrow of space.  The fact is that our experience, as far as space is concerned, is that we have many points of reference, the things about us which we have always regarded as static, so it is natural that we see ourselves as doing the moving.  Although we may consciously accept that these things, collectively, are in relative motion, for example, the Earth is moving round the sun, so everything on the Earth is in motion; our senses tell us that the things around us are stationary. We can see that individual objects are stationary relative to other objects.  Intuitively we know that when we move from room to room in our house, we are doing the moving, not the house.  When we get in the car and drive to town we know it is the car that is moving along the road, not the other way round.  Reason, as well as intuition, tells us that this is right,  because if it were space that was moving, our roads would have to move in many directions and at a range of different speeds at the same time in order to accommodate the observed flow of traffic.  Such is not the case with time; we have no static reference point, therefore we are able to perceive ourselves as static, and time as moving.  Relativity says that in the case of space, any point from which an observation is made has as much right as any other to be regarded as stationary.  If the same is true of time, it is natural that we would regard ourselves is stationary, and time as flowing.  This is reinforced by our perception of time flowing in exactly the same way past everyone and everything around us.  It makes equally sound sense, though, to regard time as a static entity, and to see ourselves as moving through time.  Unlike space, however, we all seem constrained to move through time in the same direction.
Title: What is Time? If there was no light would Time cease to be?
Post by: Geezer on 22/10/2010 03:06:02
I disagree, time in my opinion is not a real entity, time can not control anything it does not "steer a series of events" it simply is a result we obtain from measuring the passing intervals of certain events for example.

You are quite entitled to your opinion of course, but time actually does control all physical and chemical processes, and it has been demonstrated repeatedly that time is not a universal constant. That means that, relative to each other, physical and chemical processes may vary depending on their particular time frames.
Title: What is Time? If there was no light would Time cease to be?
Post by: Bill S on 22/10/2010 16:29:29
Quote from: Geezer
That means that, relative to each other, physical and chemical processes may vary depending on their particular time frames.

Could that not be interpreted a little differently?  E.g. physical and chemical processes may appear to vary, depending on the F of R from which they are observed. Time is one of the things that seems to vary. 

Let's say I drop a small piece of calcite into a solution of HCl. My clock tells me how long it takes to dissolve.  An observer who perceives me to be in motion relative to her will observe a different time for the reaction.  Does this not mean that the reaction is independent of any objective measurement of time?
Title: What is Time? If there was no light would Time cease to be?
Post by: imatfaal on 22/10/2010 16:57:15
Bill - surely we have formulas which will allow all different frames of reference to come to the same conclusion. 

That is to say that every observer could, if he knew his own velocity relative to the chemical reaction frame, calculate the perceived reaction time of any other observer with a known relative velocity.  So the reaction time is variable - but definitely not subjective.  The practical definition of objective is that independent observers will agree, providing these observers know their velocity relative to the reaction frame of reference, they will all agree on the timing.
Title: What is Time? If there was no light would Time cease to be?
Post by: Bill S on 22/10/2010 17:41:10
Quote from: Imatfaal
That is to say that every observer could, if he knew his own velocity relative to the chemical reaction frame, calculate the perceived reaction time of any other observer with a known relative velocity.


I have no problem with every observer being able to calculate the time in every other observer's F of R.  My point is that, according to S R, every observer has the right to consider her/his time to be as correct as any other's.  There is no objective time, unless we say that the time measured in the F of R in which the experiment is conducted is given special status.
Does S R allow that? 
Title: What is Time? If there was no light would Time cease to be?
Post by: imatfaal on 22/10/2010 17:50:01
Not sure I agree with that - SR allows all time to be objective, none are given special status, every reference frame can be transformed to another - if we all agree on something (ie the proper time in somesuch reference frame) then surely it is objective. 
Title: What is Time? If there was no light would Time cease to be?
Post by: Geezer on 22/10/2010 18:17:44
I always like to come back to the Hafele-Keating experiment because it does seem to confirm that there is no absolute time, and that processes are indeed controlled by local time (assuming I understand it correctly of course).

The multiple observers watching each other discussions (which are, I think, impossible to test anyway) always give me an 'orrible sore head  [:D]
Title: What is Time? If there was no light would Time cease to be?
Post by: simplified on 23/10/2010 15:20:10
A mass radiates gravitation for  receiving local time. Great gravitation streams   complicate reception of local time a little. A photon does not radiate gravitation therefore does not receive local time. However unknown time operates external movement of a photon. [;)]
Title: What is Time? If there was no light would Time cease to be?
Post by: Bill S on 23/10/2010 17:01:18
Quote from: Imatfaal
if we all agree on something (ie the proper time in somesuch reference frame) then surely it is objective.

If we all agree about the "proper time" in a given F of R, then, surely, we have also to agree on the "proper time" in all the other frames, and since these are all different, but all right, we have to conclude that time is subjective, in that it depends on the F of R.
Title: What is Time? If there was no light would Time cease to be?
Post by: imatfaal on 23/10/2010 21:15:23
subjective (http://en.wiktionary.org/wiki/subjective)
...
Formed, as in opinions, based upon subjective feelings or intuition, not upon observation or reasoning, which can be influenced by preconception; coming more from within the observer rather than from observations of the external environment..."

objective (http://en.wiktionary.org/wiki/objective) 
Of or relating to a material object, actual existence or reality.
Not influenced by the emotions or prejudices.
Based on observed facts."

As all observers will agree about proper time - and it has nothing to do with ideas and bias within the scientist but rather is based on observed facts; I would definitely say it was objective.  It may be strictly reliant on the FoR, but it doesn't depend on the opinion of the observer, it relies on a well defined fact about the observer.  Very few objective facts pop straight out from measurement - they need processing, not to massage the facts, but to bring all the different observations into a comparable form.   
Title: What is Time? If there was no light would Time cease to be?
Post by: Bill S on 24/10/2010 02:40:59
Thanks imatfaal. I was using "subjective" in the sense of relating to the inherent nature of a person or thing, in this case the individual F of R.

Wouldn't it be great to have a language in which each word had only one meaning.
Title: What is Time? If there was no light would Time cease to be?
Post by: simplified on 24/10/2010 17:22:52
Interesting
              http://casa.colorado.edu/~ajsh/schwp.html
Title: What is Time? If there was no light would Time cease to be?
Post by: imatfaal on 25/10/2010 11:13:14
Bill

no it would be really boring!   [;D] 

And I don't like even your revised definition  [B)]  If something is inherent in an object - then it should be capable of being measured in an objective manner (though it isn't necessarily done objectively - observer might skew data to fit her pet theory: ie subjective) .  I do however know exactly the sense/meaning you are using - I just don't think subjective is the right word - and I cannot think of the right word


Title: What is Time? If there was no light would Time cease to be?
Post by: simplified on 26/10/2010 16:36:22
I would like to know borders of local time in a kinematic field  [?]
Title: What is Time? If there was no light would Time cease to be?
Post by: imatfaal on 26/10/2010 17:02:32
I am sorry simplified - I just cannot follow your arguments/quesions
Title: What is Time? If there was no light would Time cease to be?
Post by: king5118 on 27/10/2010 10:00:15
Light is a human invention and is linked to the past and future, but you are quite right in saying that it is also linked to light. If we hadn't invented time we would still rely on the sun and moon to measure our activities throughout the day and night. People get so wrapped up with time and forget that as an invention by us it can be abused and used to explain so many things. I would say that the main focus of our attention should be distance and not time. It is theoretically possible to travel faster than light and just takes someone to come up with the idea.
Title: What is Time? If there was no light would Time cease to be?
Post by: king5118 on 27/10/2010 10:16:32
If our sun didn't exist and the stars that we can see were using the light of their own galaxies then could we still say that they were in the past? Or is it relative to where we are in conjunction with them.
Title: What is Time? If there was no light would Time cease to be?
Post by: Bill S on 27/10/2010 16:07:56
Quote from: imatfaal
no it would be really boring!

Of course it would be boring if a language in which each word had only one meaning were the only language we had, but there could be advantages to having a sort of techno-speak in which everyone could agree on definitions.   
Title: What is Time? If there was no light would Time cease to be?
Post by: Bill S on 27/10/2010 16:13:40
Quote from: King5118
It is theoretically possible to travel faster than light and just takes someone to come up with the idea.

Relativity does not preclude travelling at superluminal speed.  My understanding is that serious problems arise if you try to accelerate from sub- to super-luminal speed.  are you saying that that is theoretically possible?
Title: What is Time? If there was no light would Time cease to be?
Post by: Bill S on 27/10/2010 16:26:38
Quote from: King5118
If our sun didn't exist and the stars that we can see were using the light of their own galaxies then could we still say that they were in the past? Or is it relative to where we are in conjunction with them.

I'm not sure that I follow this.  What difference would the absence of our sun make to the scenario?
The fact that we see distant stars, galaxies etc. as they were in the past is due to the length of time their light takes to reach us.  This even applies to our sun; we see it as it was about 8 mins ago.
Could it be that we create some of our own problems by thinking of time as "flowing"?  It might be simpler (some hopes  [;D])to think of time as static, and ourselves as travelling through it.
Title: What is Time? If there was no light would Time cease to be?
Post by: simplified on 28/10/2010 14:42:32
Moving cannot be without time.
Title: What is Time? If there was no light would Time cease to be?
Post by: Ron Hughes on 28/10/2010 15:25:36
Suppose that the Universe consisted of only a single electron, would time exist? The quick answer would be no because nothing is moving with respect to anything else.That is not entirely true and I will explain if anyone asks. Now suppose we insert another electron, that is in motion WRT the other electron, into the Universe. Now there are two electric fields in our hypothetical Universe and they are changing with WRT each other. Time is any change WRT something else.
Title: What is Time? If there was no light would Time cease to be?
Post by: Bill S on 28/10/2010 23:23:00
Quote from: Ron Hughes
Suppose that the Universe consisted of only a single electron, would time exist? The quick answer would be no because nothing is moving with respect to anything else.That is not entirely true and I will explain if anyone asks.

Explanation, please.
Title: What is Time? If there was no light would Time cease to be?
Post by: CPT ArkAngel on 29/10/2010 03:00:14
The event horizon of a particle or a black hole is defining the 0 timerate of it. Gravity around it defines the local timerate. According to photon Theory, there is a point in the middle (not really a singularity but due to symmetry of the gravitational field) where the timerate is 0. This point will move according to interference with the event horizon from outside fields.
Title: What is Time? If there was no light would Time cease to be?
Post by: Ron Hughes on 29/10/2010 03:13:29
(Play like) you were an entity capable of creating space/time and the matter that makes our Universe. Instead you create only a single electron which would include it's electric field. At the instant you create it that electric field would start expanding at C, This expanding field would obey the inverse square law which would mean that time and space started at the instant the electron was created.
Title: What is Time? If there was no light would Time cease to be?
Post by: Bill S on 29/10/2010 22:10:28
Thanks R H.  Just to make sure we are not running into Mark McCutcheon's expanding electrons [:o)], it is only the electric field that expands?
Title: What is Time? If there was no light would Time cease to be?
Post by: Ron Hughes on 29/10/2010 23:25:04
Yes the field is what expands. If you somehow move the electron that will create a photon. You could ask, move it with respect to what? The answer is move it with respect to the expanding field that started from it's original position.
Title: What is Time? If there was no light would Time cease to be?
Post by: Bill S on 30/10/2010 17:02:01
Quote from: R H
If you somehow move the electron that will create a photon.

Is that because you would have to hit the electron with something to move it, and the collision would "create a photon"? 
Would it matter what you used to hit the electron?
Title: What is Time? If there was no light would Time cease to be?
Post by: Ron Hughes on 30/10/2010 17:28:24
Just the act of moving it in any way see this,  http://www.thenakedscientists.com/forum/index.php?topic=34333.0;topicseen , and this. http://www.thenakedscientists.com/forum/index.php?topic=34754.msg328711#msg328711
Title: What is Time? If there was no light would Time cease to be?
Post by: simplified on 18/01/2011 16:19:14
Propagation of my time definition:
   "Time is quantity of motion"  [;)]
Title: What is Time? If there was no light would Time cease to be?
Post by: QuantumClue on 18/01/2011 21:28:15
Time is in three parts:  Future-which never arrives and exists only in ones' imagination.  Present-which exists only for an infinitesimal split second and Past which goes on forever in ones' memory but never returns except as mistakes which we make and are inclined to repeat.  Thanks for comments.  Joe L. Ogan

This is wrong. Time does not have a description of past, present and future. These objects do not exist within relativity and quantum theory.
Title: What is Time? If there was no light would Time cease to be?
Post by: QuantumClue on 18/01/2011 21:31:54
Propagation of my time definition:
   "Time is quantity of motion"  [;)]

This is also wrong. This is a Newtonian definition of time. Time does not equal motion.
Title: What is Time? If there was no light would Time cease to be?
Post by: QuantumClue on 18/01/2011 21:33:05
Moving cannot be without time.

Since time is quantized in quantum mechanics, and since time has no flow in relativity, this is not true.
Title: What is Time? If there was no light would Time cease to be?
Post by: Geezer on 18/01/2011 21:46:58
Propagation of my time definition:
   "Time is quantity of motion"  [;)]

This is also wrong. This is a Newtonian definition of time. Time does not equal motion.

Time is measured in terms of motion. Just because it's Newtonian, it does not mean it's "wrong".

Also, unless you are only trying to start an argument, if you believe something is wrong, it would be nice if you provided an explanation for why you think it's wrong.

Can you explain how you are able to measure time without motion?
Title: What is Time? If there was no light would Time cease to be?
Post by: QuantumClue on 18/01/2011 21:50:58
Suppose that the Universe consisted of only a single electron, would time exist? The quick answer would be no because nothing is moving with respect to anything else.That is not entirely true and I will explain if anyone asks. Now suppose we insert another electron, that is in motion WRT the other electron, into the Universe. Now there are two electric fields in our hypothetical Universe and they are changing with WRT each other. Time is any change WRT something else.

This is probably the best explanation I've heard so far. However, just because we have more than one moving object in our universe does not mean that clocks necesserily tick.

Explaining why is a little harder to explain. Time evolution, like you would find in the Schrodinger equation (you could even look at the writeup recently which has a small part on the evolution equation) is a type of diffeomorphism. Time constraints in a classical Hamiltonian will eventually find the Schrodinger Equation effectively having no time description.

This is called the Wheeler de Witt equation, and its at the heart of quantum theory and General Relativity.
Title: What is Time? If there was no light would Time cease to be?
Post by: QuantumClue on 18/01/2011 21:52:09
Propagation of my time definition:
   "Time is quantity of motion"  [;)]

This is also wrong. This is a Newtonian definition of time. Time does not equal motion.

Time is measured in terms of motion. Just because it's Newtonian, it does not mean it's "wrong".

Also, unless you are only trying to start an argument, if you believe something is wrong, it would be nice if you provided an explanation for why you think it's wrong.

Can you explain how you are able to measure time without motion?

Oh it is wrong. Motion does not equate to time. In general relativity motion ceases to exist, we have what are called pure gravity models. If the universe was an object, internal energies ceases to move due to the Wheeler de Witt equation.
Title: What is Time? If there was no light would Time cease to be?
Post by: Geezer on 18/01/2011 22:17:35
Propagation of my time definition:
   "Time is quantity of motion"  [;)]

This is also wrong. This is a Newtonian definition of time. Time does not equal motion.

Time is measured in terms of motion. Just because it's Newtonian, it does not mean it's "wrong".

Also, unless you are only trying to start an argument, if you believe something is wrong, it would be nice if you provided an explanation for why you think it's wrong.

Can you explain how you are able to measure time without motion?

Oh it is wrong. Motion does not equate to time. In general relativity motion ceases to exist, we have what are called pure gravity models. If the universe was an object, internal energies ceases to move due to the Wheeler de Witt equation.

That does not answer my question. How do you measure time?
Title: What is Time? If there was no light would Time cease to be?
Post by: QuantumClue on 18/01/2011 22:30:23
Propagation of my time definition:
   "Time is quantity of motion"  [;)]

This is also wrong. This is a Newtonian definition of time. Time does not equal motion.

Time is measured in terms of motion. Just because it's Newtonian, it does not mean it's "wrong".

Also, unless you are only trying to start an argument, if you believe something is wrong, it would be nice if you provided an explanation for why you think it's wrong.

Can you explain how you are able to measure time without motion?

Oh it is wrong. Motion does not equate to time. In general relativity motion ceases to exist, we have what are called pure gravity models. If the universe was an object, internal energies ceases to move due to the Wheeler de Witt equation.

That does not answer my question. How do you measure time?

We are not sure you can in General Relativity. Matter fields vanish, and what is left is an energy field which does not change at all.

In quantum theory, it looks like an approach to this is to measure motion as static frames of tiny little moments called the Planck Time. In theory, time does not have flow, but is a succession of tiny beginnings and ends, each existing for the smallest frame of time possible, and so does not contain a motion as such, not a continuous one any way.
Title: What is Time? If there was no light would Time cease to be?
Post by: Geezer on 18/01/2011 22:32:29
Propagation of my time definition:
   "Time is quantity of motion"  [;)]

This is also wrong. This is a Newtonian definition of time. Time does not equal motion.

Time is measured in terms of motion. Just because it's Newtonian, it does not mean it's "wrong".

Also, unless you are only trying to start an argument, if you believe something is wrong, it would be nice if you provided an explanation for why you think it's wrong.

Can you explain how you are able to measure time without motion?

Oh it is wrong. Motion does not equate to time. In general relativity motion ceases to exist, we have what are called pure gravity models. If the universe was an object, internal energies ceases to move due to the Wheeler de Witt equation.

That does not answer my question. How do you measure time?

We are not sure you can in General Relativity. Matter fields vanish, and what is left is an energy field which does not change at all.

In quantum theory, it looks like an approach to this is to measure motion as static frames of tiny little moments called the Planck Time. In theory, time does not have flow, but is a succession of tiny beginnings and ends, each existing for the smallest frame of time possible, and so does not contain a motion as such, not a continuous one any way.

Yes, but how do you measure time without motion?
Title: What is Time? If there was no light would Time cease to be?
Post by: QuantumClue on 18/01/2011 22:36:46
That's the stubborn illusion of reality Einstein often talked about. Past and future seem to be stubbornly persistent illusions - motion is measured with our equations, but the truth and crux of the matter is that a unified approach will not use the classical postulations of equations of motion. They will adhere to support the framework of relativity.
Title: What is Time? If there was no light would Time cease to be?
Post by: Geezer on 18/01/2011 22:44:21
That's the stubborn illusion of reality Einstein often talked about. Past and future seem to be stubbornly persistent illusions - motion is measured with our equations, but the truth and crux of the matter is that a unified approach will not use the classical postulations of equations of motion. They will adhere to support the framework of relativity.

How can you say it's wrong to use motion to measure time when you apparently have no idea how to meausure it?
Title: What is Time? If there was no light would Time cease to be?
Post by: QuantumClue on 18/01/2011 22:50:05
I say it's wrong, as in it being a wrong approach. I say wrong approach, because any other approach at the moment would not fit the requirements of the theory.

Of course, it may turn out we no longer will require the classical constraints of the theory on time evolution. It may turn out that quantum theory develops an understanding of what we experience, and the quite obvious contradictory statements of General Relativity.
Title: What is Time? If there was no light would Time cease to be?
Post by: Geezer on 18/01/2011 23:04:49
Of course, it may turn out we no longer will require the classical constraints of the theory on time evolution.

Oh! So now you are saying that we are still constrained by the classical theory. You were only speculating that the macroscopic theory of time may have to change because it is inconsistent with the quantum view.

So, there was nothing wrong with Simplified's statement after all.

Title: What is Time? If there was no light would Time cease to be?
Post by: QuantumClue on 18/01/2011 23:15:12
No it is wrong. My last post is stating if theory was wrong.

That is like saying a hypothetical Tom would be right also, when he states that electrons don't exist as a wave function round the nuclei of atoms... because there is that chance theory is wrong. Science doesn't operate like that, usually we say interpretations that don't fit theory are wrong, not the other way around until experimentation proves otherwise.
Title: What is Time? If there was no light would Time cease to be?
Post by: Geezer on 18/01/2011 23:40:54
No it is wrong. My last post is stating if theory was wrong.

That is like saying a hypothetical Tom would be right also, when he states that electrons don't exist as a wave function round the nuclei of atoms... because there is that chance theory is wrong. Science doesn't operate like that, usually we say interpretations that don't fit theory are wrong, not the other way around until experimentation proves otherwise.

Yes, but that's all a big bum steer, so let's try to stick to the question at hand.

Other than repeatedly insisting it is wrong, you have failed to demonstrate that there is anything wrong with the statement "Time is a function of motion." You have even said that we are constrained by the classical theory of time.

You can't simply throw out a theory that works perfectly well and fail to replace it with anything! Clearly, you don't know much about how real science works, because if we did that every time we found an inconsistency, science would be nothing but chaos.

What I think you should be saying is that there is an apparent inconsistency between the quantum views and the macroscopic views. That does not mean that any view is wrong. It just means that we have not yet resolved the inconsistencies. (BTW, it's quite possible we never will.)

Somebody might come along with a great theory next week that stands both the macroscopic views and the quantum views on their heads, but even if they do, we'll continue to use the old models for a considerable time before we discard them entirely.
Title: What is Time? If there was no light would Time cease to be?
Post by: QuantumClue on 18/01/2011 23:54:11
Yes, ok Geezer, I've not explained myself. So all of this:

We are not sure you can in General Relativity. Matter fields vanish, and what is left is an energy field which does not change at all. This is the Wheeler de Witt equation.

In quantum theory, it looks like an approach to this is to measure motion as static frames of tiny little moments called the Planck Time. In theory, time does not have flow, but is a succession of tiny beginnings and ends, each existing for the smallest frame of time possible, and so does not contain a motion as such, not a continuous one any way.

That's the stubborn illusion of reality Einstein often talked about. Past and future seem to be stubbornly persistent illusions - motion is measured with our equations, but the truth and crux of the matter is that a unified approach will not use the classical postulations of equations of motion. They will adhere to support the framework of relativity.


...Was for nothing at all then? Sheesh.
 
Title: What is Time? If there was no light would Time cease to be?
Post by: QuantumClue on 18/01/2011 23:54:46
I can provide references if that would be more illuminating?
Title: What is Time? If there was no light would Time cease to be?
Post by: QuantumClue on 19/01/2011 00:01:23
There are a great array of papers on time here. Some of them touch on the topic that time ceases to exist where movement is involved, others are wholey dedicated to the topic. There is also a paper on the topic of no flow existing in time, which leads to the conclusions of a quantized time.

 http://www.fqxi.org/community/essay/winners/2008.1

These are all the references one needs.
Title: What is Time? If there was no light would Time cease to be?
Post by: Geezer on 19/01/2011 00:10:15
There are a great array of papers on time here. Some of them touch on the topic that time ceases to exist where movement is involved, others are wholey dedicated to the topic. There is also a paper on the topic of no flow existing in time, which leads to the conclusions of a quantized time.

http://www.fqxi.org/community/essay/winners/2009.1

These are all the references one needs.

I'm disappointed you are not able to explain this for yourself. You seemed so adamant in your assertions.

Do any of them explain how your watch, or the clock in your PC measure time without measuring motion?
Title: What is Time? If there was no light would Time cease to be?
Post by: QuantumClue on 19/01/2011 00:11:31
A few of the papers, for the last time Geezer, explain that moving clocks cease to exist - that is quantum clocks, objects moving relative to other things.
Title: What is Time? If there was no light would Time cease to be?
Post by: QuantumClue on 19/01/2011 00:19:18
You edited your message since my most recent with:

I'm disappointed you are not able to explain this for yourself. You seemed so adamant in your assertions.

I have explained myself Geezer. It is you who is refusing to listen.

Title: What is Time? If there was no light would Time cease to be?
Post by: QuantumClue on 19/01/2011 00:20:56
I've posted the wrong link, bare with me: http://www.fqxi.org/community/essay/winners/2008.1
Title: What is Time? If there was no light would Time cease to be?
Post by: QuantumClue on 19/01/2011 00:22:06
What is ultimately wrong in physics, is the topic url I posted before, its been edited. The url is now on the nature of time.
Title: What is Time? If there was no light would Time cease to be?
Post by: Geezer on 19/01/2011 00:26:47
A few of the papers, for the last time Geezer, explain that moving clocks cease to exist - that is quantum clocks, objects moving relative to other things.

Yes, but who's talking about quantum clocks? You didn't limit your loud assertion to the time measured by quantum clocks. All you said was that time is not a function of motion. Not only that, but you don't seem to be able to provide any ideas about how to measure time, other than say that we can't.

When you say "for the last time", can I take it that you are giving up rather than simply admitting you made a sweeping statement when you could have agreed that a qualified statement might have been a lot more appropriate? 
Title: What is Time? If there was no light would Time cease to be?
Post by: QuantumClue on 19/01/2011 00:28:14
Geezer, listen to what I am saying. You cannot assert about motion in GR because moving clocks cease to exist. Why is this part troubling you so?
Title: What is Time? If there was no light would Time cease to be?
Post by: Geezer on 19/01/2011 00:37:44
Rats! I thought you were serious about the "last time".

No, you listen to what I'm saying, and at least attempt to answer my original question which was, "How do you measure time without motion?" Despite the fact that you keep trying to evade the issue, it's not really a trick question. 
Title: What is Time? If there was no light would Time cease to be?
Post by: Geezer on 19/01/2011 00:40:01
BTW - we can split this into a new topic if you want.
Title: What is Time? If there was no light would Time cease to be?
Post by: QuantumClue on 19/01/2011 00:40:31
Rats! I thought you were serious about the "last time".

No, you listen to what I'm saying, and at least attempt to answer my original question which was, "How do you measure time without motion?" Despite the fact that you keep trying to evade the issue, it's not really a trick question. 

I said you can't in GR. That is the answer, that is my answer, and that is truth. Why have you made me say this over three times now to you? THAT is the answer. There is no if's, no but's... You cannot measure time with motion in relativity, because both cease to exist in GR.

This truely is the last time.
Title: What is Time? If there was no light would Time cease to be?
Post by: Geezer on 19/01/2011 00:56:12
Rats! I thought you were serious about the "last time".

No, you listen to what I'm saying, and at least attempt to answer my original question which was, "How do you measure time without motion?" Despite the fact that you keep trying to evade the issue, it's not really a trick question. 

I said you can't in GR. That is the answer, that is my answer, and that is truth. Why have you made me say this over three times now to you? THAT is the answer. There is no if's, no but's... You cannot measure time with motion in relativity, because both cease to exist in GR.

This truely is the last time.

Oh goody! That means I get the last word.

So what you seem to be suggesting is that there is no such thing as time, so, obviously we can't measure something that doesn't exist. It's a pity you didn't say that in the first place rather than insisting that we were measuring time the wrong way.
Title: What is Time? If there was no light would Time cease to be?
Post by: simplified on 19/01/2011 11:13:14
Rats! I thought you were serious about the "last time".

No, you listen to what I'm saying, and at least attempt to answer my original question which was, "How do you measure time without motion?" Despite the fact that you keep trying to evade the issue, it's not really a trick question. 

I said you can't in GR. That is the answer, that is my answer, and that is truth. Why have you made me say this over three times now to you? THAT is the answer. There is no if's, no but's... You cannot measure time with motion in relativity, because both cease to exist in GR.

This truely is the last time.
Name your time by other name. [:P]
Title: What is Time? If there was no light would Time cease to be?
Post by: yor_on on 21/01/2011 15:46:51
Your own time is unchangeable. You can easily check that with your wristwatch against something else near you (heartbeats:). Others 'time' isn't though making them highly unreliable as your 'time pieces'. So when your boss points out that you're too late again, point out back that we left the Newtonian universe 1905. Possibly earlier?
Title: What is Time? If there was no light would Time cease to be?
Post by: simplified on 24/01/2011 17:32:23
One second is defined quantity of Earth rotation.If atomic clock rests on Earth,then its second coincides with terrestrial second .If atomic clock quickly travels, then it shows wrong quantity of Earth rotation,though shows a right quantity of local motion of  local objects. In my definition a quantity of motion can be different.You should understand  measurement of motion quantity by another motion quantity.
Title: What is Time? If there was no light would Time cease to be?
Post by: Bill S on 27/01/2011 22:17:09
I'll say one thing for time; it can certainly generate a lot of scientific discussion, philosophical floundering and even p...ing contests, but at the end of the day, I think I'm still with St Augustine: "What then is time? If no one asks me, I know; if I wish to explain it to someone who asks, I know not."
Title: What is Time? If there was no light would Time cease to be?
Post by: yor_on on 29/01/2011 12:58:15
QC Julian Barbour makes some sense to me :)
Great minds think alike.

I like his views and he is quite poetic presenting them. "the quantum universe is static. Nothing happens; there is being but no becoming. The flow of time and motion are illusions." comes close to how I see it too. the difference possibly being that I define it as 'emergences', and as such having a 'reality' by its own for each 'scale' defined , as we look at it.

Ahem :)

A sweet Pdf.
Thnx.
Title: What is Time? If there was no light would Time cease to be?
Post by: simplified on 30/01/2011 07:54:32
Some sense is not definition.  [;)]
Title: What is Time? If there was no light would Time cease to be?
Post by: QuantumClue on 30/01/2011 09:27:42
QC Julian Barbour makes some sense to me :)
Great minds think alike.

I like his views and he is quite poetic presenting them. "the quantum universe is static. Nothing happens; there is being but no becoming. The flow of time and motion are illusions." comes close to how I see it too. the difference possibly being that I define it as 'emergences', and as such having a 'reality' by its own for each 'scale' defined , as we look at it.

Ahem :)

A sweet Pdf.
Thnx.

He is on the dot - something I tried explaining to Geezer but failed miserably.
Title: What is Time? If there was no light would Time cease to be?
Post by: Bill S on 30/01/2011 21:34:01
I found Julian Barbour's book heavy going in places, but, apart from a few details, I felt that "Platonia" had much in common with the concept of an infinite cosmos, in which everything "is", and all change is illusion resulting from limited perception.   
Title: What is Time? If there was no light would Time cease to be?
Post by: Geezer on 31/01/2011 08:48:45
something I tried explaining to Geezer but failed miserably.

QC, kindly point me to the bit where you tried to explain. I must have missed it.
Title: What is Time? If there was no light would Time cease to be?
Post by: QuantumClue on 31/01/2011 08:54:53
something I tried explaining to Geezer but failed miserably.

QC, kindly point me to the bit where you tried to explain. I must have missed it.

Are you kidding? I went to witts end last time trying to explain this to you, and you were being infinitely stubborn. I'll find it for you.
Title: What is Time? If there was no light would Time cease to be?
Post by: QuantumClue on 31/01/2011 09:08:27
Simplified:

Propagation of my time definition:
   "Time is quantity of motion"


QC:

This is also wrong. This is a Newtonian definition of time. Time does not equal motion.

Geezer

Time is measured in terms of motion. Just because it's Newtonian, it does not mean it's "wrong".

Also, unless you are only trying to start an argument, if you believe something is wrong, it would be nice if you provided an explanation for why you think it's wrong.

Can you explain how you are able to measure time without motion?


QC: qouted Ron Hughs and said

This is probably the best explanation I've heard so far. However, just because we have more than one moving object in our universe does not mean that clocks necesserily tick.

Explaining why is a little harder to explain. Time evolution, like you would find in the Schrodinger equation (you could even look at the writeup recently which has a small part on the evolution equation) is a type of diffeomorphism. Time constraints in a classical Hamiltonian will eventually find the Schrodinger Equation effectively having no time description.

This is called the Wheeler de Witt equation, and its at the heart of quantum theory and General Relativity.


QC: qouted Geezer

Oh it is wrong. Motion does not equate to time. In general relativity motion ceases to exist, we have what are called pure gravity models. If the universe was an object, internal energies ceases to move due to the Wheeler de Witt equation.

Geezer

That does not answer my question. How do you measure time?

QC

We are not sure you can in General Relativity. Matter fields vanish, and what is left is an energy field which does not change at all.

In quantum theory, it looks like an approach to this is to measure motion as static frames of tiny little moments called the Planck Time. In theory, time does not have flow, but is a succession of tiny beginnings and ends, each existing for the smallest frame of time possible, and so does not contain a motion as such, not a continuous one any way.


Geezer

Yes, but how do you measure time without motion?

QC

That's the stubborn illusion of reality Einstein often talked about. Past and future seem to be stubbornly persistent illusions - motion is measured with our equations, but the truth and crux of the matter is that a unified approach will not use the classical postulations of equations of motion. They will adhere to support the framework of relativity.

Geezer

How can you say it's wrong to use motion to measure time when you apparently have no idea how to meausure it?

QC

I say it's wrong, as in it being a wrong approach. I say wrong approach, because any other approach at the moment would not fit the requirements of the theory.

Of course, it may turn out we no longer will require the classical constraints of the theory on time evolution. It may turn out that quantum theory develops an understanding of what we experience, and the quite obvious contradictory statements of General Relativity.


Geezer

Oh! So now you are saying that we are still constrained by the classical theory. You were only speculating that the macroscopic theory of time may have to change because it is inconsistent with the quantum view.

So, there was nothing wrong with Simplified's statement after all.




By this point I was getting quite frustrated. You where being highly pedantic with each reply I gave you, no matter how I summed the posts up to you....




QC

No it is wrong. My last post is stating if theory was wrong.

That is like saying a hypothetical Tom would be right also, when he states that electrons don't exist as a wave function round the nuclei of atoms... because there is that chance theory is wrong. Science doesn't operate like that, usually we say interpretations that don't fit theory are wrong, not the other way around until experimentation proves otherwise.


Geezer

Yes, but that's all a big bum steer, so let's try to stick to the question at hand.

Other than repeatedly insisting it is wrong, you have failed to demonstrate that there is anything wrong with the statement "Time is a function of motion." You have even said that we are constrained by the classical theory of time.

You can't simply throw out a theory that works perfectly well and fail to replace it with anything! Clearly, you don't know much about how real science works, because if we did that every time we found an inconsistency, science would be nothing but chaos.

What I think you should be saying is that there is an apparent inconsistency between the quantum views and the macroscopic views. That does not mean that any view is wrong. It just means that we have not yet resolved the inconsistencies. (BTW, it's quite possible we never will.)

Somebody might come along with a great theory next week that stands both the macroscopic views and the quantum views on their heads, but even if they do, we'll continue to use the old models for a considerable time before we discard them entirely.





Now I simply got angry. How can you say I have not attempted to explain myself despite the several exhanges we have had???



QC

There are a great array of papers on time here. Some of them touch on the topic that time ceases to exist where movement is involved, others are wholey dedicated to the topic. There is also a paper on the topic of no flow existing in time, which leads to the conclusions of a quantized time.

 http://www.fqxi.org/community/essay/winners/2008.1

These are all the references one needs.


Geezer

I'm disappointed you are not able to explain this for yourself. You seemed so adamant in your assertions.

Do any of them explain how your watch, or the clock in your PC measure time without measuring motion?





Which is an understatement. I have been able to explain myself, and I was more than adamant in my assertions. I quite simply and clearly told you that time does not have a newtonian flow that time is quantized, and general relativity has equations in the field theories where movement finally ceases to exist, due to diffeormorphism constraints on the theory. How this is not explaining myself clearly is beyond me. You were just being difficult, and have been off with me ever since, hence why you banned me for the three days for nothing. I said That I did not regret the thing I said, and you banned me after a sufficient warning. But because I NEVER regretted saying what I said, you decided to get super personal about it. Well there, that's the story in balamory.
Title: What is Time? If there was no light would Time cease to be?
Post by: yor_on on 31/01/2011 11:33:12
Now QC, some of it I agree with, other not so much. I know that QM want everything to become 'quanta' of some sort. And if we look above Plank-scale it seems to work. So redefining it I would say that above Plank scale we should be able too see 'quanta'. Does that mean that we have defined the 'smallest things'? Well, inside SpaceTime we might have done so. But then we come to what creates a SpaceTime. And there we seem to have a lot of 'stuff' that I see as being under Plank scale. So, would SpaceTime exist without that other 'stuff'? I don't think so. Can we say that those too are 'quanta'? Nope.
Title: What is Time? If there was no light would Time cease to be?
Post by: QuantumClue on 31/01/2011 14:48:26
Now QC, some of it I agree with, other not so much. I know that QM want everything to become 'quanta' of some sort. And if we look above Plank-scale it seems to work. So redefining it I would say that above Plank scale we should be able too see 'quanta'. Does that mean that we have defined the 'smallest things'? Well, inside SpaceTime we might have done so. But then we come to what creates a SpaceTime. And there we seem to have a lot of 'stuff' that I see as being under Plank scale. So, would SpaceTime exist without that other 'stuff'? I don't think so. Can we say that those too are 'quanta'? Nope.

That old age question, of how to create a dynamical vacuum seems to interest people the most. Whatever happened to the idea something can simply, be.
Title: What is Time? If there was no light would Time cease to be?
Post by: yor_on on 31/01/2011 15:34:32
Well QC. It's more than to 'be' to me?
One of the really big fundamental questions about our universe is how to see it.
As a 'flow' having no 'quanta' at its possibly 'fundamental' plane, or if it is made out of 'quanta'. There are three directions you can go here. Quanta, Flow, or both. I'll go for both :)

And the thing differing them being our perception of 'scales' inside our 'arrow of time'. Maybe, if we had the eyes of a God, we would find them all to be one 'whole expression' where what we call QM and macroscopic loses its definitions? I don't know there but I find that approach to be more possible than to give only one of them the 'origin'. And if I'm right then there is your 'be'.
==

The idea of a flow as the opposite to a 'discrete event' comes from us, inside our arrow.
Take that arrow away, then show me a quanta?
And without being able to do so, where then is the 'flow'?
Nowhere.
Title: What is Time? If there was no light would Time cease to be?
Post by: yor_on on 31/01/2011 16:08:32
You can also look at it this way I think.

Everything working in a linear procession are in need of 'durations' under which it transforms. That's life, all sorts of life, and also matter. Light is something interacting inside three dimensions and time without the need to take a 'place' in those, according to the Pauli exclusion principle, so light is a slightly different proposition.

But for everything else we know there is a need for a positional, even if steered by probability when defining, 'place' in our 'room time geometry'. From there comes the idea of our 'quanta' aka 'thingies' needing a 'duration'. But for a 'thingie' to have a duration and a singular 'existence' there is needed a 'interface' in where it does not. That's 'built in' in all such definitions. The way around that is to define it as a 'flow' in where all 'thingies' are, in some way, inseparable. But our universe don't seem to care for neither of those definitions, or rather for both, simultaneously in fact as shown by the two slit experiment.

So whom am I to declare the universe wrong?
==

And it's from this concept the idea of a 'background' comes. In a universe of 'thingies' having their own durations and existence the need for a 'background' from where they show themselves is self evident. In a flow the need for a background disappears as there is nothing 'separating' it. But I think our universe are both, and that it is where you stand conceptually that makes you see one or the other. And that's pretty da*n mystical, ain't it :)
Title: What is Time? If there was no light would Time cease to be?
Post by: simplified on 31/01/2011 16:44:52
QC tried to show time definition.We did not see result.[8D]
Title: What is Time? If there was no light would Time cease to be?
Post by: QuantumClue on 31/01/2011 16:55:54
QC tried to show time definition.We did not see result.[8D]

The definition of time... In a model where time does not exist, like relativity, how can you define such a thing? You can only define it using our outdated concepts, such as flows and enrtopy and movements. That is where theory breaks down, and no longer yields us truthful results.

Also, how do you define something which can never be an absolute?
Title: What is Time? If there was no light would Time cease to be?
Post by: QuantumClue on 31/01/2011 16:56:15
Moreover, if time flows, what is it flowing relative to?
Title: What is Time? If there was no light would Time cease to be?
Post by: yor_on on 31/01/2011 17:21:02
Yeah, without 'times arrow' neither flow nor 'discrete events' make sense. Both need the arrow to exist. Without it you just might get your 'be' though, but even so there is needed to be 'something' creating our arrow. And that's why I find those constants so interesting.
Title: What is Time? If there was no light would Time cease to be?
Post by: Geezer on 31/01/2011 17:27:33
QC, you still have no explained how its possible to measure time without motion. Or are you saying time does not exist?

Title: What is Time? If there was no light would Time cease to be?
Post by: QuantumClue on 31/01/2011 17:29:04
Yeah, without 'times arrow' neither flow nor 'discrete events' make sense. Both need the arrow to exist. Without it you just might get your 'be' though, but even so there is needed to be 'something' creating our arrow. And that's why I find those constants so interesting.

However, if everything is static, in the world of fundamental particles, where time does not exist with a flow, how can an arrow exist? If everything is static like Julian proposes, then there is no before, no after event because events are strictly defined under passing moments, which is just a fancy way of looking at time.
Title: What is Time? If there was no light would Time cease to be?
Post by: QuantumClue on 31/01/2011 17:30:21
QC, you still have no explained how its possible to measure time without motion. Or are you saying time does not exist?



I have said to you time does not exist. Why do you ask this again? I told you in relativity, time ceases to exist, and moving clocks no longer exist.
Title: What is Time? If there was no light would Time cease to be?
Post by: QuantumClue on 31/01/2011 17:32:02
If motion ceases to exist because diffeomorphism invariances leads to the Wheeler de Witt equation in GR, then time cannot exist. Moving objects is what makes time happen in relativity. If moving clocks cease to exist, then time as we know it also ceases to exist.
Title: What is Time? If there was no light would Time cease to be?
Post by: simplified on 31/01/2011 17:48:51
QC tried to show time definition.We did not see result.[8D]

The definition of time... In a model where time does not exist, like relativity, how can you define such a thing? You can only define it using our outdated concepts, such as flows and enrtopy and movements. That is where theory breaks down, and no longer yields us truthful results.

Also, how do you define something which can never be an absolute?
Show the history without quantity of Earth motion.Show calculations of changings without quantity of motion.
Title: What is Time? If there was no light would Time cease to be?
Post by: QuantumClue on 31/01/2011 17:55:16
QC tried to show time definition.We did not see result.[8D]

The definition of time... In a model where time does not exist, like relativity, how can you define such a thing? You can only define it using our outdated concepts, such as flows and enrtopy and movements. That is where theory breaks down, and no longer yields us truthful results.

Also, how do you define something which can never be an absolute?
Show the history without quantity of Earth motion.Show calculations of changings without quantity of motion.

So many of you are quick to believe time does not pass for a photon, yet relative to us, they seem to move from place to place, and still take 8 mins to reach us from the sun...

...Why is the concept of this so difficult for people to comprehend... Einstein said time was an illusion. We are stuck in that sphere of illusion, so we still percieve objects and planets and stars all move to the tune of time. But the equations of relativity tell us something quite different - we cannot jump out of this sphere of illusion. This illusion will always seem contradictory to us: But I have faith in relativity. I have faith in quantum mechanics. So did Einstein, and so did every other famous scientist who knew about the Wheeler de Witt equation.

We are told that time does not exist in GR. It will never deter me to believe time does not exist, just because others cling to their own delusions on motion present in their sphere of lies.
Title: What is Time? If there was no light would Time cease to be?
Post by: yor_on on 31/01/2011 18:02:53
QC, you still have no explained how its possible to measure time without motion. Or are you saying time does not exist?



Geezer, I haven't followed all of your discussion, but as I see it. Can there be any 'events' taking place without a 'arrow of time'? I don't think so myself, all change need a duration, and to get a duration some sort of 'arrow' should be involved. So if there is something existing without this arrow there need to be another principle defining it. If we look at 'time' from 'motion' then maybe you might assume that just as motion can take all types of direction, maybe 'times arrow' can do the same?

If it could, yeah I know, this is not 'main stream' :) But if it could, would it be possible to find 'states' where that arrow more or less took out itself? And if there could be something like that, a little like particles and anti particles annihilating each other, what would that look as to us? A motion? Or no motion at all? A arrow existing, or no arrow at all?
Title: What is Time? If there was no light would Time cease to be?
Post by: yor_on on 31/01/2011 18:16:07
You can also turn the concept around.

Assume that what really is is something where 'times arrow' doesn't have a direction. Then a universe, any universe of a duration, will have to be a 'exception' of sort. And as soon as you introduce a duration to something you also will create a 'distance'. Distance is something defined by you measuring durations, and defining differences between them, like naming this A and that B. Speed is a distance measured under an arrow of time.

So maybe?
Title: What is Time? If there was no light would Time cease to be?
Post by: QuantumClue on 31/01/2011 18:18:00
QC, you still have no explained how its possible to measure time without motion. Or are you saying time does not exist?



Geezer, I haven't followed all of your discussion, but as I see it. Can there be any 'events' taking place without a 'arrow of time'? I don't think so myself, all change need a duration, and to get a duration some sort of 'arrow' should be involved. So if there is something existing without this arrow there need to be another principle defining it. If we look at 'time' from 'motion' then maybe you might assume that just as motion can take all types of direction, maybe 'times arrow' can do the same?

If it could, yeah I know, this is not 'main stream' :) But if it could, would it be possible to find 'states' where that arrow more or less took out itself? And if there could be something like that, a little like particles and anti particles annihilating each other, what would that look as to us? A motion? Or no motion at all? A arrow existing, or no arrow at all?

This is a good point which I bolded.

How can there be an arrow? It is part of our psychological makeup which makes us think there is some arrow which extends from our past to our future states. If that be the case, we would be able to map some origin in the universe, where everything came from, then draw a line, and point it to the future, but there is no such method because big bang occurred in every point in spacetime. The very fact the universe cannot be geometrically-modelled to suit a past and a future with a single arrow is evidence enough that such a psychological arrow of time, is exactly psychological. Not a true physical representation.
Title: What is Time? If there was no light would Time cease to be?
Post by: yor_on on 31/01/2011 18:32:36
*There we differ QC :)

To me a arrow either is 'there' or it isn't. and what seems to make it possible for us are e.g the constants, like the 'Plank scale' for example. There are a lot of interesting constants and I hope we will find more. To me they seem to stake out the 'borders' for SpaceTime, it's just a question of finding the right ones defining how the 'arrow' can work.

And I differ between 'Time' as a concept and the 'arrow of time' myself. 'Time' as such need no specific arrow, and to us it will seem as unchanging, but any universe experiencing events will need arrow(s) to be defined.

So yes, to me 'times arrow' is more than just psychological. In relativity you have only one unchanging duration. Your expiration date will be the same no matter what you do, so to speak. You can live next to black hole or on Earth, traveling almost at lights speed or having a prolonged beach party, your clock will still measure the same amount of heartbeats in your 'frame of reference'. That someone else can give you another 'time definition' won't matter for this.
Title: What is Time? If there was no light would Time cease to be?
Post by: QuantumClue on 31/01/2011 18:33:24
Psychological Arrow of time

''Psychological time is, in part, the cataloguing of ever increasing items of memory from continuous changes in perception. In other words, things we remember make up the past, while the future consists of those events that cannot be remembered. The ancient method of comparing unique events to generalized repeating events such as the apparent movement of the sun, moon, and stars provided a convenient grid work to accomplish this. The consistent increase in memory volume creates one mental arrow of time. Another arises because one has the sense that one's perception is a continuous movement from the known (Past) to the unknown (Future). Anticipating the unknown forms the psychological future which always seems to be something one is moving towards, but, like a projection in a mirror, it makes what is actually already a part of memory, such as desires, dreams, and hopes, seem ahead of the observer.

The association of "behind = past" and "ahead = future" is itself culturally determined. For example, the Chinese and the Aymara people both associate "ahead = past" and "behind = future".[8] In Chinese, for instance, the term "the day after tomorrow" literally means "behind day" while "the day before yesterday" is referred to as "front day" and in Hindi (an Indian Language), the term used for "tomorrow" and "yesterday" is the same.[citation needed]

The other side of the psychological passage of time is in the realm of volition and action. We plan and often execute actions intended to affect the course of events in the future. Hardly anyone tries to change past events. Indeed, in the Rubaiyat it is written (sic):

The Moving Finger writes; and, having writ,
  Moves on: nor all thy Piety nor Wit
Shall lure it back to cancel half a Line,
  Nor all thy Tears wash out a Word of it.
- Omar Khayyám (Fitzgerald translation)''


http://en.wikipedia.org/wiki/Arrow_of_time
Title: What is Time? If there was no light would Time cease to be?
Post by: QuantumClue on 31/01/2011 18:34:31
*There we differ QC :)

To me a arrow either is 'there' or it isn't. and what seems to make it possible for us are e.g the constants, like the 'Plank scale' for example. There are a lot of interesting constants and I hope we will find more. To me they seem to stake out the 'borders' for SpaceTime, it's just a question of finding the right ones defining how the 'arrow' can work.

And I differ between 'Time' as a concept and the 'arrow of time' myself. 'Time' as such need no specific arrow, and to us it will seem as unchanging, but any universe experiencing events will need arrow(s) to be defined.

So yes, to me 'times arrow' is more than just psychological. In relativity you have only one unchanging duration. Your expiration date will be the same no matter what you do, so to speak. You can live next to black hole or on Earth, traveling almost at lights speed or having a prolonged beach party, your clock will still measure the same amount of heartbeats in your 'frame of reference'. That someone else can give you another 'time definition' won't matter for this.

It's like another illusion see. What we think is an arrow of time, is just our psychological makeup, cataloging events as they unturn to us.
Title: What is Time? If there was no light would Time cease to be?
Post by: Geezer on 31/01/2011 18:37:54
QC,

So how do you define the speed of light if there is no such ting as time? Come to that, how do you define any speed without using time?
Title: What is Time? If there was no light would Time cease to be?
Post by: QuantumClue on 31/01/2011 18:47:19
QC,

So how do you define the speed of light if there is no such ting as time? Come to that, how do you define any speed without using time?

As I said, for all practical purposes, time is a very useful man-made device. It helps us with our equations, how to catalogue our events. I never said it was never used. General Relativity paints a different picture to the world we have become akin to. It is different to the world we set our equations to, where time was invariant within the equations of motion, and the passing of events.

You'll just have to learn to understand, two worlds exist right now. Both are contradictory, and the only machines stuck in the middle trying to comprehend the two are ourselves.
Title: What is Time? If there was no light would Time cease to be?
Post by: yor_on on 31/01/2011 18:48:34
Your heartbeats are no illusion QC :)

And they will produce the same amount no matter what you do. Assuming, for example, one million heartbeats to one life. then that is the 'duration' taken under all circumstances, traveling near light, being on Earth, or at a neutron star, assuming that one could survive such of course:)

That's what relativity says about 'durations' and 'times arrow'.

That, your own 'room time geometry', aka Frame of reference always will present you with the same durations. So no, it's not an illusion to me, but neither is a 'quanta', or a 'flow'. To me they all seem to come with how you define what you look at, and from where you look.
Title: What is Time? If there was no light would Time cease to be?
Post by: QuantumClue on 31/01/2011 18:50:26
Your heartbeats are no illusion QC :)

And they will produce the same amount no matter what you do. Assuming, for example, one million heartbeats to one life. then that is the 'duration' taken under all circumstances, traveling near light, being on Earth, or at a neutron star, assuming that one could survive such of course:)

That's what relativity says about 'durations' and 'times arrow'.

That, your own 'room time geometry', aka Frame of reference always will present you with the same durations. So no, it's not an illusion to me, but neither is a 'quanta', or a 'flow'. To me they all seem to come with how you define what you look at, and from where you look.

...for us physicists believe the separation between past, present, and future is only an illusion, although a convincing one.

Albert Einstein
Title: What is Time? If there was no light would Time cease to be?
Post by: yor_on on 31/01/2011 18:58:32
To get to it being a 'illusion' I will have to stop looking at what I actually experience and instead exchange it for other 'time definitions'. But doing so will not change the amount of heartbeats, or my expiration date, as I actually experience it. There is a difference between conceptualizing that 'times arrow' differs as compared to others, with speed and mass, and looking at what's really happening to yourself. Knowing that others 'arrows' may magnify or contract relative yours does not change your own duration.

And so we comes back to what you can measure, and what you can know conceptually only :)
Title: What is Time? If there was no light would Time cease to be?
Post by: simplified on 31/01/2011 19:07:57
Your heartbeats are no illusion QC :)

And they will produce the same amount no matter what you do. Assuming, for example, one million heartbeats to one life. then that is the 'duration' taken under all circumstances, traveling near light, being on Earth, or at a neutron star, assuming that one could survive such of course:)

That's what relativity says about 'durations' and 'times arrow'.

That, your own 'room time geometry', aka Frame of reference always will present you with the same durations. So no, it's not an illusion to me, but neither is a 'quanta', or a 'flow'. To me they all seem to come with how you define what you look at, and from where you look.

...for us physicists believe the separation between past, present, and future is only an illusion, although a convincing one.

Albert Einstein

Liar opinion is illusion too.
Title: What is Time? If there was no light would Time cease to be?
Post by: yor_on on 31/01/2011 19:11:34
Let me put it this way. In the twin experiment we have those twins meeting again, after one having taken a really speedy tour through the universe. When they meet one will be provably 'older' biologically. But, let me ask you. do you think any of them found their heartbeats changing relative their respective 'frames of reference'? If you think the durations experienced was a illusion shouldn't they do that? So your own unique 'room time geometry' will present you with one, defined, duration, no matter what you do. So the 'arrow' do exist, well as I see it, furthermore rather 'unchangingly' from your own measurable plane.

That you can't say the same about the universe in general is another thing. But from your perspective traveling close to light it will not be your 'durations' that have changed, any measurement you make will tell you the same thing. That da*ed universe is the one speeding up, not you :)
Title: What is Time? If there was no light would Time cease to be?
Post by: Geezer on 31/01/2011 19:16:11
...for us physicists believe the separation between past, present, and future is only an illusion, although a convincing one.

Albert Einstein


Ironic that he talked a lot about the speed of light, don't you think?

Also, as atoms can spontaneously change state, if there is no such time as time, does that mean they are in both states simultaneously? (I probably should not use the words simultaneous and spontaneous, because they both imply that time is real.)
Title: What is Time? If there was no light would Time cease to be?
Post by: QuantumClue on 31/01/2011 19:21:22
...for us physicists believe the separation between past, present, and future is only an illusion, although a convincing one.

Albert Einstein


Ironic that he talked a lot about the speed of light, don't you think?

Also, as atoms can spontaneously change state, if there is no such time as time, does that mean they are in both states simultaneously? (I probably should not use the words simultaneous and spontaneous, because they both imply that time is real.)

Why didn't you just say superpositioning :)

Einstein said that in relativity, the lifes of things, like particles that whizz across the galaxy to even our mortal lifetimes are all smeared out from past to the future. They are all frozen in time.

It's not that a particle exists simultaneously at the same time. The lifetime of a particle is smeared out over an eternal present. The eternal present is specifically chosen here because as we have seen, past and future are devoid of a true meaning in relativity.
Title: What is Time? If there was no light would Time cease to be?
Post by: QuantumClue on 31/01/2011 19:24:16

Ironic that he talked a lot about the speed of light, don't you think?

Ironic, or just being choosey? What other language would we use?
Title: What is Time? If there was no light would Time cease to be?
Post by: QuantumClue on 31/01/2011 19:29:52
http://books.google.co.uk/books?id=R4LEj_LUtd0C&pg=PA98&lpg=PA98&dq=einstein+said+reality+is+frozen+in+time?&source=bl&ots=GzRadHaAZJ&sig=C3NZFVOqno1zoMyXMkDmHRBo4NI&hl=en&ei=MA1HTcbqG9OF5AaT8eEJ&sa=X&oi=book_result&ct=result&resnum=8&ved=0CEUQ6AEwBw#v=onepage&q&f=false

''It is space and time, married together as space- time, Einstein said, ... is the space-time of the universe frozen into the “now” of the present moment. ...''
Title: What is Time? If there was no light would Time cease to be?
Post by: simplified on 31/01/2011 19:35:56

Ironic that he talked a lot about the speed of light, don't you think?

Ironic, or just being choosey? What other language would we use?
If you will create one useful thing then we shall believe in your some useless  things too. But you have not made. [:D]
Title: What is Time? If there was no light would Time cease to be?
Post by: QuantumClue on 31/01/2011 19:38:05

Ironic that he talked a lot about the speed of light, don't you think?


Ironic, or just being choosey? What other language would we use?
If you will create one useful thing then we shall believe in your some useless  things too. But you have not made. [:D]

Why the hostility, and what kind of useful thing did you have in mind? I didn't make relativity theory, the mathematical assertions I speak of where derived purely from the soil of theoretical physics. If you don't like it, tough. I don't need to get my head around this. The only person loosing out will be yourself in the end of the day.
Title: What is Time? If there was no light would Time cease to be?
Post by: Geezer on 31/01/2011 19:40:17

Ironic that he talked a lot about the speed of light, don't you think?

Ironic, or just being choosey? What other language would we use?

I don't know what language he should have used, but I do know that the term "speed" is utterly meanlingless without a definition for time.

Talking about the speed of anything and also saying that there is no such thing as time is pure contradiction.
 
Title: What is Time? If there was no light would Time cease to be?
Post by: yor_on on 31/01/2011 19:45:36
Yes, if you apply a conceptual outlook at 'Time' you will find that it somehow is directly connected to what and where you do/are. So 'Time' as such is, conceptually looking, of no exact definition/duration for SpaceTime. It's like we all existed in a clear sea, with all of us coloring the water differently depending on what we are doing relative someone else, the colors being our 'respective general time' when compared against each other, with the difference that those colors do not mix together.

But if we work after what each and one of us can measure in reality, then your duration will be as I said, the exact same, no matter what you do. Without that I would agree with you in that the arrow should be an illusion. And in a way, conceptually, it seems to be. But against it you have those unchanging durations I'm discussing.
Title: What is Time? If there was no light would Time cease to be?
Post by: Geezer on 31/01/2011 19:49:52
with all of us coloring the water differently


Look at me! I'm coloring the water yellow  [;D]

(sorry!)
Title: What is Time? If there was no light would Time cease to be?
Post by: yor_on on 31/01/2011 19:50:41
:)
Title: What is Time? If there was no light would Time cease to be?
Post by: QuantumClue on 31/01/2011 19:54:21

Ironic that he talked a lot about the speed of light, don't you think?

Ironic, or just being choosey? What other language would we use?

I don't know what language he should have used, but I do know that the term "speed" is utterly meanlingless without a definition for time.

Talking about the speed of anything and also saying that there is no such thing as time is pure contradiction.
 

I know.

I have stated time and time again, especially in this thread that relativity, notably GR is purely contradictory to our everyday experiences. Therefore it should be no surprise Einstein said his theory dictated we sense an illusion we call time.
Title: What is Time? If there was no light would Time cease to be?
Post by: yor_on on 31/01/2011 20:08:15
Well, I beg to differ there QC.
But it is strange.

And that I can agree on :)
Title: What is Time? If there was no light would Time cease to be?
Post by: simplified on 31/01/2011 20:16:32
Let me put it this way. In the twin experiment we have those twins meeting again, after one having taken a really speedy tour through the universe. When they meet one will be provably 'older' biologically. But, let me ask you. do you think any of them found their heartbeats changing relative their respective 'frames of reference'? If you think the durations experienced was a illusion shouldn't they do that? So your own unique 'room time geometry' will present you with one, defined, duration, no matter what you do. So the 'arrow' do exist, well as I see it, furthermore rather 'unchangingly' from your own measurable plane.

That you can't say the same about the universe in general is another thing. But from your perspective traveling close to light it will not be your 'durations' that have changed, any measurement you make will tell you the same thing. That da*ed universe is the one speeding up, not you :)
Einstein has created some useful things therefore we trust in his useless things.
Title: What is Time? If there was no light would Time cease to be?
Post by: Geezer on 31/01/2011 20:18:55
I have stated time and time again, especially in this thread that relativity, notably GR is purely contradictory to our everyday experiences. Therefore it should be no surprise Einstein said his theory dictated we sense an illusion we call time.

But that's my point. You keep referring to time, and yet you maintain there is no such thing. If there is no such thing, we can't use any terms that rely on time's existence.

When you can establish an alternative to "time" and explain how we can use that to explain some of the phenomena we observe or how that might be a benefit science, people might take you seriously.
Title: What is Time? If there was no light would Time cease to be?
Post by: QuantumClue on 31/01/2011 22:19:39
I have stated time and time again, especially in this thread that relativity, notably GR is purely contradictory to our everyday experiences. Therefore it should be no surprise Einstein said his theory dictated we sense an illusion we call time.

But that's my point. You keep referring to time, and yet you maintain there is no such thing. If there is no such thing, we can't use any terms that rely on time's existence.

When you can establish an alternative to "time" and explain how we can use that to explain some of the phenomena we observe or how that might be a benefit science, people might take you seriously.

''Then again, who can ever have time, when we can never take time.''

The Merovingian
Title: What is Time? If there was no light would Time cease to be?
Post by: QuantumClue on 31/01/2011 22:35:46
Anyway, the very fact that I use time in my language, is a reference itself that Einstein was right, strengthening his opinion that time is the ultimate illusion, and one which is of a psychological makeup. We cannot escape time in our makeup, it will always be part of our language, for what good would it be not to use such a description, for beings like ourselves that are so caught up in the moment?
Title: What is Time? If there was no light would Time cease to be?
Post by: Geezer on 01/02/2011 00:58:52
Anyway, the very fact that I use time in my language, is a reference itself that Einstein was right, strengthening his opinion that time is the ultimate illusion, and one which is of a psychological makeup. We cannot escape time in our makeup, it will always be part of our language, for what good would it be not to use such a description, for beings like ourselves that are so caught up in the moment?

AFAIK Einstein never came up with any testable theory on this. As such, it's simply speculation which has no practical benefit to science.
Title: What is Time? If there was no light would Time cease to be?
Post by: Bill S on 01/02/2011 02:19:35
Quote from: yor-on
Your heartbeats are no illusion
Just because your heartbeats appear to remain constant in your F of R does not mean they are not an illusion, albeit a persistent one. 

Quote from: Geezer
Talking about the speed of anything and also saying that there is no such thing as time is pure contradiction.

Surely it is a contradiction only if you insist that motion exists. If the cosmos is infinite and unchanging (like Barbour's Platonia) then change, motion, speed and time are all illusions.  Of course, they are illusions that are absolutely necessary for us to make sense of our restricted view of reality.  Without these persistent illusions we would be unable to make sense of our world.   
Title: What is Time? If there was no light would Time cease to be?
Post by: yor_on on 01/02/2011 02:44:27
Well Bill, I can't speak for your heartbeats, but I trust in mine.
There is a limit for everything. Descartes said "Je pense donc je suis," I think, therefore I am, but I'm sure he counted on his heart keeping on beating as he said it:)

As for the comparison it's pretty straightforward. As long as you accept that there is a 'reality' outside your skull you will have to accept that it contains durations. If you then look at those durations you will find that they do not change for you, no matter what you do.

As soon as you lift in someone else's time measuring, not agreeing with yours, you will have stepped outside what you can measure and certify as 'real' in favor of a conceptual description, general and resting on second hand measurements. One of the things making physics accepted is that the experiments must be repeatable. Any secondhand information you have about clocks will fail that test. You will not be able to prove it, as little as he will be able to prove your definition of the 'time/durations'. But we all know that as soon you two are in the same approximate 'room time geometry' you will find that you agree on the durations and time there.

It all depends on how you see it. You can, if you like, consider yourself without substance only expressed as thoughts of no defined duration, but there will still needed to be a linear causality chain to them, and that I will call 'times arrow'. But if you do you better be quiet about it. I saw some men in white coats :) And one never knows. They may not accept the statement that you only are an avid follower of physics, trust me on that at least.
Title: What is Time? If there was no light would Time cease to be?
Post by: Geezer on 01/02/2011 03:30:25
Surely it is a contradiction only if you insist that motion exists. If the cosmos is infinite and unchanging (like Barbour's Platonia) then change, motion, speed and time are all illusions.  Of course, they are illusions that are absolutely necessary for us to make sense of our restricted view of reality.  Without these persistent illusions we would be unable to make sense of our world.   

If you maintain these are illusions, I'd like to see a theory that we can actually test. Do you know of any?

Title: What is Time? If there was no light would Time cease to be?
Post by: QuantumClue on 01/02/2011 09:20:24
Surely it is a contradiction only if you insist that motion exists. If the cosmos is infinite and unchanging (like Barbour's Platonia) then change, motion, speed and time are all illusions.  Of course, they are illusions that are absolutely necessary for us to make sense of our restricted view of reality.  Without these persistent illusions we would be unable to make sense of our world.   

If you maintain these are illusions, I'd like to see a theory that we can actually test. Do you know of any?



When it comes to quantum cosmology, very little of it can ever really be proven, so I am afraid that is a no. It is akin to singularities, and all the other phenomenon of quantum cosmology we just beleive in without question.

Again, you are all so quick to believe a photon does not experience time, yet when I tell you the universe doesn't, you all shudder with disbeleif... and it's not surely because I said it. I have cited several papers all saying the same thing, so the problem cannot be with me, it must be with you lot.
Title: What is Time? If there was no light would Time cease to be?
Post by: QuantumClue on 01/02/2011 09:49:26
Quote from: yor-on
Your heartbeats are no illusion
Just because your heartbeats appear to remain constant in your F of R does not mean they are not an illusion, albeit a persistent one. 

Quote from: Geezer
Talking about the speed of anything and also saying that there is no such thing as time is pure contradiction.

Surely it is a contradiction only if you insist that motion exists. If the cosmos is infinite and unchanging (like Barbour's Platonia) then change, motion, speed and time are all illusions.  Of course, they are illusions that are absolutely necessary for us to make sense of our restricted view of reality.  Without these persistent illusions we would be unable to make sense of our world.   

Wonderful replies Bill. Yes, think of our existences as a frame of reference. Just like a photon where time ceases to exist, the universe as a whole also has no time pass.

Well done!
Title: What is Time? If there was no light would Time cease to be?
Post by: QuantumClue on 01/02/2011 09:54:04
Anyway, the very fact that I use time in my language, is a reference itself that Einstein was right, strengthening his opinion that time is the ultimate illusion, and one which is of a psychological makeup. We cannot escape time in our makeup, it will always be part of our language, for what good would it be not to use such a description, for beings like ourselves that are so caught up in the moment?

AFAIK Einstein never came up with any testable theory on this. As such, it's simply speculation which has no practical benefit to science.

eerrrr... no not quite Geezer. Saying that is completely symmetric to saying Einsteins theory was all speculations. There was a mathematical basis behind his theory, also it agreed well with experimentation. It was not all baseless speculations.

The fact no time passes for the universe is a direct consequence of relativity. If you believe in relativity and you are not one of these cranks that go about saying it is wrong, then you must come to understand that when I say time does not pass in a quantum cosmological model, I say this because theory permits me to, and I have relativity's stamp of approval. I could go into the math if that would persuade you to consider this more than mere speculation if you wanted.
Title: What is Time? If there was no light would Time cease to be?
Post by: QuantumClue on 01/02/2011 10:03:28
Anyway, the very fact that I use time in my language, is a reference itself that Einstein was right, strengthening his opinion that time is the ultimate illusion, and one which is of a psychological makeup. We cannot escape time in our makeup, it will always be part of our language, for what good would it be not to use such a description, for beings like ourselves that are so caught up in the moment?

AFAIK Einstein never came up with any testable theory on this. As such, it's simply speculation which has no practical benefit to science.

What do you mean no practical benefit for science? Just because it has no practical benefit for you, does not mean science does not benefit.

This I have been speaking of, has caused one of the most interesting paradoxes in science. As Bohr said, now we know the paradox, we can now work on the solution.

So, I completely disagree. It's very beneficial; and for those who want to learn how to unify the forces, and relativity to quantum mechanics, then they need to understand the nature time - it would be very beneficial of us to do so.
Title: What is Time? If there was no light would Time cease to be?
Post by: yor_on on 01/02/2011 16:20:26
You know, time invites people to a mess. It's one of the most messed up fields I know. Some mix durations with how the brain treats it, wanting to find the answer to times arrow in the brains electro chemistry. Others falls back on that it seems to differ from frame to frame.

None of that changes the facts. We live in a macroscopically linear universe when it comes to times arrow. Your arrow and expiration date do not change with different frames of reference, or 'room time geometries' as I call it.

To prove times arrow an illusion you will need to introduce another way of how the Universe 'communicates'. People still make the terrible mistake of searching for one 'single solution' to a problem, but our universe doesn't treat itself that way. To see what I mean I'll point out Lorentz contraction. With it and time dilation distances are no truth any more, but I don't see people thinking about that? Why, isn't that really weird? That no speed is sacrosanct any more, no distance an truth? What does it make the universe?

So, if I now speed up all matter in the universe near light, but letting them keep the exact same speed-proportions relative each other. How would the universe look to us? Would SpaceTime contract? It should.

Would all matter suddenly glow with 'potential energy' :)
Really? No, matter would behave the exact same.

And would your arrow disappear? Your life-expectancy shrink.
Nope.

The arrow is here to stay, to me the question is what it is, and what makes it. But stop calling it an 'illusion'. If it was I should be able to 'stop the universe', but in reality we all know it's the other way around. The universe will stop me, and you, and you, and you, ad infinitum..
Title: What is Time? If there was no light would Time cease to be?
Post by: simplified on 01/02/2011 16:43:41
Surely it is a contradiction only if you insist that motion exists. If the cosmos is infinite and unchanging (like Barbour's Platonia) then change, motion, speed and time are all illusions.  Of course, they are illusions that are absolutely necessary for us to make sense of our restricted view of reality.  Without these persistent illusions we would be unable to make sense of our world.   

If you maintain these are illusions, I'd like to see a theory that we can actually test. Do you know of any?


They have formulated their definition of time: " Time is illusion ". Let's check up: then illusion is time. [:o]
Title: What is Time? If there was no light would Time cease to be?
Post by: Geezer on 01/02/2011 18:16:46
QC - If you can't cite an experiment to confirm this is anything more that speculation, it remains speculation.

Just because Einstein kicked some ideas around in his latter years, it does not make it accepted science. Remember, he also thought that QM was nonsense, and so far, he was wrong on that one.

If you can't provide any empirical evidence to support your ideas, please don't tell other people time is all lies and illusion when all the empirical evidence confirms the existence of time. To do so is only creating unnecessary confusion.
Title: What is Time? If there was no light would Time cease to be?
Post by: QuantumClue on 01/02/2011 19:49:20
QC - If you can't cite an experiment to confirm this is anything more that speculation, it remains speculation.

Just because Einstein kicked some ideas around in his latter years, it does not make it accepted science. Remember, he also thought that QM was nonsense, and so far, he was wrong on that one.

If you can't provide any empirical evidence to support your ideas, please don't tell other people time is all lies and illusion when all the empirical evidence confirms the existence of time. To do so is only creating unnecessary confusion.

I know you are a moderator, but you are bound to discipline. I will surely report you for derailing a thread with baseless claims.

Relativity is not an ''idea which was kicked around''.

Why does this part escape you so evidently?
Title: What is Time? If there was no light would Time cease to be?
Post by: Geezer on 01/02/2011 20:35:49
I was not referring to relativity. I was referring to his ideas on time being an illusion.

And I'm not derailing the thread. I'm just asking you to present some evidence to support your claim that time is an illusion. That was an idea that Einstein came up with very late in his career, long after his work on relativity.
Title: What is Time? If there was no light would Time cease to be?
Post by: yor_on on 01/02/2011 21:08:56
My own opinion of Einsteins Statement 'time is an illusion' is that what he meant was the Newtonian idea of 'absolute time, unchanging'. That one is indeed an illusion when comparing 'frames of references'. But on the other hand, Newton was absolutely correct if we just stick to what we can measure directly, without comparing it with other sources.

It's a little like expecting Einstein to have refuted QM as a theory. He was uncomfortable with some of its ideas, and he didn't accept it as the 'final answer to life the universe and all' but he did accept it.

"On the basis of quantum theory there was obtained a surprisingly good representation of an immense variety of facts which otherwise appeared entirely incomprehensible. But on one point, curiously enough, there was failure: it proved impossible to associate with these Schrodinger waves definite motions of the mass points - and that, after all, had been the original purpose of the whole construction. The difficulty appeared insurmountable until it was overcome by Born in a way as simple as it was unexpected.

The de Broglie-Schrodinger wave fields were not to be interpreted as a mathematical description of how an event actually takes place in time and space, though, of course, they have reference to such an event. Rather they are a mathematical description of what we can actually know about the system. They serve only to make statistical statements and predictions of the results of all measurements which we can carry out upon the system. (Quantum Mechanics, Albert Einstein, 1940)

Albert Einstein Quotes on Quantum Physics: Quantum Mechanics, Theory of Light, Quanta, Particle-Wave Duality, History and Evolution of Quantum Theory. "It seems to be clear, therefore, that Born's statistical interpretation of quantum theory is the only possible one. The wave function does not in any way describe a state which could be that of a single system; it relates rather to many systems, to an 'ensemble of systems' in the sense of statistical mechanics." (Albert Einstein, on Quantum Theory, 1936)

All attempts to represent the particle and wave features displayed in the phenomena of light and matter, by direct recourse to a space time model, have so far ended in failure. And Heisenberg has convincingly shown, from an empirical point of view, that any decision as to a rigorously deterministic structure of nature is definitely ruled out, because of the atomistic structure of our experimental apparatus. (Quantum Physics, Albert Einstein, 1954)

Albert Einstein Quotes on Quantum Physics: Quantum Mechanics, Theory of Light, Quanta, Particle-Wave Duality, History and Evolution of Quantum Theory "All these fifty years of conscious brooding have brought me no nearer to the answer to the question, 'What are light quanta?' Nowadays every Tom, Dick and Harry thinks he knows it, but he is mistaken." (Albert Einstein, 'The Born-Einstein Letters' Max Born, translated by Irene Born, Macmillan 1971)

I still do not believe that the statistical method of the Quantum Theory is the last word, but for the time being I am alone in my opinion. (Albert Einstein, 1936, 'The Born-Einstein Letters' Max Born, translated by Irene Born, Macmillan 1971)

And he said one very wise thing. "I think that matter must have a separate reality independent of the measurements. That is an electron has spin, location and so forth even when it is not being measured. I like to think that the moon is there even if I am not looking at it."

I'm sure almost all of us agree on that one. Matter exist, as the moon, very much so.

Here's a nice paper on thermodynamics and time (http://plato.stanford.edu/entries/time-thermo/)  
Title: What is Time? If there was no light would Time cease to be?
Post by: Geezer on 01/02/2011 21:19:20
Thanks Yoron. According to this gent, AE's views became a bit more extreme latterly, but that may not be accurate.

http://everythingforever.com/einstein.htm

Surprising as it may be to most non-scientists and even to some scientists, Albert Einstein concluded in his later years that the past, present, and future all exist simultaneously. In 1952, in his book Relativity, in discussing Minkowski's Space World interpretation of his theory of relativity, Einstein writes:

"Since there exists in this four dimensional structure [space-time] no longer any sections which represent "now" objectively, the concepts of happening and becoming are indeed not completely suspended, but yet complicated. It appears therefore more natural to think of physical reality as a four dimensional existence, instead of, as hitherto, the evolution of a three dimensional existence."

Title: What is Time? If there was no light would Time cease to be?
Post by: yor_on on 01/02/2011 21:37:25
This is what Einstein actually said.

"Since there exists in this four dimensional structure [space-time] no longer any sections which represent "now" objectively, the concepts of happening and becoming are indeed not completely suspended, but yet complicated. It appears therefore more natural to think of physical reality as a four dimensional existence, instead of, as hitherto, the evolution of a three dimensional existence."

The rest of it is the authors assumptions about what he meant by it, I would say :)

If you look at the citation he is only telling us that from a conceptual viewpoint you can't lift out one 'room time geometry' from all that makes a universe and say. "This is the gold standard for time." And he's perfectly right in that. But he would also agree with me that my 'times arrow', as measured in my 'frame of reference' by me, to be unchanging and 'absolute', delivering me the same timely experience, no matter where I was, or did.

The question about 'durations' aka times arrow doesn't crave that all durations has to be of the same exact 'amount', simultaneously in a universe. but it do seem to crave that they, to you, need to deliver the exact same amount for you, constantly and unchangingly so? Might have to do with thermodynamics that one, and biological living systems, and therefore also with chaos theory.
Title: What is Time? If there was no light would Time cease to be?
Post by: Geezer on 01/02/2011 21:45:26
Ah, right! Thanks.

Yes, I completely agree that there is no "gold standard" as Einstein predicted. So, saying Einstein said time was an illusion is something of an illusion?
Title: What is Time? If there was no light would Time cease to be?
Post by: yor_on on 01/02/2011 21:57:22
Time is an illusion, as soon as you compare frames of reference  :)

Your personal 'arrow of time' on the other hand, will give you the exact same expiration date, no matter how fast you go, or what 'infinite mass' you happen to put your camper by.

And that's weird, ain't it :)

But I differ between 'time' and 'times arrow'. When you look at a universe as a 'whole' then there is no 'now'. And from a strictly measuring view there can't be any 'now' existent as all 'events' happens before you experience them, but loosely we can speak about a 'now' if we stick to what you can measure 'now' from and in your 'frame of reference/room time geometry', that as I see it :)

I use 'room time geometry' because what you see is in fact the whole universe, and all adapted to your 'frame of reference'. Frames of reference makes them into a puzzle of sorts, but the universe will always be a 'whole experience' for anyone observing it.

Ahem ::))
Title: What is Time? If there was no light would Time cease to be?
Post by: Geezer on 01/02/2011 22:10:15
Yes, I agree with your points, although I'd prefer to say that time is not a universal constant rather than an illusion. (There is no master clock.)

A possible analogy might be atmospheric pressure. It's certainly not constant either, but it's also not an illusion.
Title: What is Time? If there was no light would Time cease to be?
Post by: yor_on on 01/02/2011 22:22:57
Yeah, but those room time geometries is all different depending on what you compare them too. Take three ships and Earth, all with different speeds relative each other. Let all three ships 'coast' uniformly moving. You can now pick any of them as the reference point for 'zero motion' and define all other speeds relative that one. Any of those objects (including Earth) will have a different relation Time dilation and Lorentz contraction relative any of the other objects.

All of them will have a unique 'time' relative you. But all of you will, when stopping at Earth, agree that you have the, more or less, exact same 'original' time durations, and that you will be able to confirm by measuring :)

Traveling again none of you will experience your 'time durations' or 'distances' to change, and no measurements will tell you differently. In what manner you might find 'stuff' changing it won't be you, but the universe and those other objects. A really interesting question to me if it really is possible to assume that every 'room time geometry' is the same?

An impossible one to answer really, but it seems like a possibility as we all will share, almost, the same durations when joining.

So I find 'time' weird, and how 'room time geometries' adds up to a universe. But 'Times arrow' I'm pretty sure of existing, for us all.
Title: What is Time? If there was no light would Time cease to be?
Post by: Geezer on 01/02/2011 22:26:58
Agreed! (My analogy was not very good [:D])
Title: What is Time? If there was no light would Time cease to be?
Post by: yor_on on 01/02/2011 22:32:41
Nah, it wasn't bad. I'm constantly seeing things I miss when I write. That's why I have to, eh, rearrange them :)
Title: What is Time? If there was no light would Time cease to be?
Post by: yor_on on 01/02/2011 22:39:27
"A really interesting question to me if it really is possible to assume that every 'room time geometry' is the same?" That one came out weird :)

What I meant was if there was a possibility of a 'original duration' being the same, as it never is measurably changing from any of those four objects own 'room time geometry'.

Because if that is a possibility, then what is it that change?
You, or the universe?
==

If you want to treat it as 'information', ignoring what we see as 'real', like matter and space and light, then. Is it the way the information is presented only? No, that can't be true as we then would have no time dilation, only the illusion of it. And Muon's hitting Earth is a good example of time dilation. So the information we receive have to be real, but if my information also informs me that I do not change? Why would motion, or mass, be able to to change my whole universe? Remember also that we theoretically can imagine Plank sized Black holes, after all, isn't that one of the things CERN hopes to create?
==

Ah, what I meant there was that 'size' as long as we're talking about being over Plank scale, is no hindrance to changing a universe, time dilation and Lorentz contraction-wise. As the 'infinite  gravity' at its 'center' should create the same time dilation/Lorentz contraction than any other Black holes 'center' should be able to do, as I see it.
==

So, what do I know about where I am? Well, it's my own 'room time geometry' centered on my 'frame of reference' so? But it's no unique thing, is it? You have one, he, she, even that pebble must have one slightly different from mine? And still we all swear on that we all are here together, don't we :) I can pick up that pebble and throw it so fast that it almost reach light speed, well, not really :) Would that change its 'room time geometry'? Would its SpaceTime contract?

Frames of reference is a headache :)
Title: What is Time? If there was no light would Time cease to be?
Post by: Geezer on 01/02/2011 22:54:51
Yoron,

Not sure I get "original duration". I'm assuming "room time" is like local frame. Is that right?
Title: What is Time? If there was no light would Time cease to be?
Post by: yor_on on 01/02/2011 23:10:15
I mean whatever type of measurement you do by it, involving a defined duration. Nowadays we use lights 'vibrations' for defining durations, like that cool clock you have :)

And as long as you all, on Earth, find those 'durations' to agree with each other measurements then that's what I like to call 'original'. A little like having Earth as the 'origin' and final measurement for the 'twin experiment'.

If it is so that we all will measure a more or less exact same 'duration' being at one same place. Then find our measurements to still 'ring true' when speeding away, that is, from our own frame never differ with what we measured on Earth. Then the question becomes, do they?

Or are they actually correct. If so, can I conceptually assume all 'durations' aka 'times arrow' to be unchangingly 'the same' when measured inside their own frames, no matter from where one does it? Then what you see as differing with speed is an expression of an 'outside', communicating, with you and your 'arrow of time' keeping the same 'duration' constantly, never changing.

and also, that would give us a weird 'gold standard' of sorts, in that from that perspective all 'durations' would have a 'ground state', if you like :)
==

And yeah, 'room time geometries' are always local to me. Yours, or someone else's 'frame of reference', but as observed from, and by, themselves.
Title: What is Time? If there was no light would Time cease to be?
Post by: yor_on on 02/02/2011 00:58:06
Look at lights speed in a vacuum. There is a similarity to me in that it never will differ for you, no matter what 'frame of reference' being the 'source' of it. But its frequency will, in fact the frequency I think is a rather good approximation to how 'contracted' and 'time dilated' something else will be relative you. At least I think that's correct? Will need to ponder that one, won't I :)

But that weird 'speed' seems as unchangeable as your own measurements and 'durations' are to you.
==
The problem with using this idea becomes in that.

Assume that we all have a unique 'SpaceTime'. Not only you but 'everything' from Plank scale and up. Then assume that in their own 'room time geometry' all 'durations' constantly being the same unchanging, that is, a 'gold standard' of sorts. Then assume your 'room time geometry' interacting with all other 'room time geometries'. This means that with every object observed there comes a complimentary unique 'SpaceTime' although to you 'invincible' as any 'information' you receive is filtered through your 'SpaceTime'

They all have the same 'original duration' as I defined it. But, they all will describe any other 'room time geometry' as presenting, for example, a 'twisted' duration. That means that you won't agree on the time. But we still have the idea of them containing a same 'duration' as proved by you moving to into their 'room time geometry' finding that your measurements correlate perfectly with theirs.

Can both those statements be true?

That we all have one unchanging 'arrow of time' but still find the other ones 'arrow of time' to be different when being in different 'room time geometries'?

Where is the 'interface' between those descriptions?
==

And what does it make 'durations' aka 'the arrow of time'?
A local phenomena only?

Well, yes but, if it is right, shared by us all? How can something be so extremely 'local' at the same time as everything have it, those 'ground state durations' as I call them? And how, if my assumption of those 'frames of reference' becoming subtly 'rearranged', from Plank scale and up, is true. How do they 'connect'. Because they do, into particles, molecules, matter, us, and a whole seamless universe.
Title: What is Time? If there was no light would Time cease to be?
Post by: yor_on on 02/02/2011 02:09:23
You know Geezer, your idea of time being an expression of motion may be on the dot. In a slightly weird manner, speed gives a time dilation. But if your 'durations' won't change according to your measurements and you still come back finding yourself now younger than your twin?

What exactly made that 'change'? It's one of sciences golden rules that gets violated here if we accept that we should disregard our measurements and experiments in advantage for a conceptual truth, unable to be proven where we are.

Einstein said 'SpaceTime', not Space and Time did he :)

So what did we change? Our whole 'room time geometry' of course. And that one can only be one. To any of us there will only be one existing, no matter what others report. And I think it acceptable to look at it as a whole thing. Not differing it in my 'spaceship' versus the rest of the universe. After all, everything change when you speed up. And if you accept Lorentz contraction and time dilation as real then those ships you meet will have contracted and relative you be time dilated too.

So, what is energy?
==

One of the best proofs I know for Lorentz contraction being a real thing, no illusion, is to spin a round plate as close as you can to light speed and then watch it 'crack up' as it contracts around its rim. Well, we can't, but we have Black holes that are assumed to spin close to light. Maybe we could use them for testing if it's correct? Don't know how but maybe, near the EV?

Then we have the muons of course :)
==

If you look at speed from that point of view its no 'speed' anymore. It's a contraction of the 'room time' you exist in. And that 'room time' have to be only yours, otherwise all would share the contraction.

So now we have two questions.

What is energy?
And what is distance?
==

Accepting that it is a whole 'room time geometry' that gets changed we can test it too. Because then your room time will behave differently, and it does. But what about those unchanging durations and measurements I make inside my ship then? There distance never changed, and the times 'durations' is still the same to me as they were on Earth?

Title: What is Time? If there was no light would Time cease to be?
Post by: yor_on on 02/02/2011 03:08:46
well, either you think of 'one SpaceTime' with all those 'frames of reference' constantly melting into each other, more or less magically and 'invincibly'. Or you decide that we all have different 'room time geometries' and that even though we share and perceive only one, the one that each one of us perceive must be slightly different and unique. If you choose the later it's no longer any miracle that others 'SpaceTimes' won't get influenced by yours.
==

But it won't explain why we all perceive it as the same 'SpaceTime'. And it don't explain how we cooperate and live together. And neither does it address how we interact, gaining energy. After all, then there should be a infinity of 'SpaceTimes' if that was right, right?
=

Why we perceive only one is fairly simple though. You can't perceive anything more than that, can you? :)
==

I wrote "But what about those unchanging durations and measurements I make inside my ship then? There distance never changed, and the times 'durations' is still the same to me as they were on Earth?"

That one becomes slightly weird. If I assume that my whole 'room time geometry' contracted and also assume that 'time' is a indivisible part of it, why didn't my time contract too? If it did I might assume that the time dilation would work the other way, and that the one traveling would become the older twin. So my 'room' contracts, but my 'time' magnifies?

Think of it as equally large 'measures'. With the rooms 'markers' contracting, and the other indivisible part of a SpaceTime, the 'times' markers growing. That is, each 'duration' of 'times arrow' must have 'expanded' for me in a way, 'slowing down' my aging as compared to my twin on Earth. A little like squeezing a balloon in the middle watching it adapt by bulging out at the ends. It's a very unscientific description, and all of this is a thought experiment, please remember this. But it catches one important part of SpaceTime, its indivisibility.

We think we have a 'SpaceTime' where the conservation laws rules. Where nothing is 'lost' and where 'energy' is what gets consumed in the transformations of work, to finally 'work done'.

So what is 'Energy'?
Title: What is Time? If there was no light would Time cease to be?
Post by: Geezer on 02/02/2011 03:56:05
I think there are limitless spacetimes as you suggest, but as all motion is controlled by the time within a frame, everything behaves normally within that frame.
Title: What is Time? If there was no light would Time cease to be?
Post by: yor_on on 02/02/2011 04:04:55
Yep, and none of us will ever perceive it differently. At least as I understand it :)
Motion is one weird idea :)

I mean, it seems so simple, we all know what it is, but it changes a whole 'SpaceTime'. And you don't even need to 'move'. Invariant mass does it as well :)
Title: What is Time? If there was no light would Time cease to be?
Post by: yor_on on 02/02/2011 04:33:19
Think of gravity waves. We have found indications/evidence of them existing when looking at binary stars. When such a wave rolls through you, you both contract and expand, but you won't ever notice it. Neither will you 'break'. Now think of 'time', not its 'arrow' but primeval time itself. Imagine it as a ocean joined 390 degrees against 'reality', which means our other three dimensions :) is seamlessly joined to it, none existing on its own. I'm not saying that you can't have one or two dimensional SpaceTimes, but we don't have them, as far as I know.

Now consider the arrow, let us assume that as the 'room contracts' times arrow finds it 'denser'. What happens when traffic meets a 'obstacle' like the road shrinking to only one lane? it slows down doesn't it? And what happens when there becomes four lanes again, it speeds up, right?

Now, I do not think of time as an ocean, well mostly I don't at least. And I do not think of SpaceTime as something getting 'denser' as it contracts :) But I still think that this might be an analogue to what should happen for 'times arrow' to slow down, without me being able to notice and measure it.

And now I better stop, before my imagination throws me over the edge :)
==

Just one question.
Assuming this analogue was correct. What would it do to my suggestion that all 'durations' are the same, as measured from ones own frame of reference? Would it invalidate it, or not?

 :)


Title: What is Time? If there was no light would Time cease to be?
Post by: Geezer on 02/02/2011 06:30:53
Think of gravity waves.

Must I? I can't even think about what I should be doing tomorrow [;D]

Seriously, yes. All durations are the same relative to ones own frame of reference. While I'm sure you could drive a Mack (or Volvo) truck through my atmospheric pressure analogy, it's not so unlike time. Wherever you are, you, and everything else, are subject to that pressure (or time). And time (or spacetime if you prefer) rules all.

Nothing can escape it (well, maybe sub atomic particles can) so all processes (physical, chemical, etc) are dictated by time within a locality (or frame).

If you take it to an extreme, if time were to stop in a given locality (which I suspect is impossible), and we were in that locality, how would we know that it had even stopped, or for how long? As time clearly controls motion, if time stops, so does motion.

Or,

Motion does not determine time. Time determines motion.

Title: What is Time? If there was no light would Time cease to be?
Post by: QuantumClue on 02/02/2011 12:54:18
I was not referring to relativity. I was referring to his ideas on time being an illusion.

And I'm not derailing the thread. I'm just asking you to present some evidence to support your claim that time is an illusion. That was an idea that Einstein came up with very late in his career, long after his work on relativity.

yeah, that is relativity - timelessness is part of relativity, a direct solution to General Relativity, hence it is not merely an idea which was kicked around. Understand that and you might get over yourself for a second.
Title: What is Time? If there was no light would Time cease to be?
Post by: yor_on on 02/02/2011 13:28:01
Let's consider the opposite then. Time as being non-existent. That one is very tricky. Because as soon as you have a linear causality chain you will find we have a name for it, namely 'times arrow'. You might want to call it thermodynamics, but I don't.

In fact it still will be a linear causality chain macroscopically. You see, I'm free to in-cooperate any ideas that exist into that 'arrow'. So you might want to look at it as some weird causality-chain defined by 'decay' where there is nothing 'in reality' that have an general order, but where it all change differently at some quantum level. That's cool with me, and you know what :) That's our arrow of time.

The only thing an 'arrow' introduces is the idea of a 'future', a 'past' and the 'present'. So to 'kill' that idea you will have take away those attribute, invalidating you capacity of cogitation as you 'disappear in a puff of smoke', or as we all 'know it all' instantly, as we no longer is bound by any linear process ordering our thoughts, events, etc..

As I see it.
Title: What is Time? If there was no light would Time cease to be?
Post by: imatfaal on 02/02/2011 14:21:00
QC - would be interested in details of the general solutions to GR that require time not to pass; there are some that end up in large closed loops - which is bad, and slightly naive simplifications which require stationarity.   
Title: What is Time? If there was no light would Time cease to be?
Post by: yor_on on 02/02/2011 15:20:06
Assume that we all have different 'room time geometries' but still share only one 'SpaceTime':)

Can one make sense of that?
How about times arrow? Could that be what differ one indivisible 'SpaceTime' into those 'room time geometries'? Nah, or maybe? I don't know if that's possible, after all, we see a different 'SpaceTime' when speeding, distances time and all. Okay, turn it around. Is there anything unchanging in all those 'room time geometries'? Something 'common' to them all? Well, the constants are still the same, ain't they?

So we have those to lean on.

Let's look at a time dilation from the perspective of all 'durations' being of one same 'size'. how would I explain that I 'slowed down' relative the universe if so? I didn't, it was the universe 'speeding up' I say, but that includes all other in it too.

Yeah, why not? I have 'kept my durations' relative the the universe. But isn't their 'time' also of this 'even unchanging duration'? How did they 'speed up' then?

So? I don't think it's a good one.
Title: What is Time? If there was no light would Time cease to be?
Post by: yor_on on 02/02/2011 15:33:39
So we have two choices as I see it. Assuming, and there is nothing I know disproving this, that all durations actually are of one same 'size/duration'. How do I define a time dilation?

Either I have to use the idea of 'everything' in my 'time dilation' acting in a magnifying/contracting manner on me, that means presenting me with something acting solely on me, like if I was in a bubble of some sort, but that won't explain that contraction of space. If I want to explain that I will have to define a Lorentz contraction as being an illusion, and keeping the time dilation only.

Or I will use the idea of all 'room time geometries' being unique. then I change 'my SpaceTime' by speeding. But that leaves us with explaining why we can interact, and exist, together seamlessly, and why your 'durations/aging' answered that 'speed' of mine, if we now exist in different 'SpaceTimes'?

A third is defining it as 'frames of reference' but as that is the same as my 'room time geometries' the only difference being that all 'frames' are treated equally, and our 'reality' only can exist on a conceptual plane?

That one does not satisfy me.
==

In fact, my 'room time geometries' are no different from 'frames of reference'. Both need to be defined in their interacting with 'everything else', and both need to be explained from the concept of us only observing one SpaceTime.
==

So what is it, 'interacting' in between everything else?
Radiation?
Title: What is Time? If there was no light would Time cease to be?
Post by: simplified on 02/02/2011 18:19:44
People thought   mirage is illusion of water, but  mirage is sky under a corner of refraction  [:D]
Title: What is Time? If there was no light would Time cease to be?
Post by: yor_on on 02/02/2011 18:24:38
Yeah, it's a truly weird subject. But time dilation is a direct result of 'frames of reference', or if you like, the idea of 'frames of reference' is the answer to 'time dilation'.

And as it exists?
Title: What is Time? If there was no light would Time cease to be?
Post by: simplified on 02/02/2011 18:53:23
Yeah, it's a truly weird subject. But time dilation is a direct result of 'frames of reference', or if you like, the idea of 'frames of reference' is the answer to 'time dilation'.

And as it exists?
I have no right to answer your question,because I do not recognise imaginary experiments of Einstein. [:-X]
Title: What is Time? If there was no light would Time cease to be?
Post by: yor_on on 02/02/2011 18:58:37
I was not referring to relativity. I was referring to his ideas on time being an illusion.

And I'm not derailing the thread. I'm just asking you to present some evidence to support your claim that time is an illusion. That was an idea that Einstein came up with very late in his career, long after his work on relativity.

yeah, that is relativity - timelessness is part of relativity, a direct solution to General Relativity, hence it is not merely an idea which was kicked around. Understand that and you might get over yourself for a second.

I never seen Einstein saying that the 'arrow' doesn't exist?
Can you show me where he said that?

That he said that there is no 'now' doesn't state that you don't have a 'arrow of time'. He just defined in from a conceptual point of view in where your 'now' and mine 'now' will differ. But the arrow of time is what makes you age, and think.
===

And another thing, saying that time doesn't exist in fact invalidates Einsteins 'SpaceTime'. Notice the combination :) Space&Time. Einstein saw it as one 'whole thing', indivisible as I've understood it?

Maybe there are some quote from him where he redefine relativity too, that I missed?
Title: What is Time? If there was no light would Time cease to be?
Post by: yor_on on 02/02/2011 19:43:35
"In 1949, Gödel wrote an essay in which he used Einstein’s relativity theory to point out that we can send a message to the past. Gödel went a step further. Using Einstein’s general theory of relativity, he arrived at a universe whereupon we can physically travel to the past. He stated that “by making a round trip on a rocket ship in a sufficiently wide curve, it is possible in these worlds to travel into any region of the past, present, and future, and back again, exactly as it is possible in other worlds to travel to distant parts of space.”

  In the same book, some pages later in an article entitled “Reply to Criticisms,” Einstein commends politely his colleague’s paper with these words: “Kurt Gödel’s essay constitutes, in my opinion, an important contribution to the general theory of relativity, especially to the analysis of the concept of time.” This often-quoted statement has been used as a reaffirmation by Einstein that, according to physics, time travel is possible. But did Einstein really confirm Gödel’s findings? Not at all. Just the opposite. He immediately wrote: “The problem here involved disturbed me already at the time of the building up of the general theory of relativity, without my having succeeded in clarifying it.”

    How does Einstein clarify the issue? He doesn’t do it within general theory of relativity. He goes outside relativity to thermodynamics and invokes the Second Law, the law of irreversibility of natural processes and the increase of entropy—the inseparable sister of energy. With the aid of a diagram, Einstein shows Gödel that we cannot “telegraph” back to our own past because the flow of time has an arrow and “there exists no free choice for the direction of the arrow.” He makes it clear that he wants to secure “the one-sided (asymmetrical) character of time.” These are strong and unambiguous words. Einstein explains the reason: “What is essential in this is the fact that the sending of a signal is, in the sense of thermodynamics, an irreversible process, a process which is connected with the growth of entropy (whereas, according to our present knowledge, all elementary processes are reversible).” "
===

And from a solely local definition of a 'frame of reference'. Defining and measuring from its 'inside', whatever that may be, we will find 'entropy' work, or as I see it 'times arrow' keeping a constant duration, the arrow unmeasurable as differing, no matter what you do, speeding away or not.

And that's what I call unique 'room time geometries'.
Title: What is Time? If there was no light would Time cease to be?
Post by: yor_on on 02/02/2011 20:04:00
The problem of 'time' only comes to be when we try to define it as 'unchanging' for a whole universe. We all know from our own reality that we have a 'arrow' splitting reality in a past as well as a 'future' with something we call a 'present' or 'now' defining where we 'are' in a temporal fashion. What you want to call that 'arrow' I don't really care, as long as you agree on it being undifferentiated, as measured from, and in, your own 'frame of reference'.

And then we come to Newtons 'universally unchanging time'. Well, forget it. That one is a illusion, a conceptual expectation, coming from our experiences of being able to correlate our 'arrows' with each other, inside a more or less common 'frame of reference' aka Earth.

That's not the truth. Einstein gave us the way it really worked. But as measured from the 'inside' of one single 'frame' Newtons definition will hold as far as I can see.

So?
==

Maybe it would be worth it defining what one think might be a 'illusion'?

A. If you make a measurement inside your own 'frame of reference' stating some fact, testable for others. Is that a illusion?

B. If you can't find a way/experiment to prove a proposition to be false, does that make that proposal a illusion?

C. If you believe yourself to define a truth conceptually, without being able to test it inside your 'frame of reference', is that an illusion?
==

A. My idea of 'common durations' of 'times arrow' inside your 'frame of reference' is testable, if we use a wristwatch against heartbeats, or any clock as compare to any other time device, keeping durations.

As for B.? The only way to define it as 'false' is from a 'conceptual standpoint' as I know it? Making a general statement that 'time', or a 'now',doesn't exist on that 'conceptual plane'.

As for C.?
Well, that's for those of you that see it as there existing no 'arrow' to decide.
Is C. a illusion?

It's a tangle that one.
Title: What is Time? If there was no light would Time cease to be?
Post by: Bill S on 02/02/2011 20:41:55
Quote from: yor-on
as measured from the 'inside' of one single 'frame' Newtons definition will hold as far as I can see.

Surely, if that were not the case, the "frame" would not be an inertial frame.
Title: What is Time? If there was no light would Time cease to be?
Post by: yor_on on 02/02/2011 20:53:41
I'm losing you there Bill?
A inertial frame is defined as a uniformly moving one?

Do you mean that in a accelerating frame my heartbeats will differ when compared to my wristwatch? That is, that times arrow, when measured, will express itself differently. If you mean that, do you have a experiment, or a reference to how?
==

If you assume a constantly (uniformly) accelerating frame, then we're talking gravity according to the principle of equivalence. That should then be Earth. And there my statement will hold.

If you assume an non-constant acceleration? I don't see how 'times arrow' can differ if measured inside that frame of reference? To see the difference I understand it as you will have to compare 'frames of reference'. If we take two space ships coasting either can become your 'inertial frame of reference' for the other. If one of them accelerates, constantly or not, you will observe a temporal difference when compared to your 'frame of reference', just as they will assign to you too, as they pass you. But that only is possible in comparisons between 'frames'. From the inside of its 'frame of reference' the accelerating ship will measure the same timely durations as they had when coasting, well, as far as I know?
Title: What is Time? If there was no light would Time cease to be?
Post by: yor_on on 02/02/2011 22:24:21
You might be thinking of what happens with a photon that get sent out from a light-source in the front of a accelerating ship, wandering to the aft where you receive it. But then you're thinking energy, and not times arrow.

And as I understands it, when it comes to the energy in a constantly accelerating ship, as that is equivalent to the photon falling into a gravity well, it will blue shift to you sitting further down the gravity well. The same goes for a non-constant acceleration.

Associating the changing frequency with timely 'durations'?

If we would use the light waves as a clock, defining 'times arrow' as ten cycles per second. Then measuring the wave inside that accelerating ship finding it to be twenty cycles per second as it now is compressed relative us. We then could assume that a second now was of a longer duration than before. So far I'm with you, assuming that I'm right in my assumptions about what happens with the light here.

This reasoning is slightly wrong, the clock measuring it inside the ship would be as influenced by gravity, being matter waves, as all other waves. For it to work we will have to assume someone still relative the acceleration measuring that light. So now I'm dropping into 'frames of reference' anyway, comparing between them. But it still works I think.

But notice that the 'durations' I'm discussing as 'never changing', those measured inside any 'frame of reference', or 'room time geometry', won't change. They will, to you, be of the exact same 'duration' as always.

Why I think this works is because the photon will 'cycle up' in 'energy' relative you watching it as it 'falls' into that gravity well. And so should you, assuming that matter-waves react to gravity the same as ordinary waves do. And the reason you do have nothing to do with motion, rather your proximity to the gravity well? 

But, and this is a big 'but'. Either it's like i think but then it can't be only the gravity, can it? No, it has to be the whole 'room time geometry' hasn't it? not only gravity but the speed in itself.. Or I'm totally bicycling up the walls :)


But then consider what happens if you're up very close to light speed, now turning of your engines, not accelerating any more. According to my earlier reasoning the 'gravity well' that our engine created stops to 'exist'. And as we now are uniformly 'coasting' there is nothing differing us from any other uniformly moving object. Inside that ship the light from your light-bulb no longer will show you any other cycles than it would do from any other uniformly moving, or if you like, inertial object. So does this mean that the relativistic phenomena created by my velocity, outside the ship, have disappeared, just because I turned of the engine? The light from suns in front of me no longer blue shifted relative me?

Not as I know. So using the frequency as a proof for a time dilation inside your own frame of reference? I don't know, you got a point though. But then if we want to use that as a 'acid test' a time dilation only can come to be when accelerating, and as soon as you stop accelerating, no matter your velocity, all time dilation should 'stop'? Also it becomes a question of the Lorentz contraction, as that one still should be there, even with the engines of.

If the 'time dilation' only exist in your acceleration, then if we still see a Lorentz contraction outside the ship? Would that be real? Then you have two ways of 'shortening a journey.

A Time dilation & Lorentz contraction when accelerating
A Lorentz contraction only when 'coasting', moving uniformly.

Alternatively no Lorentz contraction at all as soon as you start 'coasting', no matter what velocity you got from your former acceleration.

Or defining only a 'time dilation' as being real.

Pick your choice :)
==

I'm open for suggestions :)
Title: What is Time? If there was no light would Time cease to be?
Post by: yor_on on 02/02/2011 22:32:08
Another interesting point. If that reasoning was true, then the 'aging' one experience inside that ship is a relation to gravity, not speed per se.

I kind'a like that idea actually :)
==

It fits very nicely with my question what the ultimate rest frame for matter is.
I suggested that it was infalling, into a black hole, if anyone remember.
That means, matter will only be 'at rest' at 'c', aka 'infinite gravity'

And then we have several statements coming together.

'Times arrow' becomes a function of gravity.
And 'speed/velocity' doesn't exist, neither does 'distance' as such.
Only gravity have a reality of its own.

I really need to think about this, it's very weird though :)
==

But to get back to my first question, would my heartbeats differ as compared to that clock?
Nope, I don't expect them too myself, so the 'even durations' I was thinking of should be the same? Which gives me a headache :)
Title: What is Time? If there was no light would Time cease to be?
Post by: Geezer on 02/02/2011 22:45:41
I thought I saw something recently about atomic clocks that are sufficiently stable to demonstrate the effects of gravity on time by comparing the results from two stationalry clocks in different graviational fields. I don't seem to be able to find it again, so maybe I imagined it.
Title: What is Time? If there was no light would Time cease to be?
Post by: yor_on on 02/02/2011 22:51:35
No, I seen some experiments sensitive enough to feel if you moved a clock down on the floor relative leaving its twin on the table recently on the net too. Probably have them somewhere?

Nevermind :)
We seen it both at least ::))
Title: What is Time? If there was no light would Time cease to be?
Post by: yor_on on 02/02/2011 23:03:53
Using cycles for defining a time dilation becomes weirder still if we consider that we all consist of 'matter waves'. Then the question becomes if the distance to the gravity well will have an importance for the 'time dilation'

I don't think it have myself? what should define it is the gravitational metric/potential and that one will differ slightly with the distance, although how much?
Title: What is Time? If there was no light would Time cease to be?
Post by: yor_on on 02/02/2011 23:15:30
And that one neatly solves what a 'time dilation' is.

A 'time dilation' if so, should be a result of gravity acting on matter/waves, compressing them locally. But what causes the 'time dilation' is then not the speed, but the gravity? Still, we need the 'speed' to define a 'gravity' to the space-ship.

And then it should only be accelerating that does it, which should make Lorentz contraction, if still existing with engines of, an illusion :)

Ahhh ::))

Hmmm.

Or, which seems more reasonable to me, considering muons, leave Lorentz contraction as being just as real as a time dilation. Which then should be noticeable as you would see a different, much stronger 'time dilation' for a accelerating object than for a 'coasting', if so. But both would present a 'time dilation' to you, as the contraction is equivalent to a time dilation.

How about lightening up that ship one second, as measured from Earth, into 99.99999% of light speed? Then turn of the engine? What would that be equivalent to time-dilation wise? Would doing ten seconds at ??.???? of light speed give the exact same 'time dilation'.

would I age 'slowest' if I kept a constant acceleration/deceleration as compared to accelerating then coasting decelerating?

I don't know?
But it's a nice idea.
Title: What is Time? If there was no light would Time cease to be?
Post by: Geezer on 02/02/2011 23:19:09
No, I seen some experiments sensitive enough to feel if you moved a clock down on the floor relative leaving its twin on the table recently on the net too. Probably have them somewhere?

Nevermind :)
We seen it both at least ::))

Yes! Thanks, that was it! What's nice about that one is that it's a very simple and static configuration. No aeroplanes or satellites involved, no twins (unless we say the clocks are twins), and no trips around the Universe. The lack of variables minimizes any opportunity for general obfuscation and wavy arm explanations.

If time is an illusion, I wonder how we might explain the results of that experiment?



Title: What is Time? If there was no light would Time cease to be?
Post by: yor_on on 02/02/2011 23:34:28
Time is a illusion Geezer :)

Gravity isn't.
If you was the clock.

Times arrow, aka the 'durations' is the same to you on the floor as on the table, but when looked at from another 'frame', like your wife standing beside you, she will see you age 'slower' the closer you come to the gravity source. I changed my discussion above a little, to fit as I was introducing two frames without thinking of it, so it was slightly wrong, but I actually think it works all the same.

So 'times arrow' becomes an expression of gravity, but the 'durations' of them, although seen to be changing from outside your own 'room time geometry' will to you inside it always be the same.

So sweet :)
And no math..
Title: What is Time? If there was no light would Time cease to be?
Post by: yor_on on 02/02/2011 23:56:11
Then maybe assigning some sort of weird 'density' to Space isn't that bad an idea?
After all, we see it in 'inertia', don't we?
Title: What is Time? If there was no light would Time cease to be?
Post by: Geezer on 02/02/2011 23:59:06
Yes, time (spacetime?) is being influenced by gravity, but isn't it time that controls the motion of the "pendulums" in our atomic clocks, as well as every other process in that locality? Just because it doesn't have a constant effect, that does not mean it's an illusion - in that respect it is a bit like air pressure.

Of course, the clocks don't know anything different. How could they? They are measuring time at their locations.
Title: What is Time? If there was no light would Time cease to be?
Post by: yor_on on 03/02/2011 00:12:21
:)

According to me heh :)
Time should be gravity. I've felt something similar a long time, but not being able to formulate it. Bills and yours comments gave me the right 'push' to see how I could put it together.

'Speed' exists of course, as well as 'distance', and 'time' too. It's on that conceptual plane speed and distance disappear to give place for gravity, as their common nominator.

I've had so much trouble with understanding frames of reference Geezer :)
But now it feels as if I have an idea of what they are.
Ahh, maybe ::))
Title: What is Time? If there was no light would Time cease to be?
Post by: Geezer on 03/02/2011 00:30:03
Were your arms waving when you said that?  [;D]

Look! I'm waving mine too.
Title: What is Time? If there was no light would Time cease to be?
Post by: yor_on on 03/02/2011 00:31:15
It's a very weird universe. It exist in two simultaneous frames as I see it.

One is the one we know of personally. In that one you will have a 'times arrow' that always have the same duration, not caring what you do. In it you will find 'distance'. Even if contracting with your speed, distance won't disappear. And distance combined with your 'durations' will present you with 'speed/velocity'.

Then we come to the fact that your 'room time geometry' change. That's primary caused by the Lorentz contraction. The simplest solution there is to define it as an 'illusion' as we have defined 'time dilations' as being a purely local result of gravity.

If that is wrong the universe becomes more complicated as your 'room time geometry' then will have to include a whole SpaceTime, one unique for each participant/object existing.

So I have to admit that I would prefer Lorentz contraction to be a illusion, as that simplifies my present understanding.

Because I imagine that I now can define a time dilation as being a local effect, 'invincible' for those inside that frame, but getting 'time-dilated' according to those watching, being at rest relative it.

And it delivers some weird facts, if it is true? According to my understanding a 'time-dilation' then only can exist under your acceleration, as soon as you close those engines and start to coast it 'stops'.

And the 'time dilation' is not a result of speed if so, it's a result of gravity. There are more things to it but I need to see if it makes sense first :)
==

And , I don't know if I'm waving yet :)
Possibly, we'll have to see.

But it makes sense so far
Ah, maybe :)

==

Anyway, the other frame of reality is the conceptual.

There we have several possibilities. I said that making Lorentz contraction a illusion would simplify it, and that's true. but on the conceptual plane I'm not that sure. If I treat it as a real effect I will have the possibility of questioning 'distance'. I really need to question some of my ideas, and see if they will work without Lorentz contraction being real.

Awh, but this is still very cool to me.
Let's hope I remember it tomorrow :)
Title: What is Time? If there was no light would Time cease to be?
Post by: Geezer on 03/02/2011 00:44:50

is to define it as an 'illusion' as we have defined 'time dilations' as being a purely local result of gravity.


You lost me there. How does that prove time is an illusion? Gravity influences matter. I hope you're not going to tell me matter is an illusion too.

Also, if time can be dilated (QED) how can time be an illusion?
Title: What is Time? If there was no light would Time cease to be?
Post by: yor_on on 03/02/2011 01:02:33
Time is 'durations'. And those we define using clocks. When in the same 'frame of reference' we will find our clocks to correlate, making us happy. and me finding it hard to explain why I'm always so late :)

The thing with gravity goes back to how to define a 'time-dilation'. If it only exist when accelerating then it can't be the speed, do you agree? If it is a effect of your engines you need to ask yourself what they did? They 'twisted' your 'room time geometry'. And how did they do that?

Gravity.

If it is so, and it's my proposal, for now, that it might be so, then 'times arrow' is a result of gravity, nothing else. 'Energy', as I see it, is a description of relations coming together, so that one I ignore for now.

No, matter, as well as all the other stuff like time, is 'real' to us, just like 'forces'. It's more like there is two realities, the conceptual and the one where I 'exist' physically.
==

If you look at how I discussed that photon wander, from the light-bulb to your eye, in that space-ship, getting blue-shifted, compressed and of a higher frequency then you will see that I equalized that with falling into a gravity-well. To me that is in fact absolutely equivalent, transforming speed into gravity. And gravity is also what keeps SpaceTime together, inertia being its expression locally and 'instantly' everywhere.
==

Matter is what 'creates' SpaceTime. I'm not sure it 'creates' gravity though, for the moment I prefer to look at it as it is what defines the 'gravity' we see rather. But that is just my gut talking :) And my need to find something from where it comes together.

You can look at it several ways. You can define SpaceTime as something created out of matter, gravity being its expression. Or you can define Gravity like that 'black hole' does. As a infinite source hiding inside what we call 'SpaceTime', controlled and regulated by matter. As I said, I'm weird ::))
Title: What is Time? If there was no light would Time cease to be?
Post by: Geezer on 03/02/2011 01:42:08
I agree that gravity distorts spacetime. But if we move our clock to a relatively undistorted region of spacetime, guess what? It will still measure time in that locality.

That's why I like the static case.

I don't think we can use the term "durations" either, as a duration must imply an amount of time.

It's probably a flawed model, but I tend to think of spacetime in a sort of 3-D sense that has more than three dimensions! Matter distorts all of those dimensions (including time). Anything and everything must obey the laws that spacetime imposes at a particular location.

It gets a lot more complicated when things are dynamic, but if we can't figure out the (relatively) static case, who cares  [;D]
Title: What is Time? If there was no light would Time cease to be?
Post by: yor_on on 03/02/2011 01:45:20
Thinking of the 'energy' of that photon. I called it a relation, and it is. What gives it its blue-shift Geezer? As it wanders down that 'gravity well', meeting your eye?

What is that 'blue-shift'? We call it 'energy' don't we :)
So, is that true? Can it do more work when hitting your eye than it could leaving its 'source'? Yep, it can. If so, where did it gain that 'energy'?

It did nothing more than to fall. From its own perspective it has done no 'work'. It's only as it 'interacts' it do any work. So, was work done on it? By what?

Gravity?

No, if that was true we would first of all speak of it as 'interacting' without annihilating. Secondly we know that when a photon leaves a gravitational 'field' it will adapt its energy to what it meets. Thirdly it's the exact same as so called 'potential energy'. You can have a multitude of simultaneous 'potential energies' defined by what object(s) you may interact with, simultaneously.

But there is also this truth. Although the photon may be seen as being a defined 'energy quanta', unchanging. And even though gravity does not 'interact' with it, as that would destroy all definitions we have of a photons interaction, it still delivers a bigger 'punch' the closer you are to a gravity well. And that 'punch' is also real 'work' being delivered.

So what is 'gravity', and what is 'energy'.
Title: What is Time? If there was no light would Time cease to be?
Post by: yor_on on 03/02/2011 01:47:03
Yes, there is nowhere without gravity. But then we have 'energy'. Matter for example store an awful lot of energy.
==

Durations is just my word for regular temporal measurements.

You can make a duration by using a clock. Or your heartbeats, or anything you find to be of a regular rhythm/oscillation. It states that there is a past, a present, and a future. As for it's 'length' that's an arbitrary definition. but 'durations' exist, and as they create that causality chain, we get our 'arrow of time'. They will always be the same inside my own 'frame of reference', no matter what length I define them to have. That is, my wristwatch against my heartbeats will always agree, no matter what frame I'm in measuring.
Title: What is Time? If there was no light would Time cease to be?
Post by: Geezer on 03/02/2011 01:59:16
Yes, but  [:D]

try to take it back to basics.

If matter can distort space, why can't it distort time (which is, after all, part of spacetime)?

Space gets distorted, and so does time. Unless we assume that there is some sort of master clock that controls all time, why would we be surprised to learn that clocks in different locations in spacetime do not agree with each other?
Title: What is Time? If there was no light would Time cease to be?
Post by: yor_on on 03/02/2011 02:08:47
It distort SpaceTime. The question is just if the distortion is real when it comes to Lorentz contraction, or if it is a geometrical twist? I think it's real, which complicates things for me.

The distortion is made through gravity as I see it. We use 'energy' for creating that gravity. I can't think of any other way creating that 'distortion'. Can you give me another way of doing it?
Title: What is Time? If there was no light would Time cease to be?
Post by: yor_on on 03/02/2011 02:56:40
Well, don't take it too seriously. I can see some, very few flaws, with my proposal already :) Like I could be constantly accelerating at one G which would bring me up to near light speed in a year or so :)

Ahem :)

How the he* would having the gravity equivalent to Earth deliver me a 'time dilation' good enough to see the universe 'die'? I have to admit that I was happier before realizing this ::)) So gravity isn't the answer, maybe, and neither is the the way our photon blue-shifts. even though you could use it as an example of all 'waves' moving faster relative some other frame, at rest relative you. :)

So ok, it has to be the whole room time geometry that get twisted, and what does it is 'energy', and speed. and if so? My arguments about acceleration being the only thing creating a time-dilation have to be wrong, you must get it both ways, 'coasting' as well as accelerating.

so yeah Geezer, count it as qualified hand waving :)
==

If there was a way to prove that the 'potential energy' got stored in some manner it might be a different matter. But as far as I know, even though we use the expression, there is no atoms in that spaceship 'jiggling' more due to any stored 'potential energy', due to its speed relative some origin (Earth).

So what exactly creates the 'time-dilation'?
And now I'm far from what we discussed before, our 'arrow of time'.

But I'm stuck here, any takers?
The room time geometry gets 'twisted' by what?
Speed, gravity,'energy'?

How, and where is it 'stored', does it need to be 'stored'?
If it doesn't, and a plank sized man made black hole can be made.
Containing a infinite 'gravity', what guarantees it evaporating?
Shouldn't that center see our universe 'die' first?

Ahh, better stop this :)




Title: What is Time? If there was no light would Time cease to be?
Post by: yor_on on 03/02/2011 04:01:05
Okay, some points.

One gravity, constantly accelerating, will not be equivalent to the same amount of time passed on Earth. That's a simple one.

So, what distort the room time geometry?

If it is motion solely, then 'coasting' is good enough.
If uniform motion is good enough, and there exist no gold standard for what speed that might be in black box. then any uniform motion is equivalently 'time dilated'.

The way around that is to introduce a new 'frame of reference' relative your 'black box'

Using that you can define a speed. but you can't define who is speeding relative who. but that's not really true, you can always use relativistic effects to see if it is you or the other object, or can you?

You should be able to use CMB (cosmic microwave background) for it at least?

So let's assume that you can define a speed to your 'coasting' relative the CMB, that makes it as real as it can be, relative the universe, am I right?

But I'm coasting? I'm not expending any energy so I can't say that the time dilation is a effect of me expending energy?

And I have no gravitational forces acting on me. A 'free fall' and me coasting is the same as I understands it, no matter what velocity I measure relative something else. Can you prove that wrong?

No gravity, no energy, but a time dilation?

What have I left, 'speed' yes, but only under the definition that I have found a way of proving it relative the universe.

So is time dilation a relation of 'positional aberrations' only. How does the universe know my 'speed' or that I 'distort' its 'room time geometry'?

You can use light to prove it, and we say that the muons also proves it. But what creates it?

==

Also, if you accept the muon example you will find that it is in a 'free fall' too, no accelerating forces acting on it, although it will 'accelerate', you can put a scale under its metaphorical feet to check if there is some 'gravitational effect' and you won't find one. So even though it relative the ground is gaining 'energy' it will, relative something 'coasting' beside it, being at rest relative it (same frame of reference) have gained nothing at all...

Getting tired here.
Miss spellings and stuff.

Anyway, the muon is just following a geodesic, even if  in a rather big 'dip' (One Gravity deep)equivalent according to relativity to any other geodesic.

And yep, I know the statement that being on Earth is the equivalence to one gravity constantly accelerating. That is true, if you define it in a black box, ignoring tidal forces. But the time dilation created will differ as observed from another frame of reference, and what phreaks me out is how it comes to be..
Title: What is Time? If there was no light would Time cease to be?
Post by: Geezer on 03/02/2011 04:11:33
Ah, but gravity is not constantly accelerating. The acceleration varies, and that variation produces the variation in time.
Title: What is Time? If there was no light would Time cease to be?
Post by: yor_on on 03/02/2011 04:19:04
Geezer, in this case gravity has nothing to do with it :)
And don't I hate that :)
Title: What is Time? If there was no light would Time cease to be?
Post by: Geezer on 03/02/2011 05:00:06
Which case? I'm still on about the two clocks that are obviously influenced be gravity.

You'll need to slow down if you want me to keep up with you.
Title: What is Time? If there was no light would Time cease to be?
Post by: yor_on on 03/02/2011 05:03:08
Yeah, ah, what clocks?

:)
Title: What is Time? If there was no light would Time cease to be?
Post by: yor_on on 03/02/2011 05:47:42
The really weird thing is that according to this we're all time dilated. there is no way any 'inertial frame' can be defined to a certain speed. We don't have any zero speed in the universe. so we must all contain some 'time dilation'.

That I used the CBR is possibly legal though. I'm not sure there? Probably not, I will have to look that up tomorrow :)

I hope to find some really enlightening answers here tomorrow. And by the next few days we will have us a TOE, and a big one too.

I already have two btw
But one can never get enough of them, they say?

Or can there only be one?
Title: What is Time? If there was no light would Time cease to be?
Post by: QuantumClue on 03/02/2011 11:08:12
I've lost track here of what the conversation is about any more.
Title: What is Time? If there was no light would Time cease to be?
Post by: yor_on on 03/02/2011 17:02:23
It's just a matter of time QC :)
Soon we will all get it.
==

And when it comes to CMBR

"Since its discovery in 1965, the radiation has been carefully studied and found to be a perfect blackbody as expected from theory. Since, this radiation represents fossil radiation from the initial big bang, any additional motion of Earth around the Sun, the Sun around the galactic center, and the galaxy  through space should be reflected in a slight asymmetry in the background radiation. The net motion of Earth in some specific direction should be reflected by a slight Doppler shift of the background radiation coming from that direction toward shorter wavelengths."

So maybe it would work.
Title: What is Time? If there was no light would Time cease to be?
Post by: yor_on on 03/02/2011 17:21:51
The thing irritating me is that I can explain it in form of light, but I can't get my head around what actually creates it. And to me it's also close to all other ideas we have, and cherish :)

Like 'potential energy' and 'energy'.

So do anyone know what creates it, and don't give me 'light clocks'. I have enough light clocks to dress a castle :) and the math may work but it does not define what creates it. The closest I've come I think is my 'positional aberrations' and why that should create it? I don't really know. Wavelengths we already tested in the thread, and that is not sufficient, as I could see? Gravity alone seems not the sole answer, although my own opinion still leans toward it as a answer in some weird way.

'Energy'? Well, let's define 'energy'. what the he* are we talking about there? Light quanta? Is that 'pure energy'? I don't see it that way. 'Energy' is what we use for describing 'work gained' and it comes in different amounts depending on its relations, as we can see when it comes to blue/red-shift. So 'energy' have to be a description of a relation, nothing more. Just as 'potential energy' is.

So what defines those two, from 'potential' to 'real'. Their interactions, do you agree? Then we come back to a 'time-dilation' and the question of time. What defines a time-dilation?
==

One of the really big points I expect that we can make already is that if we assign 'times arrow' to 'oscillations' then those should be apparent in our descriptions of that spaceship relative something being at rest. We use light in simplified description for what time consists of when describing a 'time dilation' but when looking at it as 'matter waves' they still seem to fall short of the real dilation? Or, can you show me the oscillations?

Let's make another experiment.

We will use a mirror pair, or anything with a defined amount of 'oscillations' mounted inside that ship. assume now that 'times arrow' have to have a direct influence on those oscillations. It should, shouldn't it?

So the 'light-corn' have a 'bumping frequency' of ah, ten per second at earth. As we start to move that oscillation must slow, as observed form Earth. Every discrepancy in the twins age relative each other, slowing as the ships speed accumulates, need to be answered by those oscillations to a equal degree, as observed from Earth. Let us assume that this is correct.

So would 'positional aberrations' be the correct nomenclature for it? Not really, we can get the same effect one meter from Earth than one million miles from it. So that is not correct either. So what does it leave us with?

Its speed relative Earth it seems. That gives us a universe where it will be very important to differ between whose speed is relative who. It will not be enough to say that A:s uniform speed relative B is equivalent to B:s speed relative A. From the viewpoint of a 'time dilation' they must differ, do you agree? If they don't differ both will have to be equally 'time dilated' which definitely would make time something very strange, with a very weird 'arrow' at least.

But I wasn't satisfied with those 'waves' was I :) That was when I tried to use them relative the 'gravity well' created by an acceleration. If you look at the balance there the gravity seems very weak and 'out of bonds' compared to the 'time dilation' possible by a constant acceleration.

So the waves created for a 'time dilation' according to that idea, inside the ship, looked at from the viewpoint of a photons blue-shift inside, is not sufficient it seems. But the 'time dilation' still exist. And how did I 'prove it'? By comparing frames, Earth looking at that bouncing light-corn, mounted inside the ship. And that ship could be one meter from Earth, as long as it held the same speed relative Earth.

That one is very weird in fact. Assume that photons blue-shift inside that ship relative you. Then define it as you age slower, as compared to those photons. But you too can be seen as composed of 'matter waves'. And those waves should should follow the same pattern as any other 'wave'? which then should mean that your waves will red-shift relative the photon, as long as it is further from the gravity-well, but then blue-shift as soon as it passed you. So your 'aging' becomes a positional effect only, But no less 'real' because of that.

so yeah, 'relations'
==

Another thing worth noting here. When we are talking about a gravitational time-dilation we can illustrate the gravity by imagining a photon blue-shifting. What that means is the the photons ah, 'clock', relative us on Earth 'ticks faster'. As it 'ticks faster' per 'duration', let's say 20 oscillations per second instead of 10/second it, according to us, will have to age faster, alternatively defining ourselves as 'aging slower'. depending on how you want to look at it.

So, am I right after all?
Gravity is time?

But what the he* would speed be if so?

(Better point out one thing here, I'm using the photon as waves in that example, and then referring to the frequency as equivalent to a 'time-dilation'. But I do not think a photon 'age', and I doubt I ever will. It's just a thought experiment.)
Title: What is Time? If there was no light would Time cease to be?
Post by: yor_on on 03/02/2011 18:31:32
So, we are getting closer possibly, but, what is it? I would like to describe it as a relation, as my universe seems to be created by, and populated by, 'relations' from 'energy' to 'time'. But the problem with that is how to explain how the interactive relation between Earth and that ship creates a real difference in age, in those twins?

As Geezer said, 'motion is time'. But exactly how is it 'time'?

Inside that ship I would assume everything to have a slight 'time dilation' as created by the constant acceleration, equivalent to a 'gravity', and as seen from the photons 'blue-shift' observed by the participants inside.

But that 'time-shift' is not enough to explain what Earth will see in form of the 'bouncing/oscillation' increase inside that ship. If we satisfied us with defining it as a geometrical aberration, made by the speed relative us then we might ignore it as a illusion. But according to relativity it isn't, it's real.

So, speed makes 'time slow' relative the universe, but only observable when comparing frames. If this is correct the universe must have a simple way of differing what frame is what, and whose time is which. How does it do it?

I define one unchanging frame of 'time', being your own, which will give you an exact same expiration date, no matter what anyone else will tell you about your appearant 'age' relative something else. Is that one wrong then? No, I'm sure of that one, it have to be true.
==

How about this then?

To make the universe able to differ those frames it need to have a 'gold standard' of 'time'. What about it, could you use my idea of those personal 'unchangeable expiration dates' as a such? That one is nice, isn't it? :)

(Not the cosmic microwave background for this. That one was for our benefit only)
====

But how could it 'use' it, if that one would be a possibility?

That must have to do with how we define it? It's like a puzzle in where you can lay the pieces different ways, they will connect and give you a picture, but there will always be another way to lay it, no way being the 'single truth' as I suspect. But I expect some ways to describe it better, hopefully giving us a clearer idea.

As far as I can see?

It's like assuming 'energy' to be 'something' by itself. Well, yeah, to me it is a sum of its relations? And some things, however useful they may be mathematically, do not 'exist' to me. Like 'potential energy', you might say that to me 'potential energy' is a description trusting in the existence of a arrow, as it, to become true, will have to interact before 'existing'. I differ between interactions and possibilities. A possibility is to me the idea of something happening in a arrow of time. A interaction is a 'event', passed and proven by our arrow to 'exist'. The past contains all what's necessary to build what you call your present. And to me the future is only expectations, built on those past interactions that we've observed to be true (existent), and then statistics.
Title: What is Time? If there was no light would Time cease to be?
Post by: simplified on 03/02/2011 19:38:45
Motion is speed.Quantity of motion is time. [8D]
Title: What is Time? If there was no light would Time cease to be?
Post by: yor_on on 03/02/2011 19:44:14
Ahh, then you and Geezer might agree?
Okay.

So what about 'gravity'?
Title: What is Time? If there was no light would Time cease to be?
Post by: simplified on 03/02/2011 20:04:52
Apples in Geezer's garden  are their quantity, but motion is not quantity of motion. Because the quantity of apples can decrease or increase.  Quantity of motion only increases.
 Here I am wrong,because quantity of  created apples only increase.
Title: What is Time? If there was no light would Time cease to be?
Post by: yor_on on 03/02/2011 20:08:54
Try another example Sim, I'm not sure what you mean by 'quantity of motion'. Are you imagining the apples containing 'frozen motion'?

So if motion is 'time' and matter is 'motion' in some other state, what is gravity?
Title: What is Time? If there was no light would Time cease to be?
Post by: simplified on 03/02/2011 20:48:42
Try another example Sim, I'm not sure what you mean by 'quantity of motion'. Are you imagining the apples containing 'frozen motion'?

So if motion is 'time' and matter is 'motion' in some other state, what is gravity?
Quantity of motion is time.Quality of motion is speed.I do not know what is gravity.
Title: What is Time? If there was no light would Time cease to be?
Post by: Ron Hughes on 04/02/2011 22:20:55
I'll make a prediction. The speed of light slows as the Universe expands and has been slowing since the BB. The less dense (mass per cubic meter) the Universe becomes the faster it's clocks will run hence the slowing of c.
Title: What is Time? If there was no light would Time cease to be?
Post by: yor_on on 05/02/2011 01:01:04
Let's try a new track.

Assume that SpaceTime is a whole thing, whatever that means. Like some blob of jelly, in that blob we have four properties we can see. Length, width, height, time. Those four are needed everywhere.

Then assume that they have some sort of 'proportion' to each other and that by doing one of them you will affect the other. Like speeding :) So you speed away.. Now, three of them are interchangeable. Width, length and height are just names for one thing.

Distance.

Then we have 'two' things to look at. Distance and time. Those two combined will give you a position. to get the position you will need all three types of distance of course, but they all represent the same. You don't need 'time' for that, even though it is meaningless introducing a position without involving the arrow i believe you can do so on any surface/dimensions theoretically. But time will get introduced when you move, you can't move without involving 'times arrow counting' durations.

For this we also have to assume that none of those dimensions are special. So when you move you simply change the jelly, pushing three dimensions in one way, and time in another. Does this explain it? I can't say it does. It depends on how you look at the universe of course. Me, I look at it from expending 'energy' and from that view a time dilation, as it actually involves less energy consumed by one of the twins relative the other (Younger than) there has to be some equivalence to the 'energy won' by him, if you like.


But I also look at 'energy'  as something only existing as a relation to what defines it. You will get a 'time dilation' from gravity too, and as the 'gravitational potential' even might be outside the invariant mass in a complicated system, as a guess here :) I could be at rest relative some system in space, moving uniformly, still having a stronger 'time dilation' than any of those objects of invariant mass.

So where is the 'energy' in that? You might assume that gravity then is a form of 'energy' on its own. Which actually make sense to me :)

If you assume that infinite gravity equals timelessness then all gravity is a time dilation. But it doesn't take care of speed, not without introducing 'invisible/non existing' things like relative mass/momentum/'energy'.

And all of those need an interaction to be true.
==

There are two ways too look at it. From the conceptual plane you can define infinite gravity as being 'timelessness', watching the object hovering forever as it closes in on that center of gravity. Then there is the viewpoint of the object itself. There, as far as I understands it, times arrow never stops ticking, and what you observe as no motion, equaling 'timelessness' that object will see as both a motion, and durations.

As I said, no motion without durations.
Title: What is Time? If there was no light would Time cease to be?
Post by: yor_on on 05/02/2011 01:50:28
So what exist without needing relations to be shown? But before we do let's put down some ground-rules. Although everything may interact through radiation there is a simple and definite difference between that, and what we observe as being 'here' in a prolonged manner. And you just have to find those socks, just where you put them last night to confirm that(fat chance of that though:).

Distance? that one is really interesting. Is Lorentz contraction a geometric illusion or is it real. but still, even if Lorentz contraction exist distance will still be needed for matter, how ever contracted it might become. And that should mean that matter and distance have a 'reality', independent of anything else. So matter exist, and 'distance' exist, even though 'plastic', changing according to invariant mass and speed. What more exists independently? Light?

I don't think so myself. Light only exist in its interactions, that we find it to have a speed is not a proof of it existing independently. Remember that I'm speaking from the inside of our jelly discussing the idea of 'independently' here. Meaning that I look at what I could expect to exist inside without needing to be defined by anything else but as seen from the inside. I do have some other ideas about light, but those are not as seen from inside our jello. And I'm jumping between concepts the whole time, but I have a purpose with it, I hope :) Anyway, speed becomes, as we can see conceptually, only a expression of what frame you are in. Then again, we have one frame where times always tick, the one you exist in, and in that one speed must exist as long as you're made out of matter. But even so it doesn't prove that light has one, as you only can observe it in its interaction. Light quanta is not your socks.

There are two ways here of course. You might want to define it as the only thing being sure is those light quanta, as nothing can exist without them confirming it through their interactions, be it touch, looking, or measuring some other way.
==

So yes, in one way you can define both speed, distance and matter to have a existence 'independently' as they never, not even nearing a infinite gravity will totally disappear. They can't. That I define 'timelessness' as 'infinite gravity' doesn't mean that any of those properties can reach that state, from any frame. Except when compressing, aka a Black Hole that is. But not by speed.

What would happen if the universe suddenly got a infinite gravity? As far as I can see the whole universe would, from a 'outside' observer, contract, maybe under what would be possible to measure for that observer. What would happen for us inside that universe then?

Assume no new 'tidal forces' coming for it, would we all disappear, becoming infinite gravity too, would we get event horizons? If we would then the universe still would need to exist.
==

So what differs a compression from speed? With a speed there will never be a state where you won't be observable from another observer, no matter how near you come to lights speed in a vacuum. With a compression there comes a 'state' in where the universe closes you in behind that event horizon and where we can't know what's happening anymore.

How did it do that?

It eats the radiation. Assume that this universe of infinite gravity exist, eating us. If there is no event horizon(s), where is its center? Assume you were alone in that universe of infinite gravity, no tidal-forces acting on you, would you notice a gravity? For this you might assume that the 'gravity' is equally in all points.

Would it exist to you?
Title: What is Time? If there was no light would Time cease to be?
Post by: Geezer on 05/02/2011 02:11:55
Assume that SpaceTime is a whole thing, whatever that means. Like some blob of jelly, in that blob we have four properties we can see. Length, width, height, time. Those four are needed everywhere.


Works for me. I'm glad to see you are finally coming round to accepting my atmospheric pressure analogy  [;D].

BTW - what flavour is the jelly? Personally, I like lime best.
Title: What is Time? If there was no light would Time cease to be?
Post by: yor_on on 05/02/2011 02:17:10
Ha, just you wait. I'm going to turn it outside in :)
The jelly should be lime, shouldn't it?
Title: What is Time? If there was no light would Time cease to be?
Post by: Geezer on 05/02/2011 02:23:10
Careful now. We don't want to accidentally rip the fabric of spacelime, do we?
Title: What is Time? If there was no light would Time cease to be?
Post by: yor_on on 05/02/2011 02:31:00
Nah, turning the universe outside in should mean that it still is 'whole', just outside in :)
Maybe ::))
Title: What is Time? If there was no light would Time cease to be?
Post by: yor_on on 05/02/2011 02:42:22
There is a definite difference between 'gravity' and anything else I can think of. If we look at SpaceTime as this ah, lime-jelly then that jelly will exist for an 'outside' observer no matter what speed he deems our universe to have. but with a 'infinite gravity' acting on it I expect him to see it disappear.
==

And one more thing. As seen from where we are we exist, and time, speed, distances, light and matter, and space all play a game defining limits for that existence. And the only thing I see crossing that boundary defining SpaceTime is gravity.

Compression is another description, is that the same? Nope, compression is a definition we use for describing matter under pressure. The pressure being gravity in my example. So, could we compress something into a black hole without using gravity? I don't know, but I know that the last state will be gravity alone, taking care of the creation of a Black Hole.
==

I called speed and distance, and matter 'independent' didn't I? I'm not sure of that description, they are independent in that they seem to be able to exist inside the jello without 'disappearing', having a true existence if you like, unlike our concept of 'energy' that I only can see as a expression of relations coming true. On the other hand they all seem to be relational to the jello 'SpaceTime', don't they? But Gravity seems to be able to 'exist' on its own even without a SpaceTime. And then we have 'light'. If I look at light as the traffic police of SpaceTime, setting the speed limit then does that mean that light also show this adaption?

Nope.

In a way light is even weirder than gravity. Where gravity is one thing only, well, as I see it :) light is two things, or more. Light have only one speed, the same from wherever you look. But it also have this 'frequency' that also is its energy, adapting to all other frames. We also find that light can be something not having any definite boundary at the same time as it express itself as 'light quanta' aka 'photons' begetting the same finite properties as all other 'particles', well, except a 'restmass'  that is.

So if I want to look at particles, how should I do it? How do we do it? We measure by radiation, or by other particles of restmass, right? We take a massless timeless point particle and 'bounce' it, or a particle of restmass. And what we use is either 'kinetic energy' or momentum. You can also point out that we use 'energy' 'frequency' for it, but as I doubt 'energy' as anything else than a relation, well, maybe, to me you can see it several ways? But however I see it light exist, and it have a 'energy' that can differ. So how do we create it? Its energy I mean. We use a relation, something, a laser pumping maybe, A cyclotron using EM. We set the 'energy' by that, and of course by what way we let it interact. But if you look at a particle my way it is the relations that define it.

And light is weird.
==

How did we define a 'time dilation'? By comparing frames, right? How do we compare frames? By radiation. That was the way Einstein realized that there would be a 'time dilation' too as I understands it. By looking at the radiation, and the way it only presented one speed, in, and from, any frame of reference chosen. It didn't matter if you already was traveling at 99.99999~ of 'c'. We on earth would still see the light from you coming at us at 'c' no matter if you were traveling towards us, or from us. And to make sense of that he looked at 'time'. Adapting 'times arrow' as compared between frames of reference made it possible for light to behave this way. But it also meant that 'time' no longer was a isolated entity, for it to adapt as it did he had to assume SpaceTime to be a 'whole experience'. It meant that we no longer could expect 'time' to be one unchanging continuous 'force' sweeping the same through SpaceTime. But there is one catch to it as I see it, to you, or me, or anyone it still is that 'force', if we just stay inside our own 'frame of reference'.

So radiation defines a 'time dilation', but to do it we need to compare frames of reference, or bring back the twin to Earth, as in the twin experiment, to actually see the biological difference created by the speed. Does this mean that any of those twins found their time elongated or compressed? Do you find your time being such? Well, as far as I understand it is both, simultaneously, you just need to choose the right 'frame of reference' to compare yourself against, and you can measure both if you like. Assuming that this is correct, have you now lost your past? Your present? Your future? Did your 'arrow of time' jump out the window as you read me :)
Title: What is Time? If there was no light would Time cease to be?
Post by: briligg on 05/02/2011 17:50:37
Plowing through this thread in order to get up to date has been really hard, and i admit to skimming sometimes... I take it the last 3 pages are thought experiments directed at building a model in which an arrow of time objectively exists, as a function of gravity, perhaps. I also take it that the kernel of the vast discord in this thread is objection to the idea that time is an illusion. As physicist like to have at least a provisional answer to questions as huge as 'what is time', even though currently physics is completely unable to answer that question scientifically, I think it is fair enough to call it an illusion. They have plenty of well established, well developed fundamental theory that implies it does not flow. All the formulas dedicated to calculating how it *does* flow in our perceived reality are not concerned with the fundamental nature of the universe, just with how it apparently functions. But 'illusion' is just a fill-in assessment as investigation continues. There is no explanation for the arrow of time, so it has to do. With your thought experiments, you have just tried to flesh in a more mentally comfortable structure, but in order to do that, you have had to engage in a huge amount of speculation. Some people prefer to solve the dilemma in this way, some prefer to use QC's method of sticking to what has been established so far, however uncomfortable and inadequate the resulting conclusions are. Humans are weeny, tiny, insignificant creatures. We will never know what is really going on in the Universe. I believe it is important to remember that, in discussions like these, in order to avoid getting all tensed up about it. After all, if there really are beings out there whose understanding are completely beyond our own (i vote there are, on another question in which nobody can claim to know) they would look at our musing on time and chuckle tolerantly. Or be bored. Or crush us with their cosmic pinkies. Something like that.

That said... i am also tremendously interested in the question. I find it pleasant to think of time as a landscape where past and future always exist, even though we for some reason cannot perceive that. I tend to think there is more than a psychological reason for that, as psychology is much less fundamental to reality than physics (unless you assert there is no such thing as objective reality, which is certainly logically defensible). The model i come up with in order to feel cozy with the idea is that our universe is sort of like a wave moving through the dimension of time, like how if you passed a 2D plane through a 3D object, the figures on the plane would constantly change, but what was visible on it at any moment existed for the entire time, and was not moving at all.

I can't find it, but i remember reading a proposition that the double-slit experiment can be nicely solved if you remove time. The patterns on the screen are due to the fact that the photons (or atoms, or what have you) are connected to the future and the past, so the fact they are measured in the future naturally affects the path they take. Feynman also was comfortable with explaining ramifications of his Feynman diagrams in terms of some particles moving from the future to the past. It comes up in the lectures of his that are available online - http://www.vega.org.uk/video/subseries/8  . The April 2010 issue of Discover reported experiments by Tollaksen and Aharonov showing future measurements affect past measurements. It is available through coverleaf.com, but you have to buy it. I'm going to try to put in an excerpt in my next post.
Title: What is Time? If there was no light would Time cease to be?
Post by: yor_on on 05/02/2011 17:59:18
Yep, you're right. It's all thought experiments, from us all :)

So, what defines SpaceTime?

Radiation defines it, everything we do, everything we see communicates through radiation, right? And radiation have a 'speed'. How do we know that radiation have a speed? Well, we use the properties we can see inside SpaceTime, like distance and time. Then we say that everything '¨moves' relative each other and that there is no way we can define anything as being at rest relative each other. That's one way of looking at it but not the only one I think.

We already know that 'gravity' can operate outside SpaceTimes boundary's. That we assume it to communicate is a direct result of our idea of SpaceTime as place where nothing is 'lost', and where the conservation laws have a reality of their own. So, how do a Black Hole 'communicate with SpaceTime? This is my view okay :) I say it doesn't. The Hawking radiation may set a spin by its pair-particle twins annihilation but that spin isn't read by anyone. the idea of a communication is only meaningful if it gets 'read'. It's not enough with assuming that it 'exist'. If nobody had heard Einstein, would the information exist? Yes, on the conceptual plane it would, but for you, here and now, and your sons? And their daughters? That's number one. Then we come to the idea of how it 'communicates'. Nobody more than me having a problem with how we define that spin? If we define it by a measurement what do we mean by that?

That it only can be when a human look at it that the wave function collapse and a 'final' state of that spin comes to bear? how about a dog? How about a steel plate stopping our particle? How about a 'beam splitter' splitting a photon? How can we guarantee that this didn't set a spin? That we will see a opposite spin is true, but that will be true any which way, won't it?

Then we come to the idea that the universe is 'meaningful'. That is the assumption you make when you define an idea of the Universe that in its 'smallest parts' containing 'information' that also need to be 'conserved' for the universe to 'work'. I agree in that the universe for us is 'meaningful' but I'm not as sure if it is our ideas of meaningfulness that defines it. I look at the Universe as a place where it's our mission to find out what that 'meaningfulness' really is. And I expect the universe to have a different conception than us. We have already changed those ideas of meaningfulness several times through history and I'm sure we will do it again.

So is the definition of what spin that particle twin have 'meaningful' to the universe? In a way maybe? We might expect that in 'n' number of annihilations it should be a fifty percent statistical even distribution. If it isn't then that should tell us something about the 'rules' we play by. So that way it should be 'meaningful'. But do the universe need to set every spin to a preset pattern? I don't expect it to need that.
==

And yes, we're still discussing 'time' and 'time dilations'. And I agree, that's my point of view too. That there is a 'arrow of time'. You might want to define it as process of 'decay' like entropy, but it won't chance the facts. Inside your own 'room time geometry' there will always exist a 'arrow of time'. That one is very easy to see. And without it nothing would be able to be discussed. When it comes to Feynman diagram they rest upon the 'time reversibility' we see on a quantum level. That reversibility is a sign of what 'time' is too, just as time dilation is. But so is the arrow you actually have. One can't just pick two and ignore the third, at least I can't?

So the arrow exist.
Title: What is Time? If there was no light would Time cease to be?
Post by: briligg on 05/02/2011 18:28:57
Coverleaf won't let me copy and paste from their site, so i have to put in an image:
(https://www.thenakedscientists.com/forum/proxy.php?request=http%3A%2F%2Fwww.briligg.com%2Fimages%2Feternal-time.png&hash=558bb02ca7fa9dafc6ef6003e8ff22a2)
Title: What is Time? If there was no light would Time cease to be?
Post by: briligg on 05/02/2011 18:44:04
Quote
We already know that 'gravity' can operate outside SpaceTimes boundary's.
How do we know that?
Quote
If nobody had heard Einstein, would the information exist? Yes, on the conceptual plane it would
The conceptual plane? As in, an actual plane that actually exists? I know the human mind really can't get away from that way of looking at it... but we aren't proposing it's a 'place', are we?
Title: What is Time? If there was no light would Time cease to be?
Post by: briligg on 05/02/2011 18:51:49
If i understand correctly, yoron, you discuss 'measurement' and 'meaning' in order to show the inadequacy of the interpretation of the double slit experiment as showing that human interaction affects reality. But if past and future always exist, their is no need to think what people 'measure' affects what is real. The particle/wave behaves according to what its past, *and* its future dictates. What it does is interact with its environment, including the dimension, if you will, of time.
Title: What is Time? If there was no light would Time cease to be?
Post by: yor_on on 05/02/2011 18:58:18
Well, yes briligg. As I see it there's a lot of stuff we define conceptually. Time dilations, entanglements, how we think it works on a Quantum level etc. And it has a 'reality' too, even though sometimes at a right corner to what we see normally. the 'reality's' clash, don't they :) If you're a convinced QM you will defend that 'reality' above all others. If you want a reality based on what we see macroscopically you will 'defend' that.

But that you know already, right?
==
As for how a black hole operate 'outside' SpaceTimes 'boundaries'? It depend on what you think information is, don't it :) But it is also a fact that a 'Black hole' is a 'singularity', closed to SpaceTime. If you want to define it otherwise you need to show me that you can. I would love to see that proof, and yes, it will need more than string theory, like a actual experiment?
Title: What is Time? If there was no light would Time cease to be?
Post by: yor_on on 05/02/2011 19:01:48
Okay, that one suits me fine. But I need to take up some history first. Let's first look at entanglements. Einstein was the one that with Podolski and Rosen set up that experiment. He did not like the idea of a 'spooky action at a distance'. "The idea is this: set up an interaction such that two particles are go off in opposite directions and do not interact with anything else. Wait until they are far apart, then measure the momentum of one and the position of the other.

Because of conservation of momentum, you can determine the momentum of the particle not measured, so when you measure it's position you know both it's momentum and position. The only way quantum physics could be true is if the particles could communicate faster then the speed of light."

That is the essence of the EPR experiment, not the spin, even though the spin is as described. The Copenhagen interpretation of this, according to Bohr is that before its measurement the particle exists in a superposition of all possible states. " Bohr liked the idea and used it in his Copenhagen interpretation. Remember our experiment with electrons? Each possible route that the electron could take, called a ghost, could be described by a wave function.

As we shall see later, the "damned quantum jumping" insures that there are only a finite, though large, number of possible routes. When no one is watching, the electron take every possible route and therefore interferes with itself. However, when the electron is observed, it is forced to choose one path. Bohr called this the "collapse of the wave function". The probability that a certain path will be chosen when the wave function collapses is, essentially, the square of the path's wave function. Bohr reasoned that nature likes to keep it possibilities open, and therefore follows every possible path. Only when observed is nature forced to choose only one path, so only then is just one path taken."

And the equations Bohr saw supporting him was primary Erwin Schrödinger's wave functions. "Erwin Schrödinger created quantum equations based on wave mathematics, a mathematical system that corresponds to the world we know much more then the matrices. After the initial shock, first Schrödinger himself then others proved that the equations were mathematically equivalent.

Bohr then invited Schrödinger to Copenhagen where they found that Schrödinger's waves were in fact nothing like real waves. For one thing, each particle that was being described as a wave required three dimensions. Even worse, from Schrödinger's point of view, particles still jumped from one quantum state to another; even expressed in terms of waves space was still not continuous. Upon discovering this, Schrödinger remarked to Bohr that "Had I known that we were not going to get rid of this damned quantum jumping, I never would have involved myself in this business."

And then we have the Pauli exclusion principle splitting what it mean to be of 'restmass' (electrons/Fermions) and of no 'restmass' at all, like photons (Bosons)and perhaps the most important of them all? The Uncertainty Principle where Werner Heisenberg in 1925 proposed that it is in fact physically impossible to determine both the position and the momentum of a particle at the same time. "if you determine an object's position with uncertainty x, there must be an uncertainty in momentum, p, such that xp > h/4pi, where h is Planck's constant."
Title: What is Time? If there was no light would Time cease to be?
Post by: briligg on 05/02/2011 19:05:53
Quote
As I see it there's a lot of stuff we define conceptually. Time dilations, entanglements, how we think it works on a Quantum level etc. And it has a 'reality' too, even though sometimes at a right corner to what we see normally. the 'reality's' clash, don't they :) If you're a convinced QM you will defend that 'reality' above all others.
Maybe that's heaven. [:D]
Title: What is Time? If there was no light would Time cease to be?
Post by: yor_on on 05/02/2011 19:09:24
Yeah :)

It's fun, isn't it ::))

When it comes to 'weak measurements' it have a one fault, maybe two, as I see it that is. It still involves a, or many, measurement(s). And it build on a conceptual truth, statistics, don't it? Then, on the other hand, is there any other way to define reality, than 'statistics'?
==

But at least we now have an idea of entanglements and the history behind it.

The point being that entanglements 'exist'. It, as far as I see, do not state that it have 'meaningful' information. We can't know, for example, its 'spin' before the measurement. If we could then that would make it 'useful' for us, and that's also what I mean by 'meaningful'. 'Meaningful' is a relation to 'useful' is a relation to 'what we can use'. So when we speak of the universe we want it to make sense from our perspective, and also to be 'useful' to us. There I think we are applying preconceptions of ourselves as the 'crown of evolution' to justify an expectation in where everything needs to make 'sense'.

Which nevertheless won't stop me, or you, from continue to try to do so :)
==

Statistics is in one way very close to constants. Constants are the universe's statement of 'what is'. It stops there, light makes 'c' a 'barrier' for no specific reason I know? The Feigenbaum constant defines a sudden linearity in a non-linear system, where the bifurcations (paths splitting)taken inside each 'constant' still will be impossible to backtrack. The list is long. Then we have Statistics, That, even though all about probabilities, still will tell me that some things are so 'probable' that I never have to worry about them. Like, for example, all oxygen moving to the corner of my room and wait for me there, that won't happen. At least I've never had it happening, yet?:)

So what differs a constant from probability? A constant just 'is' as I understands it, probability still opens for a small possibility of something different happening. So we have a 'statistical' reality conceptually, but with its borders staked by the 'Constants'. What does that remind you of? Monopoly, ever played that? Or a RPG? Same thing, certain borders set that you're not allowed to 'step over', and then inside those a 'fluid' reality in where all is possible, as defined by the 'rules'. If you look at it that way, then it's not really necessary for it to make sense, as in being 'useful' to us. The 'game' is to hack the game.
Title: What is Time? If there was no light would Time cease to be?
Post by: briligg on 05/02/2011 20:20:31
Alright, i'm struggling with this... i have no physics training, so i am trying to take it all in..

So, this work of Pauli, Hiesenberg, Bohr and Schrodinger, means that even if time is considered as static and eternal, and everything in the arrow of time is responding to its past and its future, there is still the problem of what determines its behaviour? What a 'measurement' or an 'observation' is? That even if awareness of the past and the future of a boson or fermion was possible, the Uncertainty Principle would still prevent you from knowing both the position and the momentum of a particle at a particular moment?
Title: What is Time? If there was no light would Time cease to be?
Post by: yor_on on 05/02/2011 20:27:43
So what defines reality according to the Copenhagen interpretation?
Consciousness?

Only when something 'thinking' looks? Well, that ain't true macroscopically at least. There we have objects interacting at all times, without us ever looking at it, like those sock stealing midgets I believe live under my bed, or not :) And at a QM level? what would make the paths come true, without our measurement? It would have to crave some tremendous 'micro management' from us all, wouldn't it, if it was us that was needed for anything to come true?

So I prefer to expect things to do just okay myself, whether I'm there or not. And that makes me wonder about 'entanglements' and 'beam splitters', possibly setting a 'spin' before our 'measurement'. The weak measurement seems to expect it to be possible to measure without measuring. If that is a truth then we need something explaining how the universe defines what path it will take without a measurement.

And that I think of as 'energy expended'. The path of least 'energy expended' will be the one taken, as a guess.
Title: What is Time? If there was no light would Time cease to be?
Post by: yor_on on 05/02/2011 20:32:03
Alright, i'm struggling with this... i have no physics training, so i am trying to take it all in..

So, this work of Pauli, Hiesenberg, Bohr and Schrodinger, means that even if time is considered as static and eternal, and everything in the arrow of time is responding to its past and its future, there is still the problem of what determines its behaviour? What a 'measurement' or an 'observation' is? That even if awareness of the past and the future of a boson or fermion was possible, the Uncertainty Principle would still prevent you from knowing both the position and the momentum of a particle at a particular moment?
==

Well, it's about 'paths taken' before we look, and then with us looking one 'path' being , more or less forced. But it's also about if we really need to look for a specific path being chosen, or if there is some principle defining it even without measuring.

And yes, it's definitely about what 'time' is :)
Title: What is Time? If there was no light would Time cease to be?
Post by: briligg on 05/02/2011 20:42:28
Measuring always involves physically interacting with a system, right? So couldn't it be that some sorts of interactions force the particle to take only one path, while others don't? Maybe that the trick is not to look at them one at a time, or at groups that are entangled?

Even on a macroscopic scale, if you interact with thousands of objects, the result will be dramatically different than if you interact with only one. Forces get distributes and transmitted, and such. You couldn't predict the behaviour of one object in the group if your action affects thousands of objects in the same system. You would have to interact with just that object. You could make general predictions in some cases, in others, none at all.
Title: What is Time? If there was no light would Time cease to be?
Post by: yor_on on 05/02/2011 20:45:38
Maybe :)

It's just thought experiments.

I look at it as 'relations' defining what we see. Those can be without us interacting, or with us interacting, setting up a specific experiment a certain way for example. To me the universe seems all to be relations, with some few exceptions, like gravity.
==

When it come to 'groups' I think of light. Imagine a very closeknitted web or matrix. Then shine a flashlight through it. Let someone stand on the other side in a dark room looking at the wall where that light gets represented. What do he see? Dots? move the matrix, will the dots move too?
==

Not a good one that one, the dots won't be seen to move, just 'flicker', but it's still what I'm wondering about :) what is light?
==

And yes, statistics isn't about defining a individual choice, it's about 'averages' and to look at those you will need chaos theory too.
==

Another thing, the inflationary theory, and that SpaceTime's expansion. Ever wondered how that is possible? I'm sure you have, just as I :) Why I take that one up? Well, it has a relation to how i see light, and distance too in fact. If it is right, as we're doing a lot of assumptions, let's assume that is is :) what does it tell us about 'particles'? That depends on how you define them in fact. You might want a 'particle universe' with all particles having a independent existence. Or you look at it like me, thinking that a particle seems to be defined by its relations. If you do you will still have 'matter' but on the quantum level you will have 'forces' if you like, interacting.

And now we come to the expansion, what does it do? It creates new distances, doesn't it? If it can do that then we know that one property of what we call space contain 'distance'. Can we measure it? Sure we can, as a distance, nothing else. So why doesn't particles 'expand'? The explanation as I understands it is 'gravity', gravity seems to be able to negate it. But Gravity is no 'force', is it? It's a property of SpaceTime, but no force. So why would the property differ inside a galaxy, as compared to the outside of it, or is it the 'space' differing? How about inertia? Does inertia gets stronger at a course-change being close to a neutron star, and weaker in deep space? It should shouldn't it? So even though 'gravity' might be 'everywhere' it will differ locally, and inside the galaxies it is 'stronger' than outside.

So, how do 'expansion' do it? does it pick out four dimensions and knit them together in each point of space-expansion. Or does the points come as 'whole' objects, containing distance and time directly they gets 'noticed'? That's how I see it, they have too. It's by far the simpler explanation. And where do those new points 'come from'? Nowhere I think, at least as we can see it. But this 'nowhere' is that then everywhere? It has to be, hasn't it? Just as 'gravity', and 'light' if my idea is right.

Why I 'drag it up' is primary to point out that we already accept the idea of 'stuff' coming from 'nowhere'. Another description of this light is 'virtual particles', and that weird idea of a 'quantum foam' existing. That foam isn't only in Space, it has to be in everything that exist, you and me too, but expressing itself differently depending on the relations defining it. 'Distance' seems to be constructed in the creation of new space, 'virtual particles' on the other hand seems not to bother with creating 'distance'. Speed as I said before is a matter of 'distance and time' combined. It leaves us with a weird SpaceTime, infused by all sorts of 'properties' from 'expansion' to 'virtual particles' existing 'everywhere' but expressing itself according to 'rules' we don't really know yet.

So that's what I suspect 'light' is too. Something 'existing' everywhere, but only expressing itself following strict guidelines. And that's a really, really, weird idea. But I might be all wrong in it, probably are :)
==

So what has it to do with 'time'? Assume that I'm right, then time is a 'property' too, it will be a 'constant' inside your own frame, 'room time geometry' as I like to call it :) and in fact regulate most everything you ever meet. It's an expression of change coming from the way we get modeled by the 'forces/relations' defining our macroscopic reality. That light have 'c' as a limit has nothing to do with speed. Speed is just what we see it as, but 'light' just interacts, limited by what relations it meet. And I would like the background to be, I don't really know? Gravity? Light? 'Energy'? Pick your choice. It's like we have all those 'rules', knitted together macroscopically they present us with Einsteins SpaceTime, on a quantum level they present us with 'particles' and 'forces'. Under that, and under Plank size there is something else. One of the things why I like Gravity is because it seems the 'smoothest' thing there are. You can't 'pick it apart', light has its quanta, and particles takes a 'place', even space has its foam. But 'Gravity', that's just a property, isn't it?

And time and 'c' must have a lot in common.
==

Think of it like 'levels'. From under a quantum level there is no 'space' there is no 'matter'. There is possibly a 'foam' or 'relations' existing. Their interactions have a relation to the QM-level which will have a relation to the macroscopic level. At the foam-level distance can't be. That what makes entanglements possible in QM. And distance seems to be a property of a SpaceTime over Planck-scale.
 
Title: What is Time? If there was no light would Time cease to be?
Post by: briligg on 06/02/2011 02:29:11
I think i need to read the Barbour piece on fqxi. I think i sort of follow what you are getting at, but i need more background. Anyhow, the idea of trying to model reality not in terms of points and particles, but systems, relations, and interactions, appeals to me. And as many people here, this 'c' thing definitely strikes me as a very suspicious character, probably up to all kinds of things.
Title: What is Time? If there was no light would Time cease to be?
Post by: yor_on on 06/02/2011 02:38:57
Yep, I quite agree. I mean I get a real headache reading myself, sometimes it makes no sense at all :) But I still find this 'c' weird. And that light, a shady character indeed :)

But it makes still sense, in a weird way :)
Title: What is Time? If there was no light would Time cease to be?
Post by: simplified on 06/02/2011 16:33:22
Try another example Sim, I'm not sure what you mean by 'quantity of motion'. Are you imagining the apples containing 'frozen motion'?

So if motion is 'time' and matter is 'motion' in some other state, what is gravity?
Gravity is oppressor of motion.Gravity oppresses quantity of motion(time) and quality of motion(speed). [:P]
Title: What is Time? If there was no light would Time cease to be?
Post by: yor_on on 06/02/2011 17:00:05
So gravity would retard motion, that's one way to describe it Sim. And if we then define motion as 'times arrow', yeah maybe. Maybe we could play with 'c' for that one :) letting light be what is the trend setter for all 'motion'. And then turn it around and define a state of rest as infinite gravity, that as I think it is :). And that as all matter 'free falls' when 'accelerated' by gravity, without expending energy, if you look at a photon it actually 'gains energy' in its 'acceleration' as observed from a 'inertial observer' on Earth.

But motion in itself? Is that an expression of energy? If you have a object 'free falling' following a geodesic, does that object expend any energy? Will it be time dilated if close to 'c' relative its origin (Earth). Then it will answer your description Sim, but you will have a problem in that all uniform motion, from inside that black box will be the exact same. Meaning that you won't be able to differ the speeds and so they become equivalent. And if time dilation is defined by motion you will need to answer how the equivalence of all uniform motion can be solved. On the other hand, assuming that time dilation exists depending on 'speed', not only acceleration, any such description will have to answer the same question?

Got an idea how to answer that one?
Title: What is Time? If there was no light would Time cease to be?
Post by: yor_on on 06/02/2011 17:25:11
One possible answer is that a time dilation always is defined as between 'frames'. that it exist has not with this to do as the twin traveling at no time will find himself experiencing time 'differently', so just as all motion becomes undefinable in uniform motion so will a time dilation be. But then again, where is the equivalence if we have a different 'aging' of that twin, depending on his uniform speed?

You could also argue that, as seen from his own frame, the twin experience no 'time dilation' as time is 'as always' to him. But, what have then experienced this 'time dilation'? The rest of the universe? Only those parts of a universe 'slower' relative our twin, with those at rest relative it the same 'age', and those faster than our twin 'slower aging' than the traveling twin?

Title: What is Time? If there was no light would Time cease to be?
Post by: simplified on 06/02/2011 18:16:21
Difference of oppression of motion turns into energy. [;)]
Title: What is Time? If there was no light would Time cease to be?
Post by: yor_on on 06/02/2011 18:53:46
You better explain that one in ah, 'plain English' Sim. I'm not sure how you think there?
Title: What is Time? If there was no light would Time cease to be?
Post by: yor_on on 06/02/2011 19:41:29
What if we look at it, after all, as 'density'? Like, if not 'space', but the whole 'SpaceTime' consisted of an unseen 'density' of its own, unseen by us. This 'density' would then be 'thicker' where invariant mass was and then 'thin out' in 'Space'. Then we could use 'motion' as a way to 'contract' the density, creating a friction that will slow our room time relative the rest of the universe.

Think of it as your 'time' having a even pace, marked out every meter. then as you meet the density the 'negative time expression' will space that time-meter out as seen from another frame of reference, a little like a 'gravity wave' is expected to do. In fact a 'gravity wave' should contract the 'invariant mass' and so increase the 'time dilation' slowing, whatever it wanders through, down relative any other frame.
==

Although I'm not sure of how it would do it? It's (the gravity waves) 'edge' should be able too anyway.
==

Or you can turn it around and say that the density is inverted. Define it as 'thicker', if you like, when out of the reach of 'invariant mass'.
==

Using the Higgs bosons is one possibility, although then I would like the space ships atoms to 'jiggle' as the overall 'mass' effectively should become greater the faster you move. But they don't..
==

Okay, the thing I keep coming back to is. What is a 'distance'. I can't seem to let that one go. Maybe I should change it though? Too 'what is a position' instead. In relativity a position is something defined by what relations you use. Maybe there are 'unchanging positions' when considering inertial frames? But as soon as you introduce an acceleration that balance get skewed. When it comes to uniformly moving it is a little different though. So, does there exist any single positional way to define objects in SpaceTime?

I doubt it.
Title: What is Time? If there was no light would Time cease to be?
Post by: simplified on 07/02/2011 17:32:29
You better explain that one in ah, 'plain English' Sim. I'm not sure how you think there?
If an object travels to mass then motion of this object goes in trap of gravity. Gravity likes to execute the own destination, therefore entices motion by additional energy. [:)]
Title: What is Time? If there was no light would Time cease to be?
Post by: yor_on on 07/02/2011 17:54:46
If an object travels to mass then motion of this object goes in trap of gravity. Gravity likes to execute the own destination, therefore entices motion by additional energy. [:)]

This all goes back to how to define what we talk about I think :)

Here's my definitions.

1. Gravity is a property, not a 'force'. That means I can look at 'space' as if it was a 'topology' having dips and heights and even 'swirls' and stuff, a little like a 'fluid'.

2. Energy is also a property, having no existence until a 'interaction' is being made. The relations of this interaction will define the energy, and photons red/blue-shift is the perfect example of such a relation.

3. Relative mass, momentum, potential 'energy', are also properties, only expressing themselves in a interaction. That means that although you may count on them, having them 'defined', they do not exist. And to prove that I use 'atoms jiggling' in that spaceship.

Because if they were true as anything more than a relation to something else, in this case 'speed' then they should get 'stored' in that spaceship.
==

To change it you will have to move outside what you measure, and then define the relations as being true on 'another plane' not reachable for mere humans. Which then may be true, I'm inclined to think it is, but to me there should still be something expressing those properties in 'real time' from the 'frame' that experience it e.g 'a speed'. Or we have a universe in where only 'interactions' count, and the rest becomes our framework for explaining and wanting to make sense of how those 'interactions' occur. And that we do inside a causality chain we either call 'times arrow' or 'entropy'. In fact it is our acceptance of that causality chain that makes us expect that there have to be something making sense in this universe, isn't it?

So if some spaceship speeds away in space relative us on earth. When does it get a 'gravity'? Only in the acceleration it seems to me. Does the ship 'store' any additional energy as measurable by those on that ship? Not as I know, no additional jiggling perceivable. Does it do so from any other frame of reference? Nope, no signs of 'glowing' radiation, as I now off?
==

In fact, that is wrong, from a frame of reference meeting that ship you will see light from it as being compressed/more energetic. But it is as true, that you from behind that ship will perceive it as 'stretched'/less energetic. So?
==

To see what I mean, define it as 'speed'.

Then look at it as leaving you, from the spaceships origin, Earth_1. It goes really fast doesn't it? But is the light reaching us from that ship now more 'energetic'? It isn't, but we still assign this 'energy' it shows to its 'speed. Now instead meet that ship, coming towards you on Earth_2. Wow it goes really fast, don't it, having a tremendous speed. And yeah, the light coming from it is really 'energetic' too. So, not knowing any of our physics. And only able to notice the light as  this 'speeds' expression. What would you on Earth_1 define speed as? And you on Earth_2.

We know that the energy just is a relation, but they don't. So, is this energy true? Yep, it will deliver more or less work. Does it have a relation to 'speed', yes it does. But to see if there really is more or less energy stored you need to look at the ship itself, in its own frame of reference. and there I don't expect any more jiggling of the ships 'atoms'.
Title: What is Time? If there was no light would Time cease to be?
Post by: simplified on 07/02/2011 18:32:11
If an object travels to mass then motion of this object goes in trap of gravity. Gravity likes to execute the own destination, therefore entices motion by additional energy. [:)]

This all goes back to how to define what we talk about I think :)

Here's my definitions.

1. Gravity is a property, not a 'force'. That means I can look at 'space' as if it was a 'topology' having dips and heights and even 'swirls' and stuff, a little like a 'fluid'.

2. Energy is also a property, having no existence until a 'interaction' is being made. The relations of this interaction will define the energy, and photons red/blue-shift is the perfect example of such a relation.

3. Relative mass, momentum, potential 'energy', are also properties, only expressing themselves in a interaction. That means that although you may count on them, having them 'defined', they do not exist. And to prove that I use 'atoms jiggling' in that spaceship.

Because if they were true as anything more than a relation to something else, in this case 'speed' then they should get 'stored' in that spaceship.
==

To change it you will have to move outside what you measure, and then define the relations as being true on 'another plane' not reachable for mere humans. Which then may be true, I'm inclined to think it is, but to me there should still be something expressing those properties in 'real time' from the 'frame' that experience it e.g 'a speed'. Or we have a universe in where only 'interactions' count, and the rest becomes our framework for explaining and wanting to make sense of how those 'interactions' occur. And that we do inside a causality chain we either call 'times arrow' or 'entropy'. In fact it is our acceptance of that causality chain that makes us expect that there have to be something making sense in this universe, isn't it?

So if some spaceship speeds away in space relative us on earth. When does it get a 'gravity'? Only in the acceleration it seems to me. Does the ship 'store' any additional energy as measurable by those on that ship? Not as I know, no additional jiggling perceivable. Does it do so from any other frame of reference? Nope, no signs of 'glowing' radiation, as I now off?
==

In fact, that is wrong, from a frame of reference meeting that ship you will see light from it as being compressed/more energetic. But it is as true, that you from behind that ship will perceive it as 'stretched'/less energetic. So?
I do not understand your english without math too. [:P]
Title: What is Time? If there was no light would Time cease to be?
Post by: yor_on on 07/02/2011 18:38:07
Well then, use your math. Define all your variables clearly, so that we can understand them, then prove your idea.
==

(Sim, that doesn't mean that you can assign 'x' a property of 'tachyons', or 'gravity' for example, without defining how you come to this conclusion. So you need to build every statement clearly and with proofs.)
Title: What is Time? If there was no light would Time cease to be?
Post by: simplified on 07/02/2011 19:01:38
Well then, use your math. Define all your variables clearly, so that we can understand them, then prove your idea.
==

(Sim, that doesn't mean that you can assign 'x' a property of 'tachyons', or 'gravity' for example, without defining how you come to this conclusion. So you need to build every statement clearly and with proofs.)
Axioms do not need proofs.
Title: What is Time? If there was no light would Time cease to be?
Post by: yor_on on 07/02/2011 19:41:35
Yes, gravity is a axiom. But to use it without coupling it to acceleration or invariant mass will need a definition, at least for me :) Those are the two thing I know defining 'gravity'? Uniform motion do not define any gravity to itself, neither any 'inertia' as I know that is. And to assume gravity to exist in a uniform motion it needs to be noticed as is. But is is not there :)
==

If you do like I did and assume that there would be a infinite 'gravity' permeating 'SpaceTime', with matter and speed regulating its 'expression', you might get away with it though :)  But you would need to redefine a lot of other things too to make it work I think?

But if you think you can make sense of it, do it, it's a view I like actually :)
==

There is another possibility, equating 'energy' with speed though. That one we can lend from the compressed spring. After all, the spring won't glow even though it have a added 'real mass' from a compression. And that should be true how much you ever try to compress it, as long as you let the kinetic energy cool of. But it builds on assuming that a Lorentz contraction being real. If I do so, could I assume that the Lorentz contraction applied on a moving frame of reference, as defined from another frame, equals a compression? I don't know? We meet the same problem again there, all uniform frames of reference being equal and that you're the one defining who moves against who? So it would crave a way for the universe to define who is moving against who. But if I could the mass would change, wouldn't it? It must be noticeable as an added weight in a acceleration, at least? And that would then be an added 'gravity'. In a uniform motion weight lose its meaning, but the mass of the spaceship might still be higher due to the Lorentz contraction, if real?

Maybe?
=

The more I think of it the weirder it becomes huh :)
But if we have a state of singularity in 'SpaceTime' being Black Holes?
And for this we better define a singularity as non-communicative, with a one way reception.
Because then it will fit my idea of what gravity is.

Hawking radiation may come as a result of a 'interaction' but that interaction is not a communicative event. The event horizon is a real divider between a SpaceTime and a singularity, and the in-falling light can only point in one direction, into it, not from it. If we could transfer information by entanglements I would be wrong there, but so far we can't. If we can inject energy in a entanglement as a proposition I've seen recently, then the surviving pair particle inside our universe should have an added 'energy'. And as one idea is that the negation of a anti particle inside the event horizon should leave a surplus of 'energy', creating a 'uneven balance', then that might correct that 'balance' evening out the 'score'?

Or alternatively prove that a 'injection' of energy in a entangled particle by your measurement won't work. Pick your choice there, myself I would be very surprised if a added 'injection' of energy in your measurement would work, as that seems a communication to me. On the other hand plants seem to use entangled 'energy', but there I will assume it not to be the same injected 'energy/momentum' whatever, but just a efficient way of distributing what 'energy' a entanglement already has, instead to me proving the impression I already have of 'least energy expended' being a rule. If we assume that light and the arrow of time have a relation, as I see it now, radiation to my eyes being no more than time, both expressing their self in 'changes', both obeying 'rules/constants', just like that Feigenbaum constant 'rule', impossible to backtrack? Maybe? Anyway, I'm sure I can find more arguments, but for this it's enough. Then we have two remarkable things I think. We have 'gravity' and 'compression'. Why do a compression store mass? And what happens to a 'mass' once its compressed to a point particle? Does it 'radiate', or does it just disappear?
Title: What is Time? If there was no light would Time cease to be?
Post by: yor_on on 09/02/2011 20:57:46
Rereading you Sim. Are you using Newtons definitions? Quantity of matter as mass, and Quantity of motion as the product of velocity and mass?

Then quantity of matter as mass makes sense to me, but I'm not sure how you think of quantity of motion as the product of velocity and mass? Do you include for example momentum and/or relative mass in that? Because that's the mechanical laws we use since Newton? Well, momentum may be new but 'relative mass' is a concept that have followed us a long time, in one shape or another.

So how do I look at 'gravity'? Simple, to me it's what's not explained. How did Einstein reach the rest of his conclusions?

"Einstein bases SRT on two fundamental principles: the principle of relativity and the principle of the constancy of the velocity of light. The principle of relativity originated in Galilean-Newtonian mechanics: Any frame of reference in which Newton's law of inertia holds (for some period of time) is now called an inertial frame of reference. From the laws of mechanics it follows that, if one such inertial frame exists, then an infinity of them must: All frames of reference (and only such frames) moving with constant velocity with respect to a given inertial frame are also inertial frames. All mechanical experiments and observations proved to be in accord with the (mechanical) principle of relativity: the laws of mechanics take the same form in any of these inertial frames.

The principle of relativity, as Einstein stated it in 1905, asserts that all the laws of physics take the same form in any inertial frame-in particular, the laws of electricity, magnetism, and optics in addition to those of mechanics."

So we seem to agree there? We both define your own 'room time geometry' as uniquely 'the same'. The funny thing about that is that the revelation of all frames outside your own being 'different' blinds people. The only way we use it is in defining it as 'entropy' or 'the arrow of time', but to my eyes it's a 'constant' of sorts.

Gravity then?

"he assumed that a person cannot distinguish whether he is in a gravitational field or under constant acceleration, i.e. these two things are equivalent. This assumption provided a link between gravity and mass, or gravity and energy. He concluded then that mass and energy must affect the gravitational field. Secondly, he stated that the gravitational field is not actually a force as Newton has described, but instead a curvature in space. To put it in simple words, the bodies are affected by gravity not because of a force directly exerted on them but because space is curved and therefore they have to follow space's grid. The presence of mass or energy does not affect the bodies directly; it affects the space first, and then the bodies move in this curved space. Earth always moves in a straight line (not in the Euclidian sense though). The presence of sun curves space, and therefore curves this straight line and forces earth to appear to be moving in an ellipse... Imagine being in a sea (or pool) using your finger to create ripples on the water. The presence of your finger in the water creates these waves and alters the geometry of water, it's not flat anymore. If you look through the rippled water you'll see the bottom distorted. Well, the same things happens to 3D space if you try to move your finger like that on the air! Your finger has mass and it actually can create ripples in space... The only reason you cannot see them as in the water is that the mass of your finger is so small that the ripples it creates are of the tiniest magnitude; there are so small that do not affect anything considerably."

Now, this analogue made by the author is rather cool. The problem with it is that it gives SpaceTime a 'density'. Do we accept that? I'm not sure I do. What we can say is that any description we use built on previous experiences probably are wrong when describing 'SpaceTime. It's a very large step between looking at SpaceTime from the viewpoint of celestial mechanics versus as compared to relativity, and to drag over concepts that works so well on our earthly scale to explain the universe?
Title: What is Time? If there was no light would Time cease to be?
Post by: yor_on on 11/02/2011 03:54:57
This is some questions I have. First, why do I keep on about 'relations'? Look at the universe, you have space, all empty. No matter what we expect on the quantum level there isn't a 'thing' in empty space macroscopically. It's a dark lonely emptiness with some small islands of matter. Do particles exist on their own in space? Not readily, and not alone, as I know it? And I'm not talking bosons here, but particles of restmass. Now look at it from the viewpoint of 'relations'. Then those islands sparkle with relations, they are the most concentrated places of relations existing. How do matter 'grow', and, why can it 'grow'? How can we consist of a 'insurmountable' amount of energy, each one of us, and still be able to function, transform other types of matter into ourselves and use it?

It has to be another way of looking at it. And doing it as relations seems to work for me. Think of the question we had recently here, what will happen if we could 'time travel' to the 'past'. Would we land where we started, or in space as the earth and the universe are moving in time. I said that it all should adapt, all dimensions together, as it should be one undivided 'SpaceTime'. The implications of that are staggering to me suddenly. Because I also think of the universe as a place that have this rule of 'least energy expended'.

Can you see what that way of looking at it implies? That the whole universe would adapt to your 'time travel'. Now, if that way of looking is right then a Lorentz contraction suddenly have a much larger possibility of being 'real', instead of just some geometric illusion. Because when we see it we are in fact 'time traveling', even if in the other direction temporary, in time that is.

So can we test that idea? I think Einsteins ideas already have done that. And he seems to be right too. You have a unique arrow of time, in that arrow, no matter what you do, times arrow will be the same. But the 'universe at large' will adapt to certain properties, invariant mass and motion. And that is not really what I expected if it is correct? What happened with the idea of 'least energy expended' here, if so? Well, when you 'time travel' do you really influence all other objects around you. Not if you stand where they are, so the rule may still work?

So what is wrong here? There is a dichotomy in that I might assume that when you use motion or invariant mass for your time travel you do change the universe at large and the fact that when meeting up with your twin you will find that he felt no such 'influences' acting upon him just because you traveled near light. So if I'm correct in my thinking and also assume that you really change the universe, aka, 'Lorentz contraction' and 'time dilation' both are 'real. Where does it place the 'interactions'?
Title: What is Time? If there was no light would Time cease to be?
Post by: Bill S on 11/02/2011 20:53:33
Quote
Axioms do not need proofs

Surely, axioms become axioms only because their proof has already been established, at least to the satisfaction of those who accept their axiomatic status.
Title: What is Time? If there was no light would Time cease to be?
Post by: yor_on on 12/02/2011 03:37:10
Yeah, a axiom is like one and one makes two.

Weird stuff like that :) But it is also a axiom that all uniform motion shows an absence of gravity, and inertia of course. Assigning a gravity to uniform motion would need a new definition, of either uniform motion, or gravity and inertia. That it follows a 'geodesic' induced by gravity, does not state that you can feel that gravity working on you in a 'black box' scenario. And that's also what make all uniform motion 'equivalent' to my eyes, and so all 'uniform motions/speeds' following that geodesic.

But yes, gravity must be there, even though unmeasurable in that uniform motion as we can observe light 'bending' around invariant mass, like the sun. But then we're discussing 'frames of reference' again. In the black box there is no way I know of proving it, ignoring tidal forces now, like frame dragging.
==

Reasoning along those lines you will find it natural to define it as being 'at rest' relative gravity. And if you find that natural you will need to look at where our definitions of gravity 'breaks down'. That should be a Black Hole as I see it, all light/time cones pointing one way. and if that is the 'state' where you will find matter disappear then, maybe, you might want to define that as the ultimate state of rest :). Quite questionable as you as easily then can assume all 'speeds' following a geodesic as being 'at rest', but it's still the state in where we expect matter to ??
Title: What is Time? If there was no light would Time cease to be?
Post by: simplified on 12/02/2011 15:01:29
Quote
Axioms do not need proofs

Surely, axioms become axioms only because their proof has already been established, at least to the satisfaction of those who accept their axiomatic status.
My definition of gravity is imperfect. But you have no definition of this at all. And at all you can only jump over sequence. [:P]
Title: What is Time? If there was no light would Time cease to be?
Post by: hakaya on 15/02/2011 09:11:44
Time is in three parts in the future that never arrived and are available only in my dreams, man, 'which currently exists only to separate the small second and the last to go on forever in the memory man', but not always. return unless an error and we do tend to repeat.
Title: Re: What is Time? If there was no light would Time cease to be?
Post by: Paul25 on 19/03/2020 15:15:34
Time is more related to movement
Title: Re: What is Time? If there was no light would Time cease to be?
Post by: talanum1 on 03/06/2020 10:28:54
Time is as defined in the attachment.

No, time would not cease to be if there was no light. The definition is clear about this. All that is required are particles with no charges on a circle in a Riemann Sphere.
Title: Re: What is Time? If there was no light would Time cease to be?
Post by: yor_on on 07/06/2020 16:22:43
I am trying to understand exactly what time is. When I think about time I immediately link it to light. Astronomers often tell us that they are able to take pictures of the universe billions of years in the past and that this is due to the amount of time it takes for light to travel. As I understand it, if I am looking up into the night sky what I am seeing is actually the past, in the case of the sun light I am seeing approximately 8 minutes into the past where as looking at the stars I could be looking many millions of years into the past. This is where I start to get boxed in with my own limited knowledge and perhaps some of you more knowledgeable people could enlighten me. If there was no light would time cease to exist altogether? I appreciate anyone taking the time to help me try and understand this.

Graham

You can connect the passing of time to 'c', lights speed in a vacuum. Both are local 'constants'. That doesn't tell you what time is though, just that there is a equivalence between those two. We have no explanation for why 'c' is 'c' either, just experimental proofs. A vacuum is devoid of light, unless you have matter inside it, although that may be arguable. You need to place that sun and that matter inside this vacuum to get it to interact, creating heat. So no, I think time is more than just a interaction, it's a local constant, experienced through your wristwatch.
=

syntax

I shouldn't split it into a sun and matter, the first sentence is correct, the second is wrong. A sun doesn't differ from f.ex earth. Both are matter in different states.
=

And damn, do I hate those yellow suns appearing in my old posts. Due to a failed update.
Title: Re: What is Time? If there was no light would Time cease to be?
Post by: Bill S on 09/06/2020 14:06:05
Possibly, what we need is a definition of time, in the absence of anything/everything else.
Title: Re: What is Time? If there was no light would Time cease to be?
Post by: talanum1 on 09/06/2020 16:25:11
Possibly, what we need is a definition of time, in the absence of anything/everything else.

Look at my definition at post #247.
Title: Re: What is Time? If there was no light would Time cease to be?
Post by: Bill S on 10/06/2020 20:26:30
Quote from: talanum1
Time is as defined in the attachment.
Presumably you refer to the definition in the attachment.  Valid as this might be as a mathematical definition, it does not provide a physical example of time, "in the absence of anything/everything else", that I could see.  Of course, the cursory way in which I am obliged to look at these things, these days, could mean I missed it. 
Title: Re: What is Time? If there was no light would Time cease to be?
Post by: alancalverd on 10/06/2020 22:49:09
How about going back to the obvious?

Time is what separates sequential events. If you can discern a "before" (a piece of paper) and an "after" (a pile of ash) then these observations were separated by an amount of time.

And whilst we are defining the deeper mysteries of the universe, gravity is the force between massive objects that depends only on their mass and separation.
Title: Re: What is Time? If there was no light would Time cease to be?
Post by: Bill S on 11/06/2020 12:20:33
Quote from: Alan
Time is what separates sequential events. If you can discern a "before" (a piece of paper) and an "after" (a pile of ash) then these observations were separated by an amount of time.

Good definition of time in the presence of other things, but still no definition of time in the absence of everything else.  I certainly can't find such a definition, but hope springs eternal, and all that.
Title: Re: What is Time? If there was no light would Time cease to be?
Post by: PmbPhy on 11/06/2020 12:26:54
Apples in Geezer's garden  are their quantity, but motion is not quantity of motion. Because the quantity of apples can decrease or increase.  Quantity of motion only increases.
 Here I am wrong,because quantity of  created apples only increase.
Quantity of motion is defined to be momentum in physics. In physics time]/i] is undefined as being intuitively known to all to all observers.
Title: Re: What is Time? If there was no light would Time cease to be?
Post by: Bill S on 11/06/2020 12:50:28
Quote
time]/i]

For the benefit of the maladroit; is that time/distance?
Title: Re: What is Time? If there was no light would Time cease to be?
Post by: alancalverd on 11/06/2020 13:16:34
Quote from: Alan
Time is what separates sequential events. If you can discern a "before" (a piece of paper) and an "after" (a pile of ash) then these observations were separated by an amount of time.

Good definition of time in the presence of other things, but still no definition of time in the absence of everything else.  I certainly can't find such a definition, but hope springs eternal, and all that.
In the absence of anything else there can be no change, so time is meaningless. To use the philosopher's default object, if there were no ruminant quadrupeds, "cow" would be meaningless. If nobody spoke English, so would this entire post!

My neolithic ancestor Ug showed Og the spanner he had invented "For tightening bolts". "Cool too, bro! What's a bolt?"
Title: Re: What is Time? If there was no light would Time cease to be?
Post by: Bill S on 13/06/2020 18:12:35
That’s fine. Would a fair paraphrase be: “In the absence of anything other than time, the concept of time would be meaningless”?
Title: Re: What is Time? If there was no light would Time cease to be?
Post by: talanum1 on 15/06/2020 16:46:11
“In the absence of anything other than time, the concept of time would be meaningless”?

This translate as:  "The concept of time would be meaningless, in the presence of only time." or: "The concept of time would be meaningless, in the presence of only meaningless time."
Title: Re: What is Time? If there was no light would Time cease to be?
Post by: Bill S on 16/06/2020 19:00:56
That sounds like a very convoluted "yes". :)
Title: Re: What is Time? If there was no light would Time cease to be?
Post by: PmbPhy on 18/06/2020 06:09:17
Time cannot be defined. It's a primary concept like space. We can only then provide operational definitions for space and time intervals: measure time intervals with a clock and space intervals with a rod.
Title: Re: What is Time? If there was no light would Time cease to be?
Post by: talanum1 on 18/06/2020 12:16:30
It can be defined. See post 247 under this topic.
Title: Re: What is Time? If there was no light would Time cease to be?
Post by: Bill S on 18/06/2020 12:45:01
It is possible to find a range of “definitions” of time, but each is relevant to the particular way in which the term is being used.  I agree with Pete, in that, if you consider time as a fundamental entity, that has an “existence” of its own, it probably cannot be defined.
 
One of the most concise definitions I have met is: “Time is the dimension on which the evolution of state of a system is allowed to occur”.  However, it is the evolution between points that is being measured.  We define intervals, and call these intervals time. Without the points, there would be no intervals. Without the intervals, there would be no time.
Title: Re: What is Time? If there was no light would Time cease to be?
Post by: PmbPhy on 18/06/2020 14:16:33
It can be defined. See post 247 under this topic.
That post is wrong.
Title: Re: What is Time? If there was no light would Time cease to be?
Post by: Colin2B on 18/06/2020 14:35:49
However, it is the evolution between points that is being measured.  We define intervals, and call these intervals time. Without the points, there would be no intervals. Without the intervals, there would be no time.
I don’t follow your reasoning Bill.
Let’s say we place some posts in the ground at an arbitrary distance apart and define that distance as 1unit. If we then remove the posts would you say that without those intervals there is no distance?
From my point of view when you say “We define intervals, and call these intervals time” I would say that we call those intervals measures of time.
Title: Re: What is Time? If there was no light would Time cease to be?
Post by: Bill S on 18/06/2020 14:44:36
Quote from: Yor_on #248
  A vacuum is devoid of light, unless you have matter inside it, although that may be arguable.

Arguable, indeed! If you put matter in it, is it still a vacuum?  We talk of having something in a vacuum, but is that like saying “there is nothing in here”, when we mean that what we are looking for is not there?  Is it a "narrow" usage of the term "vacuum"?

Then again; if putting something into a vacuum causes it no longer to be a vacuum, then there is no such thing as a vacuum, anywhere in the Universe.
Title: Re: What is Time? If there was no light would Time cease to be?
Post by: talanum1 on 18/06/2020 17:07:16
That post is wrong.

Is defining time using particles with fluctuating space as input wrong?
Title: Re: What is Time? If there was no light would Time cease to be?
Post by: PmbPhy on 18/06/2020 18:11:17
That post is wrong.

Is defining time using particles with fluctuating space as input wrong?

Yes.
Title: Re: What is Time? If there was no light would Time cease to be?
Post by: talanum1 on 18/06/2020 18:54:47
That post is wrong.

Is defining time using particles with fluctuating space as input wrong?

Yes.

And your reason is ... ?
Title: Re: What is Time? If there was no light would Time cease to be?
Post by: PmbPhy on 19/06/2020 03:40:06
That post is wrong.

Is defining time using particles with fluctuating space as input wrong?

Yes.

And your reason is ... ?
Same as held by any physicist, i.e. time is defined as what a clock reads. This is not a good definition because when you get into the details of what a clock is you end up with a circular definition.

See - http://www.exactlywhatistime.com/physics-of-time/

Your definition depends on the concept of fluctuation which in term relies on what fluctuation means. I was hoping to avoid getting into a debate about this since they never end. I'll just leave it here and point you to SR textbooks that you trust.
Title: Re: What is Time? If there was no light would Time cease to be?
Post by: talanum1 on 19/06/2020 10:16:10
I can leave out mention of fluctuations and simply state that flow of time depends on particles encountering points of space.

The new definition of time is attached.


* Physics from Axioms clean.pdf (126.82 kB - downloaded 547 times)
Title: Re: What is Time? If there was no light would Time cease to be?
Post by: Bill S on 19/06/2020 16:38:09
I'm certainly not qualified to comment on the maths in this paper, but just from reading the abstract, and looking a little beyond, I have to wonder about the possible translation from mathematical "reality" to any physical demonstration.  I look forward to further developments.
Title: Re: What is Time? If there was no light would Time cease to be?
Post by: PmbPhy on 20/06/2020 14:50:24
I'm certainly not qualified to comment on the maths in this paper, but just from reading the abstract, and looking a little beyond, I have to wonder about the possible translation from mathematical "reality" to any physical demonstration.  I look forward to further developments.
You will never see me define something in an large article. Defining something like time, if its actually possible. takes only a short paragraph.

Take a look at - http://www.exactlywhatistime.com/
----------
We can measure time extremely accurately, but it is very difficult to define and explain exactly what time actually is
----------
What taluma1 is trying to do is state what time is by providing a specific example of a clock. Measuring something does not tell  you what it is.

ps - I wasn't going to post again  since I knew they'd be ignored but in this case I felt I had to speak up. Anything else, Bill, I'll PM you.
Title: Re: What is Time? If there was no light would Time cease to be?
Post by: Colin2B on 20/06/2020 18:02:27
I can leave out mention of fluctuations and simply state that flow of time depends on particles encountering points of space.
It really doesn’t help. If the particles encounter points in space then they are moving and that requires pre-existence of time for it to occur. Your argument is circular/self dependent and does not define time.

Also, this is not the right place to discuss a new theory, so please confine it to the New Theories section

time is defined as what a clock reads. This is not a good definition because when you get into the details of what a clock is you end up with a circular definition.
As you also say Pete, it also leads to the confusion between the definition of the phenomenon and the definition of the measurement. That confusion leads to some very illogical conclusions not least of which is the title of this topic.
Title: Re: What is Time? If there was no light would Time cease to be?
Post by: talanum1 on 20/06/2020 18:29:40
It really doesn’t help. If the particles encounter points in space then they are moving and that requires pre-existence of time for it to occur. Your argument is circular/self dependent and does not define time.

My definition only implicitly requires the concept of time. It is my contention that implicit reference to the object being defined is allowable.
Title: Re: What is Time? If there was no light would Time cease to be?
Post by: PmbPhy on 20/06/2020 18:37:26
As you also say Pete, it also leads to the confusion between the definition of the phenomenon and the definition of the measurement. That confusion leads to some very illogical conclusions not least of which is the title of this topic.
Here's a nice coincidence for you. I just received I book I ordered from Amazon called Graduate Methods of Classical Mechanics y V.I Arnold, Springer, 1989. From page 2

The text refers to the following as one of three elements of the Galilean structure
----------
Time - A linear mapping R^4 --> R from the vector space of parallel displacements of the real "time axis."
----------
 I don't consider this to be a definition of time though since that element does not tell us what R^4 is or what the "time axis" is.

Newton took time and space to be well-known concepts not in need of definitions.
Title: Re: What is Time? If there was no light would Time cease to be?
Post by: PmbPhy on 20/06/2020 18:41:48
My definition only implicitly requires the concept of time. It is my contention that implicit reference to the object being defined is allowable.
Allowable by whom? What on God's earth does it mean to be allowable in defining things in physics? All you've done is to state a warm fuzzy feeling of what you think time is measured by. That's identical to saying that time is what measured by a clock. I've already mentioned that on an occasion or two.
Title: Re: What is Time? If there was no light would Time cease to be?
Post by: Bill S on 20/06/2020 19:10:27
Thanks for the link, Pete. Looks like a lot of good stuff there. I’ve added to my Speed Dial, so any time I have a free moment I can dip in it.

Thanks, also, for offering to be there if when I run into problems.
Title: Re: What is Time? If there was no light would Time cease to be?
Post by: PmbPhy on 20/06/2020 19:56:46
Thanks, also, for offering to be there if when I run into problems.
It's a pleasure. I'm here for you anytime my friend. :)
Title: Re: What is Time? If there was no light would Time cease to be?
Post by: Bill S on 20/06/2020 20:35:27
Quote from: Talanum 1
My definition only implicitly requires the concept of time.

I’m fascinated by the idea that the concept of time is “only implicitly” required.  Does that mean that your definition does not require a time concept?  If so, how does it work if any movement/change is involved?
Title: Re: What is Time? If there was no light would Time cease to be?
Post by: PmbPhy on 20/06/2020 21:38:16
Quote from: Talanum 1
My definition only implicitly requires the concept of time.

I’m fascinated by the idea that the concept of time is “only implicitly” required.  Does that mean that your definition does not require a time concept?  If so, how does it work if any movement/change is involved?

That's the main problem with his and others definition. It requires the concept of motion/change which are defined in terms of time. Thus it's a circular definition and fails as one. I.e. time is defined in terms of motion/change which is defined in terms of time.

There's a book called The Philosophy of Space and Time by Hans Reichenbach, Dover Pub. which some might find interesting to read.
Title: Re: What is Time? If there was no light would Time cease to be?
Post by: talanum1 on 20/06/2020 22:43:58
The definition: "time is what a clock measure" is circular since a clock has to be calibrated, hence you need time to define time. With an atomic clock the definition: "x oscillations per second" is circular since oscillation requires motion.

My definition is just implicitly circular. I guess it is circular then.
Title: Re: What is Time? If there was no light would Time cease to be?
Post by: PmbPhy on 21/06/2020 05:44:45
The definition: "time is what a clock measure" is circular since a clock has to be calibrated, ...
Wrong. A clock need not e calibrated. It can merely be built and used it's primary time interval as the unit of time. Do I need to explain what a primary time interval is? Think about a pendulum clock.
Title: Re: What is Time? If there was no light would Time cease to be?
Post by: talanum1 on 21/06/2020 12:48:23
A clock's parts move, which implies time: circular.

One needs a dynamic input to define time, by basic reasoning.
Title: Re: What is Time? If there was no light would Time cease to be?
Post by: Bored chemist on 21/06/2020 13:02:26
One needs a dynamic input to define time, by basic reasoning.
That doesn't actually mean anything.
Title: Re: What is Time? If there was no light would Time cease to be?
Post by: Bill S on 21/06/2020 13:08:16
Quote from: talanum 1
My definition is just implicitly circular. I guess it is circular then.

If that is seriously intended as an answer to the questions in #279, there is probably little point in asking any more questions. 
Title: Re: What is Time? If there was no light would Time cease to be?
Post by: Bill S on 21/06/2020 13:13:14
Quote from: Pete
Do I need to explain what a primary time interval is?

Reason v dogmatic thinking?  You could be on to a looser, there, Pete. :)
Title: Re: What is Time? If there was no light would Time cease to be?
Post by: talanum1 on 21/06/2020 14:37:44
I’m fascinated by the idea that the concept of time is “only implicitly” required.  Does that mean that your definition does not require a time concept?  If so, how does it work if any movement/change is involved?

If I use time in the definition of time the definition is circular. As I stated: a dynamical input is needed. This makes any definition of time circular.

How are you going to define time in only static terms?
Title: Re: What is Time? If there was no light would Time cease to be?
Post by: Bill S on 21/06/2020 15:03:08
Quote
How are you going to define time in only static terms?

That depends on what you mean by "static terms".  If you mean there is no motion/change in the Universe, then time has no meaning, or role in the scenario, and therefore no definition.

The question would then arise as to whether a totally static universe is even a physical possibility.
Title: Re: What is Time? If there was no light would Time cease to be?
Post by: talanum1 on 21/06/2020 16:15:58
That depends on what you mean by "static terms".

I mean: words and concepts not using motion/change.

If you mean there is no motion/change in the Universe, then time has no meaning, or role in the scenario, and therefore no definition.

So, it's impossible to define time in static terms?
Title: Re: What is Time? If there was no light would Time cease to be?
Post by: Bill S on 22/06/2020 15:26:13
Quote
So, it's impossible to define time in static terms?

Whatever may, or may not, be possible mathematically; I would be interested to see a practical example.
Title: Re: What is Time? If there was no light would Time cease to be?
Post by: Bill S on 27/06/2020 13:55:35
http://www.exactlywhatistime.com/other-aspects-of-time/eternity-and-immortality/

I’m not recommending this as a good read.  IMO it is an emporium of philosophical and religious verbosity. Even so, it includes one or two points that may be worthy of comment.  Eg:

Quote
  However, if time actually is quantized, it is likely to be at the level of Planck time (about 10-43 seconds), the smallest possible length of time according to theoretical physics, and probably forever beyond our practical measurement abilities.

If time is just the conventional measure we apply to duration between changes; one has to wonder, in what sense it could be quantized.  Appealing to the concept of Planck time does not really make any difference, as Planck time has no special significance beyond theoretical calculations.

Title: Re: What is Time? If there was no light would Time cease to be?
Post by: Just thinking on 18/06/2021 03:14:50
No, Time and light are not associated. A light year is a measurement of distance.
Title: Re: What is Time? If there was no light would Time cease to be?
Post by: talanum1 on 18/06/2021 16:35:18
I have since refined the definition so it's no longer circular. See attachment.

 [ Invalid Attachment ]
Title: Re: What is Time? If there was no light would Time cease to be?
Post by: Just thinking on 18/06/2021 17:39:42
Time is the passage of existence reality as we know it to be.
Title: Re: What is Time? If there was no light would Time cease to be?
Post by: TommyJ on 29/07/2021 15:27:04
Entropy - arrow of time.
Time exists in observations, equations, theories and laws. It is a part of apparatus that we currently have working best for the Universe understanding.
It is determined by an observer and perceived  as a human.

Think about that idea. Does light experience time? From the perspective of a photon, it's emitted, and might exist for hundreds of trillions of years, but for the photon, there's zero time elapsed between when it's emitted and when it's absorbed again.
Title: Re: What is Time? If there was no light would Time cease to be?
Post by: Eternal Student on 29/07/2021 17:43:25
Hi.

Time exists in observations, equations, theories and laws. It is a part of apparatus that we currently have working best for the Universe understanding.
   Not sure about the part  "Time exists in observations". 

It is determined by an observer and perceived  as a human.
   Humans perceive the passage of the time a certain way, if that's what you mean.   We all instinctively understand the passage of the time.  However, child psychologists may argue about exactly when a child forms an awareness of time.

Think about that idea. Does light experience time?
    Using your earlier comments, a brief answer can be given:   A photon is not a human being.  So it does not experience time as a human being would.

From the perspective of a photon, it's emitted, and might exist for hundreds of trillions of years, but for the photon, there's zero time elapsed between when it's emitted and when it's absorbed again.
    Subject to limitations described earlier, this is true enough.   Supposedly Einstein thought about something similar - what it would be like to ride along with a photon.
    We could be annoying and state that the entire statement that it "might exist for hundreds of trillions of years" is meaningless unless we bring in another observer who is not travelling with the photon.  The photon may exist for hundreds of years as far as some other observer is concerned but as you stated, there is no elapsed time for the photon.

Best wishes to you.
Title: Re: What is Time? If there was no light would Time cease to be?
Post by: gem on 29/07/2021 19:14:11
Hi all


    Subject to limitations described earlier, this is true enough.   Supposedly Einstein thought about something similar - what it would be like to ride along with a photon.
    We could be annoying and state that the entire statement that it "might exist for hundreds of trillions of years" is meaningless unless we bring in another observer who is not travelling with the photon.  The photon may exist for hundreds of years as far as some other observer is concerned but as you stated, there is no elapsed time for the photon.

Best wishes to you.

If no time elapsed in the reference frame of the photon for  hundreds of trillions of years what universe is it travelling through and where was it generated that it’s energy never changes over time 🧐
Title: Re: What is Time? If there was no light would Time cease to be?
Post by: Just thinking on 29/07/2021 19:31:58
Time is a function of matter when the first appearance of matter came into existence time began. Matter can not exist without time, time can not exist without matter. Time and matter are working together to create a passage as the passage is the essence of time.
Title: Re: What is Time? If there was no light would Time cease to be?
Post by: Eternal Student on 29/07/2021 21:39:55
Hi.

If no time elapsed in the reference frame of the photon for  hundreds of trillions of years what universe is it travelling through and  that it’s energy never changes over time 🧐

   Difficult to provide an answer in the way you may want.
     The photon has no rest frame.  In any inertial frame of reference the photon is still going to be moving with speed c.  Could you clarify what you mean by "the reference frame of the photon" as you have used in your question.
   We can push the usual definition of a co-ordinate system.  We can imagine a co-ordinate systems in which the photon is at rest.  This is an extremely degenerate co-ordinate system.  It fails to separate many distinct events in spacetime that describes the universe for most other observers.  So we cannot answer your question "where was it generated" - a description in the co-ordinate system of the photon would fail to describe a unique location and time for most observers.
    You asked "what universe is it travelling through"?  The photon would usually be considered to be in the same universe but experiences it differently to most other observers (although the photon isn't human and doesn't experience anything like we do anyway).

Besy wishes to everyone.
Title: Re: What is Time? If there was no light would Time cease to be?
Post by: gem on 29/07/2021 23:00:28

 Difficult to provide an answer in the way you may want.
     The photon has no rest frame.  In any inertial frame of reference the photon is still going to be moving with speed c.  Could you clarify what you mean by "the reference frame of the photon" as you have used in your question.
   We can push the usual definition of a co-ordinate system.  We can imagine a co-ordinate systems in which the photon is at rest.  This is an extremely degenerate co-ordinate system.  It fails to separate many distinct events in spacetime that describes the universe for most other observers.  So we cannot answer your question "where was it generated" - a description in the co-ordinate system of the photon would fail to describe a unique location and time for most observers.
    You asked "what universe is it travelling through"?  The photon would usually be considered to be in the same universe but experiences it differently to most other observers (although the photon isn't human and doesn't experience anything like we do anyway).

Besy wishes to everyone.

I am referring to the reference frame of the photon and its continuing red/blue shifting of light/photons  as it/they loses/gains energy as it travels through gravitational fields

So a change in energy of a photon but no time elapsed according to the previous posts.
❓🤨



Title: Re: What is Time? If there was no light would Time cease to be?
Post by: Eternal Student on 30/07/2021 00:57:18
I am referring to the reference frame of the photon and its continuing red/blue shifting of light/photons  as it/they loses/gains energy as it travels through gravitational fields

So a change in energy of a photon but no time elapsed according to the previous posts.

   "The reference frame of the photon" - I still don't know what that is.  I wouldn't worry, it's probably my fault not yours.  It does not seem to be a valid reference frame.

  The photon changes energy in many reference frames but you're probably talking about the co-moving frame of the FRW universe.  I also think you've identified an important property of a photon's energy - it depends on the reference frame in which it is measured.  There isn't an inertial frame of reference in which a photon is at rest and so no way to determine it's energy in that frame.

   In reality, there is no way to put a clock on the back of a photon and let it ride along with the photon.  However, we can calculate the change in a quantity called "proper time" along the path taken through spacetime.  It's always 0 which is why it can be called a null path.  If a particle with mass takes some path through spacetime then the changes in "proper time" calculated along that path are shown as time progressing on a clock that travels with that particle.  This lead us to imagine that "proper time" is what a clock should measure as it travels along with a particle.  So we can imagine that if a clock was riding on the back of a photon then it would never tick (regardless of how much co-ordinate time you allow to pass).   That's all I think that TommyJ was trying to say.
    That's not the same as saying that co-ordinate time is not useful or important in describing the worldline of a photon, it's just that this co-ordinate time is not related to the proper time experienced by the photon.  So, I suggested that TommJ may need to bring in another observer to make sense of the statement that the photon exists "for hundreds of years" but it doesn't actually have to be another observer.  As an alternative TommyJ could just make it clear that this notion of "for hundreds of years" relates to co-moving or some other co-ordinate time.
Title: Re: What is Time? If there was no light would Time cease to be?
Post by: yor_on on 30/07/2021 09:42:56
Wouldn't it be better to suggest that a 'photon' is timeless ES? We still get 'light' or radiation from the Big Bang reaching us. Well, that 'photon fog' might not be it, but slightly later?
Title: Re: What is Time? If there was no light would Time cease to be?
Post by: Eternal Student on 30/07/2021 10:33:03
Hi.

Wouldn't it be better to suggest that a 'photon' is timeless ES?
   Yes, probably.
I seem to have spent too long discssing a minor issue that hardly connects to the original topic anyway.

Best wishes to you.
Title: Re: What is Time? If there was no light would Time cease to be?
Post by: yor_on on 30/07/2021 11:04:26
Heh, then you're like me, and like a lot of people here. We keep getting distracted :) And no, it made you ponder it right? Who knows what may come from that, new angles.

And wandering minds.
Title: Re: What is Time? If there was no light would Time cease to be?
Post by: gem on 30/07/2021 16:27:03
Hi all

I think the reference frame I’m referring to is
A proper one

https://en.m.wikipedia.org/wiki/Proper_frame

And given its proven a photon does gain/lose energy in a gravitational field without changing it speed, as postulated by general relativity, isn’t this change in energy/colour of a photon in need of a time dimension given its change in frequency.

https://en.m.wikipedia.org/wiki/Gravitational_redshift

Or is it just a slower zero time in a gravitational well 💩

Either way a transfer of energy occurs in the physical world.

Whether that’s in no time is a strange thing to prove
Title: Re: What is Time? If there was no light would Time cease to be?
Post by: Eternal Student on 30/07/2021 22:42:59
Hi Gem,

   I hope you are well.  This looks like an interesting reference.  Sadly, I've only had time to glance at it.  If you could explain it more fully, that would be great.  Does redshift occur in this "proper frame"? etc.

Either way a transfer of energy occurs in the physical world.
   That bit concerns me a little.  Gravitational redshift is one way of illustrating that on a cosmological scale, energy is not conserved.  The energy isn't transferred elsewhere, it's just gone.

Best wishes.
Title: Re: What is Time? If there was no light would Time cease to be?
Post by: gem on 31/07/2021 08:09:04
Hi all

So ES if energy is transferred I’m not sure how you believe it’s lost

https://en.m.wikipedia.org/wiki/Conservation_of_energy

Take a look at the Harvard tower experiment

https://en.m.wikipedia.org/wiki/Pound–Rebka_experiment

Very best wishes to you too 💋

( you cannot wake a person who pretends to be asleep 😴)

Title: Re: What is Time? If there was no light would Time cease to be?
Post by: Halc on 31/07/2021 12:14:12
Does redshift occur in this "proper frame"? etc.
It is meaningless to talk about a photon's proper frame.

Quote
Gravitational redshift is one way of illustrating that on a cosmological scale, energy is not conserved.  The energy isn't transferred elsewhere, it's just gone.
That is a property of a hyperbolic coordinate system. The conservation laws are not valid in such a coordinate system. Newton's laws don't hold either. Objects in motion tend to come to rest over time, without any force acting on them.
Similarly, light can move only at c in some coordinate systems (including hyperbolic), but not others such as Rindler or rotating coordinates. Each of these has or doesn't have conservation properties.

In the case of the universe, I question if the energy is gone since the potential is always going up, which is 'energy from nowhere'. Neither can be quantified, so 'total energy of the universe' isn't something we can say is going up or down.

Take a look at the Harvard tower experiment
Energy is perfectly conserved in this case, and simply illustrates a change in potential between the two local measurements.
Title: Re: What is Time? If there was no light would Time cease to be?
Post by: Eternal Student on 31/07/2021 16:49:11
Hi again gem and Halc,

I agree with most of what you've said, Halc and it's saved me a lot of time explaining it, so thank you very much for that.

The Cosmological redshift and conservation law(s) are interesting and may be worth more discussion:
That is a property of a hyperbolic coordinate system.
   Yes but you make it sound as if this is just an artifact of some arbitrary co-ordinate choice.  It sounds almost as if we could have conservation of energy in our real universe provided we all just choose the right co-ordinates.  At best, this can only be achieved locally everywhere and remains reasonably valid over some larger regions if they are well approximated by some metrics (e.g. flat space).

Some concrete examples:
    I don't think that spacetime described with the FLRW metric is diffeomorphic to Flat space with the Minkowski metric (unless you deliberately pick trivial cases like setting the scale factor a(t) ≡ constant ).   To say that another way, no co-ordinate transformation will reproduce the  FLRW metric   from the  Flat Minkowski metric after tensor transformation resulting from that change of co-ordinates.  A FRW universe isn't just Flat space in strange co-ordinates, it's geometry is intrinsically different.  In particular, we can't always find time-like Killing vectors in some manifolds that might describe our universe and as a result we don't always have a conserved quantity like total energy.
   You mention Rindler co-ordinates, which are a great way to reproduce some effects of gravity or GR within the framework of SR.  However, they are limited to creating effects of being in a uniform gravitational field.  A Transformation of the Minkowski metric under Rindler co-ordinates does not, for example, reproduce the Schwarzschild metric.
    The conservation of energy is such a staple belief of modern science that there have been attempts to re-define what we might mean by energy.  There is the Landau-Lifshitz Psuedo-energy tensor (for example) which is a thing that does allow for a conserved quantity like total energy but (as the name implies) it isn't a true tensor.   Some info here, if anyone wants it:  https://en.wikipedia.org/wiki/Stress%E2%80%93energy%E2%80%93momentum_pseudotensor
   OK, few people will want to read that and you (Halc) are probably busy enough doing other things.
- - - - - - -
   A less technical discussion of the Conservation of energy:
*  It's true and perfectly good for school.  It's probably still on most examination syllabuses.
*  If you're writing an essay you can argue about the lack of conservation of energy but make sure you include quotes like this one to back up this outlandish claim:
   Energy is not conserved -  Sean Carroll     https://www.preposterousuniverse.com/blog/2010/02/22/energy-is-not-conserved/

Best wishes to everyone.

Title: Re: What is Time? If there was no light would Time cease to be?
Post by: gem on 31/07/2021 20:34:55
Hi all

So Halc yes I was and always tend to look at the conservation energy as to whether what’s being discussed makes any sense.

Does redshift occur in this "proper frame"? etc.
It is meaningless to talk about a photon's proper frame.


Take a look at the Harvard tower experiment
Energy is perfectly conserved in this case, and simply illustrates a change in potential between the two local measurements.


In regards to the Harvard tower experiment I believe it’s a key piece of an
empirical jigsaw that overlaps into lots more than just demonstrating the change in gravitational potential.

For example the velocity of an object dropped from the tower ignoring air resistance would give the same “time dilation” as the difference recorded by an atomic clock top vs bottom of the tower. ( relativity and general relativity)

As well as the change in energy of the photon that corresponds to the delta h and rest mass in the field as per general relativity.
That’s why I took exception to zero time and a change in energy of a photon.

ES the links you provided stated areas of science that are still under discussion, so would not give them to much credit for example on one of the arguments links conservation of energy with conservation of momentum

I don’t believe it’s stated momentum cannot be created or destroyed
Like it’s stated for energy

Energy is the currency of the physical world
Title: Re: What is Time? If there was no light would Time cease to be?
Post by: Eternal Student on 01/08/2021 00:14:14
Hi Gem,
ES the links you provided stated areas of science that are still under discussion, so would not give them to much credit for example on one of the arguments links conservation of energy with conservation of momentum
    OK.  Advice taken.
I suppose Noether's theorem on symmetry and conserved quantities is still fairly new (published about 1918).  The appearance of examples where energy is not conserved in cosmology came some time after that.  Recent developments about dark energy spurred renewed interest since it appears that total energy can also increase (since space is expanding but the density of vaccum energy remains constant) - but this is obviously still controversial and not well understood.

I don’t believe it’s stated momentum cannot be created or destroyed
Like it’s stated for energy
    ... Seems like the conservation of momentum.  I may have misunderstood what you were trying to say here. 

Energy is the currency of the physical world
  It certainly was.  Cosmology is ready to challenge that.

Thanks for your time and attention.  I've enjoyed reading your thoughts and insight about the Pound-Rebka experiments.
Title: Re: What is Time? If there was no light would Time cease to be?
Post by: Halc on 01/08/2021 01:32:41
you make it sound as if this is just an artifact of some arbitrary co-ordinate choice.  It sounds almost as if we could have conservation of energy in our real universe provided we all just choose the right co-ordinates.
Energy (that of light, KE, some others) is definitely coordinate system dependent, and different kinds of coordinate systems have different conservation laws.  Is there a choice of CS that might preserve energy?  It would if an inertial frame could be used because inertial frames have that property, but such a frame is valid only in a zero-energy FLRW metric where the scalefactor would be linear. So that choice is an invalid one, and I cannot think of another candidate.

Quote
At best, this can only be achieved locally everywhere and remains reasonably valid over some larger regions if they are well approximated by some metrics (e.g. flat space).
...
    I don't think that spacetime described with the FLRW metric is diffeomorphic to Flat space with the Minkowski metric (unless you deliberately pick trivial cases like setting the scale factor a(t) ≡ constant ).
It works if a(t) ≡ linear, but only relative to an inertial coordinate system, not the resulting still trivial FLRW metric. Such a frame cannot be mapped to a non-linear metric such as our actual universe.

Quote
To say that another way, no co-ordinate transformation will reproduce the  FLRW metric   from the  Flat Minkowski metric after tensor transformation resulting from that change of co-ordinates.  A FRW universe isn't just Flat space in strange co-ordinates, it's geometry is intrinsically different.  In particular, we can't always find time-like Killing vectors in some manifolds that might describe our universe and as a result we don't always have a conserved quantity like total energy.
Sounds right.

Quote
You mention Rindler co-ordinates, which are a great way to reproduce some effects of gravity or GR within the framework of SR. However, they are limited to creating effects of being in a uniform gravitational field.
I wonder about that. In a normal gravitational field resulting from a point mass, the coordinate distance from the point-mass (or the proper distance to the event horizon, not the same distance) can be determined by multiple measurements at various altitudes, without knowing the mass of the body. In Rindler coordinates, the distance to the event horizon can be known with a single measurement.  Is there an equivalent thing with a uniform gravitational field? If not, then the two cases are not equivalent.

Anyway, I was just fishing for a coordinate system where energy is not necessarily conserved.

Quote
It's true and perfectly good for school.  It's probably still on most examination syllabuses.
But we quote the law when tearing apart posts by some of the trolls like Dave Lev who claims new matter is continuously created from existing mass and all the universe having grown from some unexplained low mass seed.
Title: Re: What is Time? If there was no light would Time cease to be?
Post by: Eternal Student on 01/08/2021 01:41:35
Hi.  I've just been re-reading some previous posts.

In the case of the universe, I question if the energy is gone since the potential is always going up, which is 'energy from nowhere'. Neither can be quantified, so 'total energy of the universe' isn't something we can say is going up or down.
    Yes, that's OK.  The total energy of the universe isn't something we can always quantify.   It's not necessarily that energy has gone overall.  It's just that it's gone from "this thing" and not easily identifed as having been transferred into some other thing.
     Vaccum energy also offers the possibility of an increase in total energy (since space expands, so there is more vaccum energy appearing).


STOPPED to read a new post that has appeared.
Title: Re: What is Time? If there was no light would Time cease to be?
Post by: gem on 01/08/2021 09:26:00
Hi all,

So ES

Hi Gem,
ES the links you provided stated areas of science that are still under discussion, so would not give them to much credit for example on one of the arguments links conservation of energy with conservation of momentum
    OK.  Advice taken.
I suppose Noether's theorem on symmetry and conserved quantities is still fairly new (published about 1918).  The appearance of examples where energy is not conserved in cosmology came some time after that.  Recent developments about dark energy spurred renewed interest since it appears that total energy can also increase (since space is expanding but the density of vaccum energy remains constant) - but this is obviously still controversial and not well understood.

I don’t believe it’s stated momentum cannot be created or destroyed
Like it’s stated for energy
    ... Seems like the conservation of momentum.  I may have misunderstood what you were trying to say here. 

Energy is the currency of the physical world
  It certainly was.  Cosmology is ready to challenge that.

Thanks for your time and attention.  I've enjoyed reading your thoughts and insight about the Pound-Rebka experiments.

As you stated
 (a little sarcastically about how new it is 💋)
Noether’s theorem relies on symmetry, which is a condition in reality doesn’t occur in many situations, for example we don’t have such symmetry on the surface of the earth and therefore our atmosphere doesn’t have such symmetry as per Noether’s theorem.

Hence momentum is continually created in this location, due to the non inertial conditions.

In regards to dark energy and matter
Whilst it’s still an area of such debate I wouldn’t be so arrogant as to discard conservation of energy.

😘
Title: Re: What is Time? If there was no light would Time cease to be?
Post by: Eternal Student on 01/08/2021 17:12:50
Hi Gem.

It wasn't meant to be sarcastic or cause offence. I think I may have mis-judged your age and I'm sorry about that.  I'm old enough to think that 1918 really is still quite new as regards science.   (If I could work this thing I'd put one of those emoticon things in here).  Anyway, no offence was intended.

Best wishes to you.
Title: Re: What is Time? If there was no light would Time cease to be?
Post by: TommyJ on 01/08/2021 18:22:48
As an alternative TommyJ could just make it clear that this notion of "for hundreds of years" relates to co-moving or some other co-ordinate time.
That would be the point, thank you.
To unwrap.
1. GR is a fundamental tool, where the specific metrics, the theories are being tested. Different approach aims to refine a specific metric.

2. Today cosmology goes through new cruces in energy conservation. As the universe is expanding. With the intrinsic energy per cubic centimeter of empty space. So, as space expands, it releases stored up gravitational potential energy, which converts into the intrinsic energy that fills the newly created volume.

3. We have gone through thousands of 'in-lab', LHC tests, and so forth. And moreover, what is important, developed observation and measurement systems (including in outer space ones) remarkable given that the family of metrics at the the order of data collected is in the trillions of bytes.

Any specific model is a challenge and a step forward in the same time. Researches go and check it with a suitable part trillion bytes.
And what is done, I like to mention, supernova appeared within a predicted month in 2020 (previously we could predict it with 10 000 years precision:)
Title: Re: What is Time? If there was no light would Time cease to be?
Post by: yor_on on 01/08/2021 21:45:50
Are you suggesting a 'photon' to be time dilated Gem? 

" And given its proven a photon does gain/lose energy in a gravitational field without changing it speed, as postulated by general relativity, isn’t this change in energy/colour of a photon in need of a time dimension given its change in frequency."

So we have light from the Big Bang, some short time after, that's how we get a 'time line' of the universe. If we now introduce the 'time dilation's' photons might, or might not, have meet 'propagating' 13.8 billion years we should get a very confusing situation as we measure that light, guessing its 'origin'. I've seen some comments comparing it to clocks (oscillators) at different elevations, talking about photons blue or red shifts as being due to gravitational time dilation's but I don't think that is correct.

In that Pound–Rebka experiment the whole idea was to test GR. And the equivalence principle. Not if photons had a time dilation

https://www.forbes.com/sites/startswithabang/2016/09/30/how-do-photons-experience-time/.
Title: Re: What is Time? If there was no light would Time cease to be?
Post by: yor_on on 01/08/2021 21:59:00
When it comes to a red or blue shift you could think of it as a comparison between frames of reference, because that is what it is. If a photon had a 'proper frame of reference' I think it would be, as Ethan states, 'time less', and it wouldn't change 'energy' intrinsically either. That last one should be able to be tested by knowing the source and then introduce gravity, a mass, between the source and yourself. Move the measuring device so that the mass no longer is in the path of the propagation and then compare the energy between photons. If they are the same then it didn't change anything for that photon deviating by a gravitational potential.
=

What might confuse it, thinking of 'photons' is the explanation in where you only find a wave, as in cosmological redshifts due to 'space' expanding. But light has a dual nature as proven in double slit experiments, it can be both and it seems as if 'space' or its 'expansion' forces it into one mode, relative the observer, which is weird. I agree to that one. But you don't need a propagation, there are other ways to define it too. Although we then move from main stream definitions. I suspect ES could  think up some description mathematically in where it will seem as 'propagating' to us inside SpaceTime although it doesn't from that mathematical perspective :) A 'toy universe' sort of.

maybe the easiest way is to think of light as a 'singularity', like what's inside a 'event horizon' of a black hole. It has the same properties. We just don't know what a 'photon' or light 'see'. Just as we don't know what's inside the event horizon.
Title: Re: What is Time? If there was no light would Time cease to be?
Post by: Colin2B on 02/08/2021 13:34:04
Are you suggesting a 'photon' to be time dilated Gem? 
....
In that Pound–Rebka experiment the whole idea was to test GR. And the equivalence principle. Not if photons had a time dilation
You beat me to those comments yor_on, I’m quite busy at the moment and limited time for respoding.
There’s a lot being said in these replies, but I’m not sure they are helping gem’s original question, or at least some of it is being lost in the quantity. I’m going to cover the main points to check he is ok with important points and particularly for our listeners, who might have faded out at some stage!

if energy is transferred I’m not sure how you believe it’s lost

Let’s drop back to basics.
Within a closed system energy is conserved within the frame you are using for measurement, it’s not lost or transferred when you measure from another frame or reference.
Simple example is KE, which is dependent on velocity, but velocity is relative so KE is frame relative.
Take PE, a book on a high shelf has a higher PE measured from the floor reference than it does measured from your head reference. So when it falls the transfer of PE to KE is greater at the floor than at your head.
So, to your main question:
So a change in energy of a photon but no time elapsed according to the previous posts.
❓🤨.

I am referring to the reference frame of the photon and its continuing red/blue shifting of light/photons  as it/they loses/gains energy as it travels through gravitational fields

That’s why I took exception to zero time and a change in energy of a photon.
Let’s ignore a photon for a moment and think of a massive object eg spaceship.
We all agree that when measured from a frame ‘at rest’ the spaceship has length contraction, time dilation, etc. However, in the “reference frame of the spaceship” (proper frame) it is at rest so it experiences no length contraction etc.
So, in the “reference frame of the photon” it is at rest and the bottom of the tower is rushing up to meet it, except that the distance to the floor appears to be zero from the photon’s perspective. This is why everyone is telling you that the reference frame of the photon is meaningless, you can’t use it for any practical calculation. It would also be of little use even if you could, because all the energy measurements we take are all from our frames at the top and bottom of the tower, we are not interested in the photon’s viewpoint.

In regards to the Harvard tower experiment I believe it’s a key piece of an
empirical jigsaw that overlaps into lots more than just demonstrating the change in gravitational potential.
Yes, there is a lot in the Pound Rebka experiment, not just local gravitational redshift, but the equivalence principle and energy conservation.

I don’t believe it’s stated momentum cannot be created or destroyed
Like it’s stated for energy
Again, for a local closed system, momentum is conserved and it is stated like it is for energy.
I think I know where you are thinking here. An interesting discussion if Petro had not completely misunderstood atmospheric circulation on a rotating planet.
Title: Re: What is Time? If there was no light would Time cease to be?
Post by: gem on 02/08/2021 22:10:20
Hi all,

So Yor_on/Colin, thanks both for your considerations,

I understand your concerns I believe there's a couple of separate issues,
So the problem I had with a photon being able to change energy in reality but with no passage of time as postulated by others seemed a nonsense.

So to try to cover this aspect if I may yor_on by using your analogy, and only use one reference frame to try to ascertain whether the photon actually loses/gains energy or if its just frame dependent.

Yor_on
Quote
When it comes to a red or blue shift you could think of it as a comparison between frames of reference, because that is what it is. If a photon had a 'proper frame of reference' I think it would be, as Ethan states, 'time less', and it wouldn't change 'energy' intrinsically either. That last one should be able to be tested by knowing the source and then introduce gravity, a mass, between the source and yourself. Move the measuring device so that the mass no longer is in the path of the propagation and then compare the energy between photons. If they are the same then it didn't change anything for that photon deviating by a gravitational potential.

Now if we use this arrangement and measure a photons energy/colour from a distance source with no mass between the source and observer, and then compare it to the colour to one with a photon that had passed by a mass that had traversed between the two points, ( ignoring the deflection due to change in direction due to the field )

in this scenario we will end up with both parts of the Harvard tower experiment.
IE we would have the blue shift of the photon as it traveled towards the mass and red shift as it traveled away.
So no change in colour detected by the observer even though it could be argued the colour had indeed changed towards blue and back to original colour as it passed by the mass.
However if the said mass was spinning at a reasonable velocity I think it would be reasonable to assume the red/blue shift would not be equal and the observer would detect an aspect of whats under discussion.

Now Colin, in regards to my statement of objecting as to the tying of momentum and energy in theory's put forward.     

gem;
Quote
Noether’s theorem relies on symmetry, which is a condition in reality doesn’t occur in many situations, for example we don’t have such symmetry on the surface of the earth and therefore our atmosphere doesn’t have such symmetry as per Noether’s theorem.

Hence momentum is continually created in this location, due to the non inertial conditions.

Now as you correctly recall aspects of this were raised previously

And given you were specific as to the conditions as to placing the conservation of momentum equal footing to conservation of energy,
I will restate, my understanding is: those conditions are not met on earth, neither is symmetry, in the accelerated conditions of a gravitational field, so surely its an error to tie the two as equivalent, in the physical world.

Given we cannot observe energy being created on earth but can observe the creation of momentum.

hence the conditions below are not met.

Colin
Quote
Again, for a local closed system, momentum is conserved and it is stated like it is for energy./quote]

Title: Re: What is Time? If there was no light would Time cease to be?
Post by: TommyJ on 03/08/2021 13:03:31
Einstein postulated that time was more like a river, ebbing and flowing depending on the effects of gravity and space-time. Even one second on Earth was not the same length of time everywhere in the universe.
He also mentioned that “The distinction between the past, present and future is only a stubbornly persistent illusion.”

Newton and Einstein did agree that time moves forward.
Everything in the universe wants to move from low to high entropy, or from uniformity to disorder, moving to the almost random arrangement of galaxies in the present day. This is known as the “arrow of time.”

When forced to summarize the general theory of relativity in one sentence: Time and space and gravitation have no separate existence from matter.
Title: Re: What is Time? If there was no light would Time cease to be?
Post by: Halc on 03/08/2021 14:07:33
Now if we use this arrangement and measure a photons energy/colour from a distance source with no mass between the source and observer, and then compare it to the colour to one with a photon that had passed by a mass that had traversed between the two points, ( ignoring the deflection due to change in direction due to the field )

in this scenario we will end up with both parts of the Harvard tower experiment.
Not really since there is no measurement taken near said mass. You're giving classic properties to what is a quantum thing. It is better to speak of a classic pulse of light rather than a photon since the latter does not have a position, path, or energy except when measured, and the former has all these things.

Quote
IE we would have the blue shift of the photon as it traveled towards the mass and red shift as it traveled away.
So no change in colour detected by the observer even though it could be argued the colour had indeed changed towards blue and back to original colour as it passed by the mass.
If we're talking about said pulse, then yes, agree.  The two pulses will be measured at the same energy assuming they were both emitted from locations of identical gravitational potential and had minimal deflection/refraction along the way, and had identical velocity relative to the observer. What they pass by along the way has no effect on the energy measured, but it does affect the time it takes to get there.

Quote
However if the said mass was spinning at a reasonable velocity I think it would be reasonable to assume the red/blue shift would not be equal and the observer would detect an aspect of whats under discussion.
No, not even then.

Quote
Given we cannot observe energy being created on earth but can observe the creation of momentum.
Did I miss something? Momentum is conserved in a closed system (inertial coordinates), so creation of momentum could be used to generate a perpetual motion machine.

So, in the “reference frame of the photon”
It is always a mistake to consider such an invalid frame. There is no time and only two dimensions of space. All the usual physical terms (velocity say) are meaningless.
Quote
it is at rest
Being at rest has no meaning if velocity is meaningless, which is different from zero velocity.
Quote
and the bottom of the tower is rushing up to meet it
It most certainly is not. The bottom of the tower is just there, as is the top. There is no spatial separation between the two since this dimension of space is nonexistent.
The light pulse has no proper energy, frequency, or wavelength, so speaking of any of these is also meaningless.

You realize all this, but you still made the statements above which is only going to confuse the readers.
Title: Re: What is Time? If there was no light would Time cease to be?
Post by: gem on 03/08/2021 23:54:08
Hi all,

So Halc

Quote
Not really since there is no measurement taken near said mass. You're giving classic properties to what is a quantum thing. It is better to speak of a classic pulse of light rather than a photon since the latter does not have a position, path, or energy except when measured, and the former has all these things.

I'm not sure its to relevant to get bogged down in this regard as its the energy of a photon under discussion whether its a single photon at a time or a stream of, it is still possible to derive the discrete quanta of the packet of electromagnetic radiation which is a photon.

Now on this point:

gem
Quote
However if the said mass was spinning at a reasonable velocity I think it would be reasonable to assume the red/blue shift would not be equal and the observer would detect an aspect of whats under discussion.


Halc
Quote
No, not even then.


So if this statement is correct
Einstein
Quote
As long as we restrict ourselves to purely mechanical processes in the realm where Newton's mechanics holds sway, we are certain of the equivalence of the systems K and K'. But this view of ours will not have any deeper significance unless the systems K and K' are equivalent with respect to all physical processes, that is, unless the laws of nature with respect to K are in entire agreement with those with respect to K'. By assuming this to be so, we arrive at a principle which, if it is really true, has great heuristic importance. For by theoretical consideration of processes which take place relatively to a system of reference with uniform acceleration, we obtain information as to the career of processes in a homogeneous gravitational field.


Then my previous comments of a rotating mass (accelerated motion) have electromagnetic pass by, at say above its equator in either direction there would a delta in the values of interaction of the electromagnetic radiation with the gravitational field/spacetime.

Moving on to,

gem
Quote
Given we cannot observe energy being created on earth but can observe the creation of momentum.

Halc
Quote
Did I miss something? Momentum is conserved in a closed system (inertial coordinates), so creation of momentum could be used to generate a perpetual motion machine.

Yes you did miss several things !   

I'm afraid Colin was very specific as was I.

you do not have a closed or inertial system and you do not have symmetry so your perpetual motion machine would be powered by sunlight, therefore not perpetual motion of the first or 2nd kind
 8)







Title: Re: What is Time? If there was no light would Time cease to be?
Post by: Halc on 04/08/2021 02:10:39
Then my previous comments of a rotating mass (accelerated motion) have electromagnetic pass by, at say above its equator in either direction there would a delta in the values of interaction of the electromagnetic radiation with the gravitational field/spacetime.
Yes, it would interact, but when it finally gets measured well away from said spinning/charged mass, no change to the energy of the light will be measured as compared to light that had not passed by such a mass. I have no idea how you think Einstein's comment suggests otherwise.


Quote
I'm afraid Colin was very specific as was I.
Colin says momentum is conserved, as do I.

Quote
you do not have a closed or inertial system
If the system is not closed, then momentum is being imported from outside, which is not the creation of momentum, merely the transfer of it. You seem to assert that it can be 'created'.

I say 'inertial system' because momentum is not conserved in many non-iniertial coordinate systems. For instance, relative to the comoving coordinate system, a rock moving at a peculiar velocity of 1000 m/sec will slow continuously in the absence of a force acting on it, losing both momentum and energy, neither of which are conserved in such a coordinate system.
Title: Re: What is Time? If there was no light would Time cease to be?
Post by: gem on 04/08/2021 23:56:47
Hi all,

So Halc in regards to the physical interaction of electromagnetic radiation with a rotating mass and your denial to a delta to photon energy, consider does gravitational lensing alter in this scenario ?

Now this

Halc ; you stated :
Colin says momentum is conserved, as do I.

also
Quote
If the system is not closed, then momentum is being imported from outside, which is not the creation of momentum, merely the transfer of it. You seem to assert that it can be 'created'.

I say 'inertial system' because momentum is not conserved in many non-iniertial coordinate systems. For instance, relative to the comoving coordinate system, a rock moving at a peculiar velocity of 1000 m/sec will slow continuously in the absence of a force acting on it, losing both momentum and energy, neither of which are conserved in such a coordinate system.

My bold in the above quote

So firstly, buoyancy and convection can only occur in a non inertial reference frame.  FACT

so take a body of fluid like oceans or atmosphere here on the Earth.
then consider, there is a particle or fluid parcel at each point that is assigned the average of the properties of atoms in a small region nearby. In particular, it has a density ρ and velocity v that depend on time t and position r. The momentum per unit volume is ρv (density x velocity)

Now lets consider, energy is applied to the air at the earths surface  via heat from the sun to said fluid. it will therefore change the density. and  the droplet accelerates. This acceleration occurs because the fluid in a given volume changes with time, in short the average force per unit volume changes inside the droplet, this known as  the gradient of the pressure.

Forces that can change the momentum of a droplet include the gradient of the pressure and gravity FACT

So no inertial reference frame, no closed system and No symmetry FACT

Title: Re: What is Time? If there was no light would Time cease to be?
Post by: Halc on 05/08/2021 06:12:46
does gravitational lensing alter in this scenario ?
To repeat it, but more exactly:
So long as the respective path lengths do not change over time, or change at the same rate, what they pass by along the way has no effect on the energy measured.
So to give an example where the energy does actually change, the one path is direct from the emitter to the detector, and the second path bounces off a moving mirror that changes the path length over time. In that case, the one pulse will be a different energy at the detector as compared to the other pulse.
A mass can act as a mirror of sorts, deflecting a light pulse at any angle you please. If said mass is not moving relative to the aparatus, then the two pulses will be measured the same regardless of spin, charge, lensing, whatever. Motion of the mass or mirror in a way that alters the pathlength is what will make a difference.

Quote
no closed system and No symmetry FACT
Again, you seem to suggest that momentum of a system can be created. Indeed I don't see a closed system identified in the quoted post. What's your system? Earth? It's momentum is constantly changing due to the pull of the sun and other objects. This is momentum transfer, not momentum creation. I don't see what heat expansion (or buoyancy of all things) has to do with it. Maybe your system is much smaller, but you've not identified it. Creation of momentum is the foundation of reactionless thrust, the invention of which would make anybody a billionaire.

I made no mention of a symmetry requirement for momentum conservation, so I don't know why you're dragging that  into it.

An inertial reference frame is an abstract choice and can be pretty much applied locally to anything, as a coordinate frame to anything, or to anything at all with gravity ignored. Hence your first and third FACTs seem not to be facts at all since I can effortlessly think of counterexamples.

So please be more concise and identify a system that can change its own system momentum without import of momentum from outside the system.
Title: Re: What is Time? If there was no light would Time cease to be?
Post by: Colin2B on 05/08/2021 18:25:33
You realize all this ......
Yes, I do know it. I was trying to use examples of the inconsistencies in gem’s argument and terminology, and did qualify, for eg, that the ground could not be rushing up and is why I said  “This is why everyone is telling you that the reference frame of the photon is meaningless”.

My comments on the posts were not intended as a criticism of anyone, but a reflection of the nature of the type of question in the OP, as I was in a hurry to catch the tide (and wifi) they might not have come across as I had intended, sorry if they were misunderstood. It is in the nature of this type of thread that posts are often longer and more technical, each generating a number of replies that could easily become topics in their own right making them difficult for some people to follow. That doesn’t mean the replies should be curtailed.
Note that our attempts to explain might be in vain despite the fact that you also stated “Energy (that of light, KE, some others) is definitely coordinate system dependent”. See:
use one reference frame to try to ascertain whether the photon actually loses/gains energy or if its just frame dependent.
Gem, this doesn’t make any sense to me and I can’t understand what you mean by actually or just frame dependent.. As we have said, KE measurements are always frame dependant, but we are so used to measuring KE relative to the earth’s surface, eg for a car, that we tend to forget. In the frame of the car (its proper frame) it is at rest and so its KE is zero. So we are only ever considering frame dependent measurements, which is what we ‘actually’ measure at the top and bottom of the tower, trying to do it from the point of view of the photon is meaningless.

in this scenario we will end up with both parts of the Harvard tower experiment.
IE we would have the blue shift of the photon as it traveled towards the mass and red shift as it traveled away.
So no change in colour detected by the observer even though it could be argued the colour had indeed changed towards blue and back to original colour as it passed by the mass.
However if the said mass was spinning at a reasonable velocity I think it would be reasonable to assume the red/blue shift would not be equal and the observer would detect an aspect of whats under discussion.
Again I don’t see how any of this relates to what’s under discussion. To add to comment by @Halc the only measurement you have is the final state. If a car travels down a motorway speeding up and slowing down the speed camera only measures its speed at that point; and that speed, and the related KE, are both measured relative to the ground. There is no other ‘actual’ speed. A similar situation occurs when light leaves a gravitational well and then enters another distant one, it is the nett change that results in what we measure.

no closed system and No symmetry FACT
My interest in your discussion on atmospheric momentum was only in trying to dispel Petro’s false ideas about atmospheric circulation on a rotating planet.
It’s worth noting that a more energetic atmosphere does not imply an increase in momentum.
Interestingly, I did discuss some ideas on transfer of atmospheric momentum with a colleague who studies it. He recommended some interesting papers, partly because I have a passing interest in meteorology and it has implications for weather systems. I’ll dig them out when I have time.
At the moment I’m sailing so my only interest in atmospherics is predicting the next wind shift.

So please be more concise and identify a system that can change its own system momentum without import of momentum from outside the system.
As @Halc says, but please do it in another thread.

Title: Re: What is Time? If there was no light would Time cease to be?
Post by: gem on 05/08/2021 23:30:04
Hi all.

Hi Colin,

yes point taken, things have gone away from the OP
 
In regards to the photon frame, if you look back I was taking issue with this aspect also, given the zero time aspect stated in the frame of the photon, as there clearly are net changes that have been measured to the energy of a photon as it goes up/down in a gravitational field.   

So Halc

Quote
An inertial reference frame is an abstract choice and can be pretty much applied locally to anything, as a coordinate frame to anything, or to anything at all with gravity ignored. Hence your first and third FACTs seem not to be facts at all since I can effortlessly think of counterexamples.


Ok Halc hold that thought for now, we can revisit this else where.  :)